SOLUTION BLUE BOOK.docx

October 15, 2020 | Author: Anonymous | Category: N/A
Share Embed Donate


Short Description

Download SOLUTION BLUE BOOK.docx...

Description

PIPE REVIEWER Find the work posses for a Helium gas at 200C *A. 609 KJ/kg B. 168 KJ/kg C. 229 KJ/kg Solution: W = m R T = m (8.314 / M) T For helium, M = 4 W/m = (8.314/4)(20 + 273) = 609 KJ/kg

D. 339 KJ/kg

Two kilogram of gas is confined in a 1 m3 tank at 200 kpa and 880C. What type of gas is in the tank? A. Helium *B. Ethane C. Methane D. Ethene Solution: PV=mRT 200 (1) = 2 (8.314/M)(88+273) M = 30 Therefore: the gas is Ethane (C2 H8) Find the enthalpy of Helium if its internal energy is 200 KJ/kg 144 KJ/kg B. 223.42 KJ/kg *C. 333.42 KJ/kg D. 168 KJ/kg Solution: R = 8.314/4 = 2.0785 K = 1.667 for helium Cp = k R/(k - 1) = 1.667(2.0785)/(1.667 – 1) = 5.195 KJ/kg-K Cv = R/(k – 1) = 2.0785/(1.667 – 1) = 3.116 KJ/kg – K ∆h/∆U = Cp/Cv ∆h/200 = 5.195/3.116 ∆h = 333.42 KJ/kg Compute the mass of a 2 m3 propane at 280 kpa and 40˚C. 6.47 kg B. 5.1 kg C. 10.20 kg *D. 9.47 kg Solution: Propane is C3 H3--------------M = 12(3) + 8(1) = 44 PV = m R T 280(2) = m (8.314/44)(40 + 273) m = 9.47 kg Compute the air flow in ft3/min of mechanical ventilation required to exhaust an accumulation of refrigerant due to leaks of the system capable of revolving air from the machinery room for a mass of 4 lbs refrigerant. *A. 200 B. 210 C. 220 D. 230 Solution: Q = 100 x G0.5 ft3/min Q = 100 x (4)0.5 = 200 ft3/min

Compute the free-aperture cross section in m2 for the ventilation of a machinery room if the mass of refrigerant is 9 kg. 0.314 *B. 0.414 C. 0.514 D. 0.614 Solution: F = 0.138 G0.5 m2 F = 0.138 (9)0.5 = 0.414 m2 A 29.53” x 39.37” pressure vessel contains ammonia with f = 0.041. Compute the minimum required discharge capacity of the relief device in kg/hr. 106.71 kg/hr B. 108.71 kg/hr *C. 110.71 kg/hr D. 112.71 kg/hr Solution: C = f D L, kg/s C = 0.041(29.53/39.37)(39.37/39.37) = 0.03075 kg/s (3600) = 110.71 kg/hr Compute the maximum length of the discharge pipe installed on the outlet of a pressure-relief device in feet for internal pipe diameter of 0.5 inch and rated discharge capacity is 8 lb/min of air. The rated pressure of relief valve is 16 psig. *A. 0.286 ft B. 0.386 ft C. 0.486 ft D. 0.586 ft Solution: P = Pg + Patm = 16 x 1.1 + 14.7 = 32.3 psia L = 9P2d5/16Cr2 = 9(32.3)2(0.5)5/16(8)2 = 0.286 ft A thermal power plant has a heat rate of 11,363 Btu/kw-hr. Find the thermal efficiency of the plant. 28% *B. 30% C. 34% D. 40% Solution: eth = 3412/Heat rate = 3412/11,363 = 30% What is the hydraulic gradient of a 1 mile, 17 inches inside diameter pipe when 3300 gal/min of water flow with f = 0.03. *A. 0.00714 B. 0.00614 C. 0.00234 D. 0.0018 Solution: v = (3300/7.481)/(π/4)(17/12)2(60) = 4.66 ft/s L = 1 mile = 5280 ft hL = fLv2/2_D = 0.03(5280)(4.66)2/2(32.2)(17/12) = 37.7 ft Hydraulic gradient = 37.7/5280 = 0.007.14 Find the loss of head in the pipe entrance if speed of flow is 10 m/s. 5.10 m B. 10.2 m C. 17.4 m *D. 2.55 m Solution: Loss at entrance = 0.5 (v2/2g) = 0.5 [102 / 2(9.81)] = 2.55 m Wet material, containing 220% moisture (dry basis) is to be dried at the rate of 1.5 kg/s in a continuous dryer to give a product containing 10% (dry basis) . Find the moisture removed, kg/hr *A. 3543.75 kg/hr B. 3513.75 kg/hr C. 3563.75 kg/hr D. 3593.75 kg/hr

Solution: Solid in wet feed = solid in dried product [1/(1 + 2.2)](1.5) = [1/(1 + 0.1)](x) x = 0.5156 kg/s (total dried product) Moisture removed = 1.5 – 0.5156 = 0.984 kg/s = 3543.75 kg/hr Copra enters a dryer containing 70% moisture and leaves at 7% moisture. Find the moisture removed on each pound on solid in final product. 6.258 lb B. 1.258 lb C. 4.258 lb *D. 2.258 lb Solution: Solid in wet feed = solid in dried product 0.3x = 1 x = 3.333 lbs 1 = 0.93y y = 1.07527 lb Moisture removed = x – y = 3.333 – 1.07527 = 2.258 lb

A 1 m x 1.5 m cylindrical tank is full of oil with SG = 0.92. Find the force acting at the bottom of the tank in dynes. 106.33 x 103 dynes B. 106.33 x 104 dynes C. 106.33 x 105 dynes *D. 106.33 x 106 dynes Solution: P = w h = (0.92 x 9.81) (1.5) = 13.5378 kpa F = PA = 13.5378(π/4 x 12) = 10.632 KN = 10,632.56 N x 10,000 dynes/N F = 106.33 x 106 dynes Find the pressure at the 100 fathom depth of water in kpag. *A. 1,793.96 kpag B. 1,893.96 kpag C. 1,993.96 kpag kpag Solution: H = 100 fathom x 6 = 600 ft P = w h = (600/3.281)(9.81) = 1,793.96 kpag

D. 1,693.96

Find the depth in furlong of the ocean (SG = 1.03) if the pressure at the sea bed is 2,032.56 kpag. *A. 1 B. 2 C. 3 D. 4 Solution: P=wh 2,032.56 = (1.03 x 9.81) h h = 201.158 m x 3.281 ft/m x 1 yd/3ft x 1 furlong/220yd = 1 furlong Find the mass of 10 quartz of water. 10.46 kg *B. 9.46 kg C. 11.46 kg D. 8.46 kg Solution: V = 10 quartz x 1gal/4quartz x 3.785li/1gal x 1m 3/1000li

V = 0.0094625 x 10-3m3 w = m/V 1000 = m/0.0094625 x 10-3 m = 9.46 kg Find the mass of carbon dioxide having a pressure of 20 psia at 200˚F with 10 ft 3 volume. 1.04 lbs B. 1.14 lbs *C. 1.24 lbs D. 1.34 lbs Solution: PV = m R T (20 x 144)(10) = m (1545/44)(200 + 460) m = 1.24 lbs Find the heat needed to raise the temperature of water from 30˚C to 100˚C with 60% quality. Consider an atmospheric pressure of 101.325 kpa. Use the approximate enthalpy formula of liquid. 293.09 KJ/kg B. 1,772.90 KJ/kg C. 1,547.90 KJ/kg *D. 1,647.29 KJ/kg Solution: At 100˚C ht = cp t = 4.187 (100) = 418.7 KJ/kg htg = 2257 KJ/kg h2 = h1 + x htg = 418.7 + 0.60(2257) = 1,772.9 KJ/kg Q = 1(4.187)(100 – 30) + 1(1772.9 – 418.7) = 1,647.29 KJ/kg Find the enthalpy of water at 212˚F and 14.7 psi if the dryness factor is 30%. Use the approximate enthalpy formula of liquid. 461 Btu/lb *B. 471 Btu/lb C. 481 Btu/lb D. 491 Btu/lb Solution: ht = (˚F – 32) = (212 – 32) = 180 Btu/lb htg = 970 Btu/lb h = ht + x htg h = 180 + 0.3(970) = 471 Btu/lb An air compressor consumed 1200 kw-hr per day of energy. The electric motor driving the compressor has an efficiency of 80%. If indicated power of the compressor is 34 kw, find the mechanical efficiency of the compressor. 117.65 % B. 75 % *C. 85 % D. 90 % Solution: P/m = 1200kw-hr/24 hrs = 50 kw BP = 50(0.80) = 40 kw em = 34/40 = 85 % A refrigeration system consumed 28,000 kw-hr per month of energy. There are 20 % of energy is lost due to cooling system of compressor and motor efficiency is 90 %. If COP of the system is 6, find the tons of refrigeration of the system. 43.15 TR B. 46.15 TR *C. 49.15 TR D. 41.15 TR

Solution: P/m = 28,800/(24 x 30) = 40 kw BP = 40(0.90) = 36 kw Wc = 36(1 – 0.20) = 28.80 kw COP = RE/Wc 6 = RE/28.80 RE = 172.8/3.516 = 49.15 TR A 23 tons refrigeration system has a heat rejected of 100 kw. Find the energy efficiency ratio of the system. 13.42 *B. 14.42 C. 15.42 D. 16.42 Solution: QR = RE + Wc 100 = 23(3.516) + W c Wc = 19.132 kw COP = RE/Wc = (23 x 3.516) / 19.132 = 4.32 EER = 3.412 COP = 3.412(4.23) = 14.42 A 200 mm x 250 mm, 8-cylinder, 4-stroke diesel engine has a brake power of 150 kw. The mechanical efficiency is 80%. If two of the cylinders were accidentally cut off, what will be the new friction power? A. 31.50 kw B. 33.50 kw C. 35.50 kw *D. 37.50 kw Solution: em = BP/IP 0.8 = 150/IP IP = 187.5 kw FP1 = IP – BP = 187.5 – 150 = 37.50 kw FP1 = FP2 = 37.50 kw If the energy efficiency ratio of the refrigeration system is 12.6, what is the COP of the system? *A. 3.69 B. 4.23 C. 5.92 D. 6.83 Solution: EER = 3.412 COP 12.6 = 3.412 COP COP = 3.69 An air compressor has a power of 40 kw at 4 % clearance. If clearance will increase to 70 %, what is the new power? 70 kw *B. 40 kw C. 53kw D. 60 kw Solution: The power of compressor will not be affected with the changes in clearance. Therefore the power will still be 40 kw. What is the approximate value of temperature of water having enthalpy of 208 Btu/lb? 138.67 ˚C *B. 115.55 ˚C C. 258.67 ˚C D. 68.67 ˚C

Solution: h = ˚F – 32 Find the work posses for a Helium gas at 20°C. *A. 609 KJ/Kg B. 168 KJ/Kg C. 229 KJ/Kg Solution: W = m R T = m (8.314/M) T For Helium, M = 4 W/m = (8.314/4)(20 + 273) = 609 KJ/Kg

D. 339 KJ/Kg

Two kilogram of gas is confined in a 1 m3 tank at 200 kpa and 88˚C. What type of gas is in the tank? Helium *B. Ethane C. Methane D. Ethene Solution: PV=mRT 200 (1) = 2 (8.314/m)(88 + 273) M = 30 Therefore: the gas is Ethane (C2H6) Find the enthalpy of Helium if its internal energy is 200 KJ/kg. 144 KJ/kg B. 223.42 KJ/kg *C. 333.42 KJ/kg Solution: R = 8.314/4 = 2.0785 K = 1.667 for helium cp = k R/(k – 1) = 1.667(2.0785)/(1.667 – 1) = 5.195 KJ/kg-K cv = R/(k – 1) = 2.0785/(1.667 – 1) = 3.116 KJ/kg-K ∆h/∆U = cp/cv ∆h/200 = 5.195/3.116 ∆h = 333.42 KJ/kg

D. 168 KJ/kg

Compute the mass of a 2 m3 propane at 280 kpa and 40˚C. 6.47 kg B. 5.1 kg C. 10.20 kg *D. 9.47 kg Solution: Propane is C3H6------------------------M = 12(3) + 8(1) = 44 PV = m R T 280(2) = m (8.314/44)(40 + 273) m = 9.47 kg Compute the air flow in ft3/min of mechanical ventilation required to exhaust an accumulation of refrigerant due to leaks of the system capable of revolving air from the machinery room for a mass of 4 lbs refrigerant. *A. 200 B. 210 C. 220 D. 230 Solution: Q = 100 x G0.5 ft3/min Q = 100 x (4)0.5 = 200 ft3/min

Compute the free-aperture cross section in m2 for the ventilation of a machinery room if the mass of refrigerant is 9 kg. 0.314 *B. 0.414 C. 0.514 D. 0.613 Solution: F = 0.138 G0.5 , m2 F = 0.138(9)0.5 = 0.414 m2 A 29.53” x 39.37” pressure vessel contains ammonia with f = 0.041. Compute the minimum required discharge capacity of the relief device in kg/hr. 106.71 kg/hr B. 108.71 kg/hr *C. 110.71 kg/hr D. 112.71 kg/hr Solution: C = f D L, kg/s C = 0.041(29.53/39.37)(39.37/39.37) = 0.03075 kg/s (3600) = 110.71 kg/hr Compute the maximum length of the discharge pipe installed on the outlet of a pressure-relief device in feet for internal pipe diameter of 0.5 inch and rated discharge capacity is 8lb/min of air. The rated pressure of relief valve is 16 psig. *A. 0.286 ft B. 0.386 ft C. 0.486 ft D. 0.586 ft Solution: P = Pg + Patm = 16 x 1.1 + 14.7 = 32.3 psia L = 9P2d5/16Cr2 = 9(32.3)2(0.5)5/16(8)2 = 0.286 ft A thermal power plant has a heat rate of 11,363 Btu/kw-hr. Find the thermal efficiency of the plant. 28 % *B. 30 % C. 34 % D. 40 % Solution: eth = 3412/heat rate = 3412/11,363 = 30% What is the hydraulic gradient of a 1 mile, 17 inches inside diameter pipe when 3300 gal/min of water flow with f…. 0.03 *A. 0.00714 B. 0.00614 C. 0.00234 D. 0.00187 Solution: v = (3300/7.481)/(π/4)(17/12)2(60) = 4.66 ft/s L = 1 mile = 5280 ft hL =fLv2/2gD = 0.03(5280)(4.66)2/2(32.2)(17/12) = 37.7 ft Hydraulic gradient = 37.7/5280 = 0.00714 Find the loss of head in the pipe entrance if speed of flow is 10 m/s. 5.10 m B. 10.2 m C. 17.4 m *D. 2.55 m Solution: Loss at entrance = 0.5 (v2/2g) = 0.5[102/ 2(9.81)] = 2.55 m Wet material, containing 220 % moisture (dry basis) is to be dried at the rate of 1.5 kg/s in a continuous dryer to give a product containing 10% (dry basis). Find the moisture removed, kg/hr.

*A. 3543.75 kg/hr B. 3513.75 kg/hr C. 3563.75 kg/hr 3593.75 kg/hr Solution: Solid in wet feed = solid in dried product [1/(1 + 2.2)](1.5) = [1/(1 + 0.1)](x) x = 0.5156 kg/s (total dried product) Moisture removed = 1.5 – 0.5156 = 0.984 kg/s = 3543.75 kg/hr

D.

Copra enters a dryer containing 70% moisture and leaves at 7% moisture. Find the moisture removed on each pound of solid in final product. 6.258 lb B. 1.258 lb C. 4.258 lb *D. 2.258 lb Solution: Solid in wet feed = solid in dried product 0.3x = 1 x = 3.333 lbs 1 = 0.93y y = 1.07527 lb Moisture removed = x – y = 3.333 – 1.07527 = 2.258 lbs A 1 m x 1.5 m cylindrical tank is full of oil with SG = 0.92. Find the force acting at the bottom of the tank in dynes. 106. 33 x 103 dynes B. 106.33 x 104 dynes C. 106.33 x 105 dynes 6 *D. 106.33 x 10 dynes Solution: P = w h = (0.92 x 9.81)(1.5) = 13.5378 kpa F = PA = 13.5378(π/4 x 12) = 10.632 KN = 10,632.56 N x 10,000 dynes/N F = 106.33 x 106 dynes Find the pressure at the 100 fathom depth of water in kpag. *A. 1,793.96 kpag B. 1,893.96 kpag C. 1,993.96 kpag 1,693.96 kpag Solution: H = 100 fathom x 6 = 600 ft P = w h = (600/3.281)(9.81) = 1,793.96 kpag

D.

Find the depth in furlong of the ocean (SG = 1.03) if the pressure at the sea bed is 2,032.56 kpag. *A. 1 B. 2 C. 3 D. 4 Solution: P=wh 2,032.56 = (1.03 x 9.81) h h = 201.158 m x 3.281 ft/m x 1 yd/3ft x 1 furlong/220yd = 1 furlong Find the mass of 10 quartz of water. 10.46 kg *B. 9.46 kg Solution:

C. 11.46 kg

D. 8.46 kg

V = 10 quartz x 1gal/4quartz x 3,785 li/1gal x 1m3/1000 li V = 0.0094625 x 10-3m3 w = m/V 1000 = m/0.0094625 x 10-3 m = 9.46 kg Find the mass of carbon dioxide having a pressure of 20 psia at 200°F with 10 ft 3 volume. A. 1.04 lbs B. 1.14 lbs *C. 1.24 lbs D. 1.34 lbs Solution: PV = m R T (20 x 144)(10) = m (1545/44)(200 + 460) m = 1.24 lbs

Find the heat needed to raise the temperature of water from 30°C to 100°C with 60% quality. Consider and atmospheric pressure of 101.325 kpa. Use the approximate enthalpy formula of liquid. A. 293.09 KJ/kg B. 1,772.90 KJ/kg C. 1,547.90 KJ/kg *D. 1,647.29 KJ/kg Solution: At 100°C hf = Cp t = 4.187(100) 418.7 KJ/kg hfg = 2257 KJ.kg h2 = hf + xhfg = 418.7 + 0.60(2257) = 1,772.9 KJ/kg Q = 1(4.187)(100-30) + 1(1772.9 – 418.7) = 1,647.20 KJ/kg Find the enthalpy of water at 212˚F and 14.7 psi if the dryness factor is 30%. Use the approximate enthalpy formula of liquid. 461 Btu/lb *B. 471 Btu/lb C. 481 Btu/lb D. 491 Btu/lb Solution: hf = (˚F – 32) = (212 – 32) = 180 Btu/lb hfg = 970 Btu/lb h = hf + x hfg h = 180 + 0.3(970) = 471 Btu/lb An air compressor consumed 1200 kw-hr per day of energy. The electric motor driving the compressor has an efficiency of 80 %. If indicated power of the compressor is 34 kw, find the mechanical efficiency of the compressor. 117.65 % B. 75% *C. 85% D. 90% Solution: Pim = 1200kw-hr/24 hrs = 50 kw BP = 50(0.80) = 40 kw em = 34/40 = 85%

A refrigeration system consumed 28,800 kw-hr per month of energy. There are 20 % of energy is lost due to cooling system of compressor and motor efficiency is 90%. If COP of the system is 6, find the tons of refrigeration of the system. 43.15 TR B. 46.15 TR *C. 49.15 TR D. 41.15 TR Solution: Pim = 28,800/(24 x 30) = 40 kw BP = 40 (0.90) = 36 kw Wc = 36(1 – 0.20) = 28.80 kw COP = RE/Wc 6 = RE/28.80 RE = 172.8/3.516 = 49.15 TR A 23 tons refrigeration system has a heat rejected of 100 kw. Find the energy efficiency ratio of the system. 13.42 *B. 14.42 C. 15.42 D.16.42 Solution: QR = RE + Wc 100 = 23(3.516) + W c Wc = 19.132 kw COP = RE/Wc = (23 x 3.516) / 19.132 = 4.23 EER = 3.412 COP = 3.412(4.23) = 14.42 A 200 mm x 250 mm, 8-cylinder, 4-stroke diesel engine has a brake power of 150 kw. The mechanical efficiency is 80 %. If two of the cylinders were accidentally cut off, what will be the new friction power? 31.50 kw B. 33.50 kw C. 35.50 kw *D. 37.50 kw Solution: em = BP/IP 0.8 = 150/IP IP = 187.5 kw FP1 = IP – BP = 187.5 – 150 = 37.50 kw FP1 = FP2 = 37.50 kw If the energy efficiency ratio of the refrigeration system is 12.6, what is the COP of the system? *A. 3.69 B. 4.23 C. 5.92 D. 60 kw Solution: EER = 3.412 COP 12.6 = 3.412 COP COP = 3.69 An air compressor has a power of 40 kw at 4% clearance. If clearance will increase to 7%, what is the new power? 70 kw *B. 40 kw C. 53 kw D. 60 kw Solution:

The power of compressor will not be affected with the changes in clearance. Therefore power will still be 40 kw. What is the approximate value of temperature of water having enthalpy of 208 Btu/lb? 138.67˚C *B. 115.67˚C C. 258.67˚C D. 68.67˚C Solution: h = ˚F – 32 208 = F – 32 ˚F = 240 ˚F = 115.55 ˚C Convert 750˚R to ˚K 390.33 ˚K B. 395.33 ˚K Solution: ˚R = 1.8 ˚K 750 = 1.8 ˚K ˚K = 416.667

C. 410.33 ˚K

*D. 416.33 ˚K

An otto cycle has a compression ratio of 8. Find the pressure ratio during compression. *A. 18.38 B. 16.38 C. 14.38 D. 12.38 Solution: P1V1k = P2V22 (V1/V2)k = (P2/P1) rkk = rp rp = (8)1.4 = 18.38 A diesel cycle has a cut off ratio of 2.5 and expansion ratio of 4. Find the clearance of the cycle. 9.11 % B. 5.55 % *C. 11.11 % D. 15.15 % Solution: rk = rc re rk = 2.5(4) = 10 rk = (1 + c)/c 10 = (1 + c)/c c = 11.11 % A dual cycle has an initial temperature of 30 ˚C. The compression ratio is 6 and the heat addition at constant volume process is 600 KJ/kg. If cut-off ratio is 2.5, find the maximum temperature of the cycle. 3638.50 ˚C *B. 3365.50 ˚C C. 3565.50 ˚C D. 3965.50 ˚C Solution: T2 = T1 rkk-1 = (30 +273)(6)1.4-1 = 620.44 ˚K QAV = m cv (T3 – T2) 600 = 1(0.7186)(T3 – 620.44) T3 = 1455.396 ˚K rc = T4/T3 2.5 = T4/1455.396

T4 = 3638.49 ˚K = 3365.50 ˚C A three stages air compressor compresses air from 100 kpa to 1000 kpa. Find the intercooler pressure between the first and second stage. 505.44 kpa B. 108.44 kpa C. 316.23 kpa *D. 215.44 kpa Solution: Px = (P12P2)1/3 Px = [(100)2(1000)]1/3 = 215.44 kpa A 10-stages air compressor compresses air from 100 kpa to 800 kpa. Find the intercooler pressure between 1st and 2nd stage. 282.84 kpa B. 113.21 kpa *C. 123.11 kpa 333.51 kpa Solution: Px = (P1s-1P2)1/s Px = [(100)10-1(1000)]1/10 = 123.11 kpa

D.

A 3-stages air compressor compresses air from 100 kpa to 700 kpa. Find the intercooler pressure between the 2nd and 3rd stage. *A. 365.88 kpa B. 375.88 kpa C. 385.88 kpa D. 395.88 kpa Solution: Px = (P12P2)1/3 Px = [(100)2(700)]1/3 = 191.28 kpa Px/P1 = Py/Px Py = Px2/P1 = (191.28)2/100 = 365.88 kpa Carnot cycle A, B and C are connected in series so that the heat rejected from A will be the heat added to B and heat rejected from B will be added to C, each cycle operates between 30 ˚C and 400 ˚C. If heat added to A is 1000 kw, find the work output of C. *A. 111.44 kw B. 549.78 kw C. 247.53 kw D. 141.89 kw Solution: e1 = e2 = e3 = (400 – 30)/(400 + 273) = 54.98 % e1 = W 1/QA1 = (QA1 – QR1)/QA1 0.5498 = (1000 – QR1)/1000 QR1 = 450.22 = QA2 0.5498 = (450.22 – QR2)/450.22 By heat balance: Qgain = Qloss mw cp (tb- ta) = mg cpg (t2-t1) (0.30)(4.187)( tb – 15) = (0.5)(1.0717)(150 – 80) tb = 44.86 oC A 350 mm X 450 mm steam engine running at 280 rpm has an entrance steam condition of 2 Mpa and 230 oC and exit at 0.1 Mpa. The steam consumption is 2,000

kg/hr and mechanical efficiency is 85%. If indicated mean effective pressure is 600 Kpa, determine brake thermal efficiency. At 2 Mpa and 230 oC (Table 3): h1 = 2849.6 s1 = 6. 4423 At 0.1 Mpa: sf = 1.3026 hf = 417.46 sfg = 6.0568 hfg = 2258 hf2 = 417.46 KJ/kg A. 23.34% *B. 15.25% C. 14.16% D. 27.34% SOLUTION: VD = 2[3.1416/4 (0.35) 2 (0.45)(280/60)]= 0.4041 m3/sec Indicated Power = Pmi x VD = 600 x 0.4041 =242.45 KW Brake Power = IP (em) = 242.45 (0.85) = 206.08KW Brake Power 206.08 = 15.25% etb = ms (h1-hf2) = (2000/3600)(2849.6 – 417.46) A steam turbine receives 5,000 kg/hr of steam at 5 Mpa and 4000oc and velocity of 30 m/sec. It leaves the turbine at 0.06 Mpa and 85% quality and velocity of 15 m/sec. Radiation loss is 10,000 KJ/hr. Find he KW developed. At 5 Mpa and 400oC: h1 = 3195.7 KJ/kg s1 =6.6459 At 0.006 Mpa: hf = 151.53 hfg = 2415.9 A. 1273.29 B. 2173.29 *C. 1373.60 D. 7231.29 SOLUTION: h2 = hf + xhfg = 151.53 + 0.85(2415.9) = 2205.045 KJ/ kg KE1 = ½ m v2 = ½ (5,000/3600)(30)2 = 625 W = 0.625 KW KE2 = ½ m v2 = ½ (5,000/3600)(15)2 = 156.25 W = 0.15625 KW By energy balance: KE1 + mh1 = KE2 + mh2 + Q + W W = (KE1 – KE2) + m(h1-h2) – Q 5000 10,000 W = (0.625 – 0.156) + (3600)(3195.7 – 2205.045) – 3600 = 1373.60 KW A steam turbine with 85% stage efficiency receives steam at 7 Mpa and 550oC and exhausts as 20 Kpa. Determine the turbine work. At 7 Mpa and 550oC: h1 = 3530.9 Kj/kg s1 = 6.9486 At 20 Kpa (0.020 Mpa): sf = 0.8320 hf = 251.4 sfg = 7.0766 hfg = 2358.3 1,117 KJ/kg B. 1,132 KJ/ kg C. 1,123.34 KJ/ kg *D. 1,054.95 KJ/kg SOLUTION: s1 = s2 = sf + xsfg 6.9486 = 0.8320 + x(7.0766) x = 0.8643 h2 = 251.40 + 0.8643(2358.3) = 2289.78 KJ/kg h1 - h2a eST = h1 – h2 0.85 =3530.9 – h2a 3530.9 – 2289.78 h2a = 2475.95 KJ/kg WT = h1 = h2a = 3530.9 – 2475.95 = 1,054.95 KJ/kg

A steam turbine with 80% stage efficiency receives steam at 7 Mpa and 550oC and exhaust as 20 Kpa. Determine the quality at exhaust. At 7 Mpa and 550oC: h1 = 3530.9 Kj/kg s1 = 6.9486 At 20 Kpa (0.020 Mpa): sf = 0.8320 hf = 251.4 *A. 96.96% B. 76.34% C. 82.34% D. 91.69% SOLUTION: sfg = 7.0766 hfg = 2358.3 s1 = s2 = sf + sfg 6.9486 = 0.8320 + x(7.0766) x = 0.8643 h2 = 251.40 + 0.8643(2358.3) = 2289.78 KJ/kg nST = h1 – h2a h1 – h2 0.80 = 3530.9 – h2a 3530.9 – 2289.78 h2a = 2538.004 KJ/kg h2a = hf + x hfg 2538.004 = 251.40 + x (2358.3) x = 96.96% A 16,000KW geothermal plant has a generator efficiency and turbine efficiency of 90% and 80%., respectively if the quality after throttling is 20% and each well discharges 400, 000 kg/hr, determine the number of wells are required to produce if the charge of enthalpy if the change of enthalpy at entrance and exit of turbine is 500KJ/kg. 4 wells *B. 2 wells C. 6 wells D. 8 wells SOLUTION: WT = ms(h3 – h4) 16,000 = ms (500) 0.9(0.8) ms = 44.44 kg/sec ms = 160,000 kg/hr 160,000 = 0.20 mg mg = 800,000 kg/hr No. of wells = 800,000/400,000 = 2 wells A liquid dominated geothermal plant with a single flash separator receives water at 204oC. The separator pressure is 1.04 Mpa. A direct contact condenser operates at 0.034 Mpa. The turbine has a polytropic efficiency of 0.75. For a cycle output of 60 MW, what is the mass flow rate of the well-water in kg/s? At 204oC: hf = 870.51 KJ/kg At 1.04 Mpa: hf = 770.38 hfg = 2009.2 hg = 2779.6 sg = 6.5729 At 0.034 MPa: hf = 301.40 hfg = 2328.8 sf = 0.9793 sfg = 6.7463 *A. 2,933 B. 2,100 C. 1,860 D. 2,444 SOLUTION: h3 = hg at 1.04 MPa = 2779.6 KJ/kg Solving for h4: s3 = s4 = sf + xsfg

6.5729 = 0.9793 + x4(6.7463) x4 = 0.829 h4 = 301.4 + 0.829(2328.8) = 2232.3 KJ/kg WT = ms (h3 – h4) 60,000 = ms (2779.6 – 2232.3) 0.75 ms = 146.17 kg/sec Solving for x2: (h1 = h2) h1 = h2 = hf + xhfg 870.51 = 770.38 + x2(2009.2) x2 = 0..049836 ms = x mg 146.17 = 0.049836 mg mg = 2,933.06 kg/sec An engine-generator rated 9000 KVA at 80% power factor, 3 phase, 4160 V has an efficiency of 90%. If overall plant efficiency is 28%, what is the heat generated by the fuel. 18,800 KW B. 28,800 KW C. 7500 KW *D. 25,714 KW SOLUTION: Gen. Output = pf x KVA = 0.8 x 9000 = 7200 KW eoverall= Gen. Output Qg 0.28 = 7200/Qg Qg = 25,714.28 KW The indicated thermal efficiency of a two stroke diesel engine is 60%. If friction power is 15% of heat generated, determine the brake thermal efficiency of the engine. 43% *B. 45 % C. 36% D. 37% SOLUTION: ne = IP/ Qg 0.60 = IP/Qg IP = 0.60 Qg BP = IP- FP = 0.60Qg – 0.15Qg = 0.45Qg etb = BP/Qg = 0.45Qg/Qg = 45% A 305 mm x 457 mm four stroke single acting diesel engine is rated at 150 KW at 260 rpm. Fuel consumption at rated load is 0.56 kg/KW-hr with a heating value of 43,912 KJ/kg. Calculate brake thermal efficiency 10.53% B. 27.45% *C. 14.64% D. 18.23% SOLUTION: mf = 0.56 kg/KW-hr x 150 KW = 84 kg/hr = 0.0233 kg/sec Brake thermal efficiency = A waste heat recovery boiler produces 4.8 Mpa(dry saturated) steam from 104°C feedwater. The boiler receives energy from 7 kg/sec of 954°C dry air. After passing

through a waste heat boiler, the temperature of the air is has been reduce to 343°C. How much steam in kg is produced per second? Note: At 4.80 Mpa dry saturated, h = 2796. 1.30 B. 0.92 *C. 1.81 D. 3.43 SOLUTION: hf = approximate enthalpy of feedwater hf = Cpt hf = 4.187(104) hf = 435.45 KJ/kg Heat loss = Heat gain m gc p(t 1 - t 2) = m s(h - h f) 7(1.0)(954 – 343) = ms(2796.0 – 436.45) m s = 1.81 kg/sec A diesel electric plant supplies energy for Meralco. During a 24-hour period, the plant consumed 240 gallons of fuel at 28°C and produced 3930 KW-hr. Industrial fuel used is 28°API and was purchased at P30 per liter at 15.6°C. What is the cost of the fuel be to produce one KW-hr? *A. P6.87 B. P1.10 C. P41.07 D. P5.00 SOLUTION: SG 15.6C = 141.5/(131.5 + 28) = 0.887 Density at 15.6°C = 0.887(1kg/li) = 0.887 kg/li SG 28C = 0.887[1-.0007(1 – 15.6)] = .879 Density at 28°C = 0.879(1 kg/li) = 0.879 kg/li V28C / V15.6C = SG15.6C / SG28C 240 / V15.6C = 0.887 / 0.879 V15.6C = 237.835 gallons x 3.785 li/gal = 900.21 li Cost = [(30)(900.21)] / 3930 = P6.87/KW-hr In a gas turbine unit, air enters the combustion chamber at 550 kpa, 277°C and 43 m/s. The products of combustion leave the combustor at 511 kpa, 1004°C and 180 m/s. Liquid fuel enters with a heating value of 43,000 KJ/kg. For fuel-air ratio of 0.0229, what is the combustor efficiency of the unit in percent? 70.38% B. 79.385% *C. 75.38% D. 82.38% SOLUTION: Heat supplied by fuel = mfQh = 0.0229(43,000) = 984.7 KJ/kg air Q = heat absorbed by fuel Q/m = Cp(T2 – T1) + ½(V22 – V12) Q/m = (1.0)(1004 – 277) + ½[(180) 2 –(43) 2]/1000 =742.28 KJ/kg air Combustor Efficiency = = 75.38% The specific speed of turbine is 85 rpm and running at 450 rpm. If the head is 20 m and generator efficiency is 90%, what is the maximum power delivered by the generator. 450.51 KW B. 354.52 KW C. 650.53 KW *D. 835.57 KW SOLUTION:

NS = (N√HP)/h5/4 85 = (450√HP)/(20 x 3.281) 5/4 Hp = 1244.52 Generator Output = (1244.52 x 0.746)(0.9) = 835.57 KW In Francis turbine, the pressure gage leading to the turbine casing reads 380 Kpa. The velocity of water entering the turbine is 8 m/sec, if net head of the turbine is 45 m, find the distance from center of spiral casing to the tailrace. *A. 3.0 m B. 3.5 m C. 4.0 m D. 4.5m SOLUTION : h = V2/2g 45 = (380/9.81) + z + [82/(2 x 9.81)] z=3m A turbine has a mechanical efficiency of 93%, volumetric efficiency of 95% and total efficiency of 82%. If effective head is 40 m, find the total head. 48.72 m B. 40.72 m *C. 36.22 m D. 34.72 m SOLUTION: eT = emehev 0.8 = 0.93(eh)(.95) ηh = 0.9055 Total head = h eh = (40)(0.9055) = 36.22 m A Pelton type turbine has 25 m head friction loss of 4.5 m. The coefficient of friction head loss (from Moorse) is 0.00093 and penstock length of 80 m. What is the penstock diameter? *A. 1,355.73 mm B. 3,476.12 mm C. 6771.23 mm D. 1686.73 mm SOLUTION: h =25- 4.5 = 20.5 v = √(2gh) = [(2 x 9.81 x 20.5)1/2] = 20.55 m/sec hL = (2fLv2)/gD 4.5 = (2)(0.00093)(80)(20.055)2 / 9.81D D = 1,355,730 m = 1,355.73 mm In an 9,000 KW hydro-electric plant the over-all efficiency is 88% and the actual power received by the customer is 110,000 KW-hrs for that day. What is the secondary power could this plant deliver during the entire day? 58,960 KW-hrs *B. 80,080 KW-hrs C. 65,960 KW-hrs D. 70,960 KW-hrs SOLUTION: Plant Capacity = 9,000(0.88)(24) = 190,080 KW-hrs Secondary Power = 190,080 – 110,000 = 80,080 KW-hrs A Pelton type turbine was installed 30 m below the gate of the penstock. The head loss due to friction is 12 percent of the given elevation. The length of penstock is 100 m and

coefficient of friction is 0.00093. Determine the power output in KW. ( Use Moorse equation) 22,273 B. 23,234 C. 32,345 *D. 34,452 SOLUTION: hL = 0.12(30) = 3.6 m h = 30 – 3.6 = 26.40 m v = (2gh)1/2 = [(2)(9.81)(26.4)]1/2 = 22.759 m/sec hL= (2fLv2)/gD 3.6 = (2 x .00093 x 100 x 22.759) / (9.81D) D = 2.728 m Q = A x v = [2](22.759) = 133.03 m3/sec Power = w Q h = 9.81(133.03)(26.4) = 34,452 KW Water flows steadily with a velocity of 3.05 m/s in a horizontal pipe having a diameter of 25.4 cm. At one section of the pipe, the temperature and pressure of the water are 21C and 689.3 Kpa, respectively. At a distance of 304.8 m downstream A hydro electric plant having 30 sq. km reservoir area and 100 m head is used to generate power. The energy utilized by the consumers whose load is connected to the power plant during a five-hour period is 13.5 x 106 kwh. The overall generation efficiency is 75%. Find the fall in the height of water in the reservoir after the 5-hour period. 5.13 m B. 1.32 m C. 3.21 *D. 2.20 m SOLUTION Energy Output = Power x time = (w Q h) x time 13.5 x 106 = 9.81(Q)(100)(0.75)(5) Q = 3669.725 m3/s Volume after 5 hrs = 3669.725(5 x 3600) = 66,055,050 m 3 Volume = A x height 66,055,050 = (30 x 106) h H =2.202 m The gas density of chimney is 0.75 kg/m3 and air density of 1.15 kg/m3. Find the driving pressure if the height of chimney is 63.71 m. 0.15 kpa *B. 0.25 kpa C. 0.35 kpa D. 0.45 kpa SOLUTION: hw = H(da – dg) = 63.71(1.15 – 0.75) (0.00981) = 0.25 kpa The actual velocity of gas entering in a chimney is 8 m/sec. The gas temperature is 25C with a gas constant of 0.287 KJ/kg-K. Determine the gas pressure for a mass of gas is 50,000 kg/hr and chimney diameter of 1.39m. 95 kpa *B. 98 kpa C. 101 kpa D. 92 kpa SOLUTION: Vg = A x v = / 4 (1.39)2(8) = 12.139 m3/s PgVg = mgRgTg P(12.139) = (50,000/3600)(.287)(25 +273) P = 97.85 kpa

A steam generator with economizer and air heater has an overall draft loss of 25.78 cm of water. If the stack gases are at 177C and if the atmosphere is at 101.3 Kpa and 26C, what theoretical height of stack in meters is needed when no draft fan are used? Assume that the gas constant for the flue gases is the same as that for air. A 611.10 B. 631.10 *C.651.10 D.671.10 SOLUTION: w = P/RT da = (101.325)/[(.287)(26 + 273)] = 1.180 kg/m3 dg = (101.3)/[(0.287)(177 +273)] = 0.784 kg/m3 Draft = (0.2578)(1000) = 257.80 kg/m3 Draft = H(da – dw) 257.80 = H(1.18 – 0.784) H = 651.10 m A foundation measures 12 ft x 14 ft x16 ft. Find the number of sacks of cement needed for 1:2:4 mixture. 302 B. 404 C. 356 *D. 598 SOLUTION: V = 12 X 14 X 16 = 2,688 ft3 (1 yd3 / 33 ft3) = 99.55 yd3 of concrete For every 1 yd3 concrete, it needs 6 sacks of cement Therefore: No. of sacks = 6(99.55) = 597.33 sacks or 598 sacks A rectangular foundation cross-section has a bed plate dimension of 8 ft x 10 ft. The uniform clearance on each side is 1 ft. The height of foundation is 4.5 ft. If the weight of the steel bar reinforcements needed is 1/2% of weight of foundation, find the weight of steel bars. Use concrete density of 2400 kg/m3 . 173.47 kg *B. 183.47 kg C. 163.47 kg D. 153.47 kg SOLUTION: A = (8 + 2) (10 + 2) = 120 m2 V = Ah = 120(4.5) = 540 ft3 = 15.29 m3 W = wV = (2400)(15.29) = 36,693.25 kg Weight of steel bars = (1/2%) W f = 0.005(36,693.25) = 183.47 kg A steam pipe having a surface temperature of 250C passes through a room where the temperature is 27 C. The outside diameter of pipe is 100 mm and emissivity factor is 0.8. Calculate the radial heat loss for 3 m pipe length. 1434.7 W B. 37.46 W *C. 2651.82 W D. 3545.45 W SOLUTION: A = DL = = 0.425m2 Solving for heat due to radiation: Tg = 250 +273 = 523K T2 = 27 +273 = 300K Qa = 20,408.4 x 104 AF(T14 – T24), J/hr = 20,408.4 x 104(0.8)(0.7539)[(523)4 – (300)4] Qr = 10,266,539.06 j/hr x 1hr/3600sec = 2851.82 W

Brine enters a circulating brine cooler at the rate of 60 m 3/hr at -*C and leaves at -18C. Specific heat of brine is 1.072 KJ/kg-K and specific gravity of 1.12. Determine the tons of refrigeration. 53.5 TR B. 65.3 TR C.33.5 TR *D. 56.9 TR SOLUTION: Density of brine = 1.12(1000 kg/m3) = 1120 kg/m3 m = (1120)(60)/3600 = 18.67 kg/sec Q = mcp = 18.67(1.072)(-8 + 18) = 200.11 KW TR = 200.11/3.516 = 56.91 Tons of refrigeration A turbo-charged, 16 cylinder, Vee-type diesel engine has an air consumption of 3,000 kg/hr per cylinder at rate load and speed. This air is drawn in through a filter by a centrifugal compressor directly connected to the exhaust gas turine. The temperature of the air from the compressor is 135C and a counter flow air cooler reduces the air temperature to 45C before it goes to the engine suction heater. Cooling water enters air cooler at 30C and leaves at 40C. Calculate the log mean temperature difference. 47.23C B. 87.82C *C. 43.34C D. 65.24C SOLUTION: a = 45-30 = 15C b = 135 – 40 = 95C mean = [a - b] / [ln(ab)] = [95-15] / ln(95/15) = 43.34C Water is flowing in a pipe with radius of 30 cm at a velocity of 5 m/s at the temperature in pipe. The density and viscosity of the water are as follows: density 997.9 kg/sec viscosity = 1.131 Pa-s. What is the Reynolds Number for this situation? *A. 2647 B. 96.2 C. 3100 D. 1140 SOLUTION: n = Dvg / v Where: D = 2(0.30) = 0.60 m vg = 5 M/SEC v = 1.131/997.9 = 0.0011334 m2 / sec Nm = 0.60(5)/0.0011334 = 2,647 Compute the amount of condensate form during 10 minutes warm-up of 180 meter pipe conveys the saturated steam with enthalpy vaporization hfg = 1,947.8 KJ/kg. The minimum external temperature of pipe is 2C. The final temperature of pipe is 195C. The specific heat of pipe material is 0.6 KJ/kg-C. The specific weight is 28 kg/m. 249.69 kg B. 982.45 kg *C. 299.64 kg D. 423.45 kg SOLUTION: mp = mass of pipe = 28(180) = 5,040 kg Heat Loss by steam = Heat loss from pipe m(hg - hf) = mpcp (t2 – t1) m(1947.8) = (5040)(0.6)(195-2) m = 299.64 kg

The discharge pressure of an air compressor is 5 times the suction pressure. If volume flow at suction is 0.1 m3/sec, what is the suction pressure if compressor work is 19.57 kw? (use n=1.35) 97 kpa *B.98 kpa C. 99 kpa D.100 kpa SOLUTION: W = [(P2/P1)n-1/n – 1] 19.57 = 1.35(P1)(0.1)/(1.35-1)[(5)1.35-1/1.35 – 1] P1 = 98 KPa The initial condition of air in an air compressor is 98 KPa and 27C and discharge air at 450 KPa. The bpre and stroke are 355 mm and 381 mm, respectively with percent cleared of 8% running at 300 rpm. Find the volume of air at suction. 541.62 m3/hr B. 551.62 m3/hr C. 561.62 m3/hr *D. 571.62 m3/hr SOLUTION: ev = 1 + c – c(P2/P1)1/n = 1 + 0.08 - 0.08(450/98)1/1.4 = 0.842 VD = D2 LN = (0.355)2 (0.381)(300/60) = 0.1885 m3/sec V1 = 0.1885(0.842) = 0.15878 m3/sec = 571.62 m3/hr An air compressor has a suction volume of 0.35 m3/sec t 97 KPa and discharges to 650 KPa. How much power saved by the compressor of there are two stages? 18.27 KW B. 16.54 KW C. 13.86 KW *D. 11.58 KW SOLUTION: W = [(P2/P1)n-1/n – 1] = (1.4 x 97 x 0.35)/(1.4 -1) [(650/97)1.4-1/1.4 – 1] = 85.79 KW For two stages : Px = (P1P2)1/2 = (97 x 650)1/2 = 251.097 KPa W = [(Px/P1)n-1/n – 1] = 2(1.4)(97)(0.35)/(1.4 – 1) [(251.0.97/97)1.4-1/1.4 – 1] = 74.208 KW POWER SAVED = 85.79 – 74.208 = 11.582 KW A twop stage air compressor has an intercooler pressure of 4 kg/cm2. What is the discharge pressure if suction pressure is 1 kg/cm2? 3 kg/cm2 B. 9 kg/cm2 C. 12 kg/cm2 *D. 16 kg/cm2 SOLUTION: Px = (P1P2)1/2 Px2 = P1(P2) 42 = 16 kg/cm2 A two stage air compressor compresses air at 100 KPa and 22C discharges to 750 KPa. If intercooler intake is 105C. Determine the value of n. 1.400 *B. 1.325 C. 1.345 D. 1.288 SOLUTION: Px = (100 x 750)1/2 =273.86 KPa Tx/T1 = (Px/P1)n-1/n (105 + 273)/(22 + 273) = (273.86/100)n-1/n 1.281 = (2.6268)n-1/n n = 1.326

A single acting compressor has a volumetric efficiency of 89%, operates at 500 rpm. It takes in air at 100 KPa and 30C and discharges it at 600 KPa. The air handled is 8 m3/min measured at discharge condition. If compression is isentropic, find mean effective pressure in KPa. *A. 233.34 B. 973.17 C. 198.34 D. 204.82 SOLUTION: P1V1K = P2V2K 100(V11.4) = 600(6)1.4 V1 = 28.768 m3/min VD = 28.768/0.89 = 32.32 m3/min W = n P1V1/n-1 x [(P2 / P1)n-1/n – 1] = [(1.4 x 100 x 32.32)/(1.4 – 1)] x [(600/100)1.4-1/1.4 – 1] W = 7562.19 KJ/min W = Pm x Vd 7562.19 = Pm x 32.32 Pm = 233.34 KPa A water-jacketed air compressed handles 0.343 m3/s of air entering at 96.5 KPa and 21C and leaving at 460 KPa and 132C; 10.9 kg/h of cooling water enters the jacket at 15C and leaves at 21C. Determine the compressor brake power. 26.163 KW *B. 62.650 KW C. 34.44 KW D. 19.33 KW SOLUTION: T2/T1 = (P2/P1) n-1/n (132+273) / (21+273) = (480/96.5)n-1/n n = 1.249 W = (1.249 x 96.5 x 0.343) / (1.249-1) [(480 / 96.5)1.249-1/1.249 – 1] W = 62.57 KW Q = heat loss = mcp(t2 – t1) = (10.9/3600)(4.187)(21 – 15) 0.075 KW Brake power = W + Q = 62.57 + 0.076 = 62.65 KW A double suction centrifugal pumps delivers 20 ft3/sec of water at a head of 12 m and running at 650 rpm. What is the specific speed of the pump? 5014.12 rpm B. 6453.12 rpm *C. 2770.73 rpm D. 9966.73 rpm SOLUTION: N = N(Q)1/2 / h3/4 Q = 20/2 ft3/sec x 7.481 gal/ft3 x 60 sec/1min = 4,488.6 gal/min h = 12 x 3.281 = 39.37 ft N = (650 x (4,488.6)1/2)/(39.37)3/4 N = 2,770.73 rpm Determine the number of stages needed for a centrifugal pump if it is used to deliver 400 gal/min of water and pump power of 15 Hp. Each impeller develops a head of 30 ft. 6 B. 4 *C. 5 D. 7 SOLUTION: Wp = w Q h 15 x 0.746 = 9.81(400 gal/min x 0.00785m3/gal x 1/60)h h = 45.20 m x 3.281 ft/m = 148.317 ft Number of stages = 148.317/40 = 4.94 stages = 5 stages

The suction pressure of a pump reads 3 in. of mercury vacuum and discharge pressure reads 140 psi is use to deliver 120 gpm of water with specific volume of 0.0163 ft 3/lb. Determine the pump work. 4.6 KW B. 5.7 KW *C. 7.4 KW D. 8.4 KW SOLUTION: P1 = -3 in Hg x 101.325/29.92 = -10.16 KPa P2 = 140 psi x 101.325/14.7 = 965 KPa w = 1/v = 1/0.163 = 61.35 lb/ft3 x 9.81/62.3 = 9.645 KN/m3 h = (P2 – P1)/w = (965 +10.16)/9.645 = 101.105 m Q = 120 gal/min x 3.785/1gal x 1m3/1000li x 1/60 = 0.00757 m3/sec P = w Q h = 9.645(0.00757)(101.105) = 7.38 KW A submersible pump delivers 350 gpm of water to a height of 5 ft from the ground. The pump were installed 150 ft below the ground level and draw down of 8 ft during the operation. If water level is 25 ft above the pump, determine the pump power. 7.13 KW B. 4.86 KW C. 7.24 KW *D. 9.27 KW SOLUTION: h = 5 + 150 – (25 – 8) = 138/3.281 = 42.06 m Q = 350 gal/min x 0.003785 m3/gal x 1 min/60sec = 0.02246 m3/sec Wp = w Q h = 9.81(0.02246)(42.06) = 9.27 KW A vacuum pump is used to drain a flooded mine shaft of 20 water. The pump pressure of water at this temperature is 2.34 KPa. The pump is incapable of lifting the water higher than 16 m. What is the atmospheric pressure? *A. 159.30 B. 32.33 C. 196.22 D. 171.9 SOLUTION: Using Bernoulli’s Theorem: P1/w + V12/2g + z1 = P2/w + V2/2g + z2 P1/w = P2/w + (V22 - V12)/2g + (z2 - z1) P1/9.81 = 2.34/9.81 + 0 + 16 P1 = 159.30 KPa A submersible, multi-stage, centrifugal deep well pump 260 gpm capacity is installed in a well 27 feet below the static water level and running at 3000 rpm. Drawdown when pumping at rated capacity is 10 feet. The pump delivers the water into a 25,000 gallons capacity overhead storage tank. Total discharge head developed by pump, including friction in piping is 243 feet. Calculate the diameter of the impeller of this pump in inches if each impeller diameter developed a head of 38 ft. 3.28 B. 5.33 *C. 3.71 D. 6.34 SOLUTION: V= DN V= D (3000/60) = (2(32.2)(38))1/2 D = 0.315 ft = 3.708 inches A fan pressure of 2.54 cm of water t 1.42 m3 per second of air at static pressure of 2.54 cm of water through a duct 300 mm diameter and discharges it through a duct 275 mm diameter. Determine the static fan efficiency if total fan mechanical is 75% and air measured at 25 and 60 mm Hg.

50.11% *B. 53.69% C. 65.67% D. 45.34% SOLUTION: wA = P/RT = 101.325/(0.287)(25 + 273) = 1.18 kg/m3 hA = hwww/wA = (0.0254)(1000)/1.18 = 21.52 m vA = 1.42/(/4)(0.3)2 = 20.09 m/s Vd = 1.42/(/4)(0.275)2 = 23.9 m/s hv = (23.9)2 – (20.09)2 / 2(9.81) = 8.54 m h = ha + hv = 21.52 + 8.54 = 30.06 m eT = wa Q h/BP 0.75 = (1.18 x 0.00981)(1.42)(30.06) / BP BP = 0.6588 KW ep = wa Q hs/BP = (1.18 x 0.00981)(1.42)(21.52) / 0.6588 = 53.69% A water cooler uses 50 lb/hr of melting ice to cool running water from 80 to 42. Based on te inside coil area, U1 = 110 Btu/hr-ft2-. Find the gpm of water cooled. 0.10 GPM B. 0.21 GPM *C. 0.38 GPM D. 0.45 GPM SOLUTION: Q = mf L = mwcpw(t1 – t2) 50 (144) = mW(1)(80-42) mw = 189.474 lb/hr V = (189.474/62.4) (7.48/60) = 0.38 GPM The charge in a Diesel engine consists of 18.34 grams of fuel, with lower heating value of 42,571 KJ/kg, and 409 grams of fuel and products of combustion. At the beginning of compression, t1 = 60. Let rk = 14. For constant cP = 1.11 KJ/kg-C, what should be the cut-off ratio in the corresponding ideal cycle? 2.05 B. 2.34 C. 5.34 *D. 2.97 SOLUTION: QA = mfQh = 0.018.34(42,571) = 780,752 KJ T2/T1 = rkk-1 T2 = (60 + 273)1.4-1 = 956.964K mt + mg = 409 mt + ma + mf = 409 ma = 409 – 2(18.34) = 372.32 grams QA = macp(t3 – t2) 780.752 = 0.37232(1.11)(T3 – 956.964) T3 = 2846,146 rC = T3/T2 = 2846.146/956.964 = 2.97

The gain of entropy during isothermal nonflow process of 5 lb of air at 60 is 0.462 Btu/R. Find the V1/V2. 3.85 *B. 0.259 C. 1.0 D. 0.296 SOLUTION: s = m R T ln(V2/V1) 0.462 = 5 (53.33/778) ln (V2/V1) V2/V1 = 3.85

V1/V2 = 1/3.85 = 0.259 An auditorium seating 1500 people is to be maintained at 80 dry bulb and 85 wet bulb temperature when outdoor air is at 91 dry bulb and 75 wet bulb. Solar heat load is 110,000 Btu/hr and supply air at 60 determine the amount of supply air. *A. 93,229.17 lb/hr B. 83,229.17 lb/hr C. 73,229.17 D. 63,229.17 lb/hr SOLUTION: Sensible heat per person = 225 Btu/hr Qa = 225(1500) + 110,000 = 447,500 Btu/hr Qa = m cp(t1 – t2) 447,500 = ma(0.24)(80 – 60) ma = 93,229.17 lb/hr In a Brayton cycle that operates between temperature limits of 300K and 1773K wit k = 1.4, determine the temperature at the end of the compression (isentropic) for maximum work of the cycle. 700K B. 590.5K *C. 730K D. 350K SOLUTION: For maximum work: T2 = (T1T3)1/2 T2 = (300 x 1773)1/2 = 730K At 35% solution leaves the absorber and 30% solution enters the absorber. The heat removed from the absorber by cooling water is 547.6 Btu and ammonia is superheated by 10. Find the pound per pound of ammonia gas from the evaporating coils. 11 B. 12 *C. 13 D. 14 SOLUTION: n = lb/lb of ammonia gas from the coils n = (1 - x2) / (x1 - x2) – 1 n = (1- 0.3) / (0.35 – 0.3) – 1 = 13 A Carnot refrigeration system operates at Tmax / Tmin = 1.5. Find the KW per tons of refrigeration. 1.91 B. 2.15 *C. 1.76 D. 1.55 SOLUTION: Wo / TR = 3.516 / COP = 3.516 / (Tmin / (Tmax – Tmin) Wo / TR = [3.516(Tmax – Tmin)] / Tmin Wo / TR = 3.516[(Tmax / Tmin) – 1] = 3.516(1.5-1) = 1.758 KW/TR Assume 8 ft3 of air at 100 psi, 100 are compressed isothermally to a volume of 2 ft 3. For each of end states of the process, find the bulk modulus. *A. 400 and 100 psi B. 400 and 110 psi C. 400 and 120 psi D. 400 and 130 An empty open can is 30 cm high with a 15 cm diameter. The can, with the open end and down, is pushed under water with a density of 1000kg/m 3. Find the water level in the can when the top of the can is 50 cm below the surface. 17.20 cm *B. 2.12 cm C. 4.20 cm D. 5.87cm SOLUTION: Consider the water pressure

Pw = w h + 1010.325 + (0.8-x)(9.81) + 101.325 = 109.173 – 9.81x Consider the air pressure P1V1 = P2V2 101.325(Ax0.3) = P2[A(0.3-x)] P2 = Pw = P 2 109.173 – 9.81x = 9.81x2 – 112.116x + 2.3705 = 0 By quadratic formula: X = 0.02118 m = 2.12 cm A cylindrical pipe with water flowing downward at 0.03 m3/s having top diameter of 0.08, bottom diameter of 0.04 m and a height of 1.5m. Find the pressure between the pipe 154.63 Kpa B. 197.93 Kpa *C. 252.44 Kpa D. 243.92 Kpa SOLUTION: + Z1 = + Z 2

Z1 – Z2 = 1.5 m Z2 – Z1 = -1.5 m V1 = = 5.968 m/s V2 = = 23.87 m/s P1 – P2 = 252.44 Kpa Determine the size of pipe which will deliver 8 liters of medium oil (v= 6.10 x 10-6 m2/s) assuming laminar flow conditions: 622 mm B. 754 mm C. 950 mm *D. 835 mm SOLUTION: V= Re = For laminar flow, Re = 2000 2000 = d = 0.835 m = 835 mm The type of flow occupying in a 1 cm diameter pipe which water flows at a velocity of 2.50 m/s. Use v = 1.13 x 10-6 m2/s for water.

*A. turbulent B. constant C. laminar

D. none of the above

SOLUTION: Re = Re = Re = 22,124 Since it is greater than 2000 then it is turbulent flow What is the force is exerted by water jet 60 mm diameter if it strikes a wall at the rate of 15 m/s? *A. 636.17 N B.442.62 N C. 764.23 N D. 563.34 N SOLUTION: F=wQv Q = A v = = 0.0424 m3/s F = (1000)(0.0424)(15) = 636.17 N A 300 mm diameter pipe discharges water at the rate of 200 li/s. Point 1 on the pipe has a pressure of 260 kpa and 3.4 m below point 1 is point 2 with a pressure of 300 kpa. Compute the head loss between points 1 and 2. 4.29 m B. 2.59 m C. 6.32 m *D. 1.87 m SOLUTION: hL hL = Water flowing at the rate of 10 m/s from an orifice at the bottom of a reservoir. Find the pressure at the bottom of the reservoir. 30 kpag B. 40 kpag *C. 50 kpag D. 60 kpag SOLUTION: h = V2/ 2g = 102/ 2(9.81) = 5.0968 m P = w h = 9.81(5.0968) = 50 kpag Steam flows through a nozzle at 400oC and 1 Mpa (h = 3263.9 KJ/kg) with velocity of 300 m/s. Find the stagnation enthalpy. 3300 KJ/kg B. 3290 KJ/kg *C. 3320 KJ/kg *D. 3309 KJ/kg SOLUTION: ho = h + v2/2000 = 3263.9 + 3002/2000 = 3309 KJ/kg Air flows through a nozzle at a speed of 350 m/s. Find the stagnation temperature if entrance temperature is 200oC. 241.25oC B. 251.25oC *C. 261.25oC D. 271.25oC SOLUTION: To = T1 + v2/2000Cp = (2000 + 273) + 3502/2000(1) To = 534.25oK = 261.25

Carbon dioxide flows through a nozzle with a speed of 400 m/s. Compute the dynamic temperature. 92.56oK *B. 94.56oK C. 96.56oK D. 98.56oK SOLUTION: For CO2: Cp = 0.846 KJ/kg-K Dynamic temperature = v2/2000Cp = 4002/2000(0.846) = 94.56oK Carbon dioxide flows through a nozzle with a speed of 380 m/s. The entrance condition of nozzle is 250oC and 1200 kpa. Find the stagnation pressure. *A. 2,136.34 kpa B. 2,146.34 kpa C. 2,156.34 kpa D. 2,166.34 kpa SOLUTION: T1 = 250 + 273 = 523oK To = T1 + v2/2000 = 523 = 3802/2000 = 595.2oK P1 = 1200 kpa T1/To = (P1/Po)k-1/k For CO2: k = 1.289 523/595.2 = (1200/Po)1.289-1/1.289 P0 = 2,136.34 kpa Air enters a diffuser with a velocity of 200 m/s. Determine the velocity of sound if air temperature is 30oC. *A. 349 m/s B. 359 m/s C. 369 m/s D. 379 m/s SOLUTION: C= Air flows through a nozzle with temperature of entrance of 420 oK stagnation temperature of 468oK. Find the mach number. 0.744 *B. 0.754 C. 0.764 D. 0.774 SOLUTION: To = T1 + v2/2000Cp 468 = 420 + v2/2000 v = 309.838 m/s C= M = v/C = 309.838/410.8 = 0.754 Air at 300oK and 200 kpa is heated at constant pressure to 600oK. Determine the change of internal energy. 245.58 KJ/kg B. 235.58 KJ/kg C. 225.58 KJ/kg *D. 215.58 KJ/kg SOLUTION: ΔU = mCv (T2 – T1) = 1(0.7186)(600 -300) = 215.58 KJ/kg An insulated rigid tank initially contains 1.5 lb of helium at 80 oF and 50 psia. A paddle wheel with power rating of 0.02 hp is operated within the tank for 30 min. Determine the final temperature. 159.22oF B. 169.22oF *C. 179.22oF D. 189.22 oF

SOLUTION: W = ΔU = m Cv (T2 – T1) 0.02 hp (0.50hr)(2545Btu/hr/hp) = 1.5(0.171)(t2 – 80) t2 = 179.22oF A 4m2 asphalt pavement with emissivity of 0.85 has a surface temperature of 50 oC. Find the maximum rate of radiation that can be emitted from the surface. A. 2,068.32 watts B. 2,078.32 watts C. 2,088.32 watts *D. 2.098.32 watts SOLUTION: Qr = e kev A Ts4 Kev = 5.67 x 10-8 ( Stefan Boltzman constant) Qr = 0.85(5.67 z 10-8)(4)*50 +273)4 = 2,098.32 watts Air at 10oC and 90 kpa enters a diffuser of a jet engine steadily with a velocity of 200 m/s. The inlet area diffuser is 0.40 m2. Determine the mass flow rate of air. A. 72.79 kg/s B. 74.79 kg/s C. 76.79 kg/s *D. 78.79 kg/s SOLUTION: W = P/RT = 80/0.287(10 + 273) = 0.985 kg/m3 m = w v A = 0.985(200)(0.40) = 78.79 kg/s Consider a refrigeration whose 40 watts light bulb remains on continuously as a result of a malfunction of the switch. If the refrigerator has a COP of 1.3 and the cost of electricity is 8 cents per kw-hr., determine the increase in the energy consumption of the refrigerator and its cost per year if the switch is not fixed. *A. P49.59 B. P47.59 C. P45.59 D. P43.59 SOLUTION: COP = RE/W ref 1.3 = 40/W ref Wref = 30.769 watts W = W b + W ref = 40 + 30.769 = 70.77 watts W = 0.07077 Kw Cost = 0.07077(8760)(P0.08) = P49.59 A 75 hp motor that has an efficiency of 91% is worn out and is replaced by a highefficiency motor that has an efficiency of 95.4%. Determine the reduction in heat gain of the room due to higher efficiency under full-load conditions. A. 2.24 KW *B. 2.44 KW C. 2.64 KW D. 2.84 KW SOLUTION: P01 = (75 x 0.746)(0.91) = 50.91 KW P02 = (75 x 0.746)(0.954) = 53.376 KW Qreduced = 53.376 – 50.91 = 2.44 KW A household refrigerator that has a power input of 450 watts and a COP of 2.5 is to cool five large watermelons, 10 kg each, to 8oC. If the watermelons are initially at 20oC, determine how long will take for the refrigerator cool them. The watermelons can be treated as a water whose specific heat is 4.2 KJ/kg-oK.

A. 2220 seconds B. 2230 seconds SOLUTION: COP = RE/Wc 2.5 = RE/450 RE = 1,125 watts RE = m cp (t2 – t1) 1125 t = (10 x 5)(4.2)(20-8) t = 2240 seconds

*C.2240 seconds

D. 2250 seconds

When a man returns to his wall-sealed house on a summer day, he finds that the house is at 32oC. He returns on the air conditioner which cools the entire house to 20 oC in 15 minutes, if COP is 2.5, determine the power drawn by the airconditioner. Assume the entire mass within the house is 800 kg of air for which cv = 0.72 KJ/kg-K, cp = 1.0KJ/kgK. 1.072 KW B. 2.072 KW *C. 3.072 KW D. 4.072 KW SOLUTION: RE = m cv (T2 –T1) = (800/15x60)(0.72)(32-20) RE = 7.66 KW Wc = 7.68/2.5 = 3.072 KW A heat source at 8000K losses 2000 KJ of heat to a sink at 500oK. Determine the entropy generated during this process. *A. 1.5 KJ/K B. 2.5 KJ/K C. -2.5 KJ/K D. 4 KJ/K SOLUTION: ΔSsource = -2000/800 = -2.5 ΔSsink = 2000/500 = 4 ΔSgen. = -2.5 + 4 = 1.5 KJ/K Helium gas is compressed in an adiabatic compressor from an initial state of 14 psia and 50oF to a final temperature of 320oF in a reversible manner. Determine the exit pressure of Helium. 38.5 psia *B. 40.5 psia C. 42.5 psia D. 44.5 psia SOLUTION: T2/T1 = (P2/P1)n-1/n (320 + 460)/(50 +460) = (P2/14)1.587-1/1.587 P2 = 40.5 psia Air pass thru a nozzle with efficiency of 90%. The velocity of air at the exit is 600 m/s. Find the actual velocity at the exit. A. 382 m/s B. 540 m/s C. 458 m/s *D. 568 m/s SOLUTION: e = (v2/v3)2 0.9 = (v2/600)2 v2 = 568.21 m/s

A 50 kg block of iron casting at 500K is thrown into a large lake that is at a temperature of 285oK. The iron block eventually reaches thermal equilibrium with the lake water. Assuming average specific hear of 0.45 KJ/kg-K for the iron, determine the entropy generated during this process. *A. -12.65 KJ/k B. 16.97KJ/K C. 4.32 KJ/K D. 6.32 KJ/K SOLUTION: ΔSiron = m c ln (T2/T1) = 50(0.45)ln(285/500) = -12.65 KJ/K ΔSlake = Q/T = [50(0.45)(500-285)]/285 = 16.97 KJ/K ΔSgen. = -12.65 + 16.97 = 4.32 KJ/K A windmill with a 12 m diameter rotor is to be installed at a location where the wind is blowing at an average velocity of 10 /s. Using standard conditions of air (1 atm, 25oC), determine the maximum that can be generated by the windmill. A. 68 KW *B. 70 KW C. 72 KW D. 74 KW SOLUTION: w = P/RT = 101.325/(0.28)(25+ 273) = 1.1847 kg/m3 m = w A v = 1.1847(π/4 x 122)(10) = 1,1339.895 kg/s KE = v2/2000 = 102/2000 = 0.05 KJ/kg Power = mKE = 1,1339.895(0.05) = 70 KW Consider a large furnace that can supply heat at a temperature of 2000 oR at a steady rate of 3000Btu/s. Determine the energy of this energy. Assume an environment temperature of 77oF. A. 2305.19 KW *B. 2315.19 KW C. 2325.19 KW D. 2335.19 KW SOLUTION: e = = 0.7315 W = e Q = 0.7315(3000) = 2194.5 Btu/s = 2315.19 KW A heat engine receives hat from a source at 1200oK at a rate of 500KJ/s and rejects the waste heat to a medium at 300oK. The power output of the heat engine is 180 KW. Determine the irreversible rate for this process. A. 190 KW *B. 195 KW C. 200 KW D. 205 KW SOLUTION: e = (1200 – 300) / 1200 = 0.75 W = 0.75(500) = 375 KW Irreversibilities = 375 – 180 = 195 KW A dealer advertises that he has just received a shipment of electric resistance heaters for residential buildings that have an efficiency of 100 percent. Assuming an indoor temperature of 21oC and outdoor temperature of 10oC, determine the second law efficiency of these heaters. A. 8.74% B. 6.74% *C. 3.74% D. 4.74% SOLUTION: COP1 = 100% efficient = 1

COP2 = (21 + 273) / (21 – 10) = 26.72 e = COP1 /COP2 = 1/ 26.72 = 3.74 % A thermal power plant has a heat rate of 11,363 Btu/KW-hr. Find the thermal efficiency of the plant. 34% B. 24% C. 26% *D. 30% SOLUTION: e = 3412 / Heat rate = 3412 / 11363 = 30 % A rigid tank contains 2 kmol of N2 and 6 kmol of CO2 gasses at 300oK and 15 Mpa. Find the tank volume us ideal gas equation. 7.33 m3 B. 5.33 m3 C. 3.33 m3 *D. 1.33 m3 SOLUTION: PmVm = Nm R Tm 15,000 Vm = (6 + 2)(8.314)(300) Vm = 1.33 m3 A spherical balloon with a diameter of 6 m is filled with helium at 20 oC and 200 kpa. Determine the mole number. *A. 9.28 Kmol B. 10.28 Kmol C. 11.28 Kmol D. 13.28 Kmol SOLUTION: PV=NRT (200)[(4/3)(π)(6/2)3] = N (8.314)(20 + 273) N = 9.28 Kmol The air in an automobile tire with a volume of 0.53 ft3 is at 90oF and 20 psig. Determine the amount of air that must be added to raise the pressure to the recommended value of 30 psig. Assume the atmospheric to be 14.7 psia and the temperature and the volume to remain constant. *A. 0.026 lb B. 0.046 lb C. 0.066 lb D. 0.086 lb SOLUTION: PV=mRT (20 + 14.7)(144)(0.53) = m1 (53.3)(90 + 460) m1 = 0.09034 lb (30 + 14.7)(144)(0.53) = m2(53.3)(90 + 460) m2 = 0.11634 lb madded = m2 – m1 = 0.11634 – 0.09034 = 0.026 lb A rigid tank contains 20 lbm of air at 20 psia and 70oF. More air is added to the tank until the pressure and temperature rise to 35 psia and 90 oF, respectively. Determine the amount of air added to the tank. 11.73 lb *B. 13.73 lb C. 15.73 lb D. 17.73 lb SOLUTION: P1V1 = m1 R1T1 (20 x 144)(V1) = 20 (53.3)(70 + 460)

V = 196.17 ft3 P2V2 = m2R2T2 (35 x 144)(196.17) = m2 (53.3)(90 + 460) m2 = 33.73 lbs madded = m2 –m1 = 33.73 – 20 = 13.73 lb A rigid tank contains 5 kg of an ideal gas at 4 atm and 40oC. Now a valve is opened, and half of mass of the gas is allowed to escape. If the final pressure in the tank is 1,5 atm, the final temperature in the tank is: *A. -38oC B. -30oC C. 40oC D. 53oC SOLUTION: PV = m R T (4 x 9.81)(V) = 5(0.287)(40 + 273) V = 11.446 m3 PV = mRT (1.5 x 9.81)(11.446) = (5/2)(0.287)(T) T = 234.74oK = -38.26oC The pressure of an automobile tire is measured to be 200 kpa(gage) before the trip and 220 kpa(gage) after the trip at a location where the atmospheric pressure is 90 kpa. If the temperature of the air in the tire before the trip is 25oC, the air temperature after the trip is: *A. 45.6oC B. 54.6oC C. 27.5oC D. 26.7oC SOLUTION: T 2 / T 1 = P 2 / P1 T2 / (25+ 273) = (220 +90) / (200 + 90) T2 = 318.55 K t2 = 45.55oC Water is boiling at 1 atm pressure in a stainless steel pan on an electric range. It is observed that 2 kg of liquid water evaporates in 30 mins. The rate of heat transfer to the water is: 2.97 KW B. 0.47 KW *C. 2.51 KW D. 3.12 KW SOLUTION: Q = mL = (2257) = 2.51 KW Consider a person standing in a breezy room at 20oC. Determine the total rate of heat transfer from this person if the exposed surface area and the average outer surface temperature of the person are 1.6 m2 and 29oC, respectively, and the convection heat transfer coefficient is 6 W/K-m2 with emissivity factor of 0.95. 86.40 watts B. 61.70 watts C. 198.1 watts *D. 168.1 watts SOLUTION: Qc = h A (t2 – t1) = (6)(1.6)(29.20) = 86.40 watts Qr = (0.95)(5.67 x 10-6)[(1.6)(29 + 273)4 – (20 + 273)4] = 81.7 watts Q = Qc + Qr = 86.40 + 81.7 = 168.1 watts

Water is boiler in a pan on a stove at sea level. During 10 minutes of boiling, it is observed that 200 grams of water has evaporated. Then the rate of heat transfer to the water is: 0.84 KJ/min *B. 45.1 KJ/min C. 41.8 KJ/min D. 53.5 KJ/min SOLUTION: Q = m L = (0.2 / 10) (2257) = 45.1 KJ/min An aluminum pan whose thermal conductivity is 237 W/m-C has a flat bottom whose diameter is 20 cm and thickness 0.4 cm. Heat is transferred steadily to boiling water in the pan through its bottom at a rate of 500 watts. If the inner surface of the bottom of the pan is 105oC, determine the temperature of the surface of the bottom of the pan. 95.27 oC *B. 105.27oC C. 115.27oC D. 125.27oC SOLUTION: A = π / 4 ( 0.20)2 = 0.0314 m2 Q= 500 = T2 = 105.27oC For heat transfer purposes, a standing man can be modeled as a 30 cm diameter, 170 cm long vertical cylinder with both the top and bottom surfaces insulated and with the side surface at an average temperature of 34oC. For a convection heat transfer coefficient of 15 W/m2- oC, determine the rate of heat loss from this man by convection in an environment at 20oC. 316.46 watts B. 326.46 watts *C. 336.46 watts D. 346.46 watts SOLUTION: Qc = k A (t2 – t1) = 15 (π x 0.30 x 1.7) (34 – 20) = 336.46 watts A 5cm diameter spherical ball whose surface is maintained at a temperature of 70 oC is suspended in the middle of a room at 20oC. If the convection heat transfer coefficient is 15 W/m2 – C and the emissivity of the surface is 0.8, determine the total heat transfer from the ball. 23.56 watts *B. 32.77 watts C. 9.22 watts D. 43.45 watts SOLUTION: A = 4 π r2 = 4 π (0.05)2 = 0.0314 m2 Qc = h A (t2 – t1) = 15 (0.0314)(70 – 20) = 23.56 watts Qr = (0.80)(5.67 x 10-6)(0.0314)[(70 + 273)4 – (50 + 273)4] = 9.22 watts Q = Qr + Qc = 23.56 + 9.22 = 32.77 watts A frictionless piston-cylinder device and rigid tank contain 1.2 kmol of ideal gas at the same temperature, pressure, and volume. Now heat is transferred, and the temperature of both system is raised by 15oC. The amount of extra heat that must be supplied to the gas in the cylinder that is maintained at constant pressure. SOLUTION:

0

B. 50 KJ C. 100 KJ *D. 150 KJ Q = m cp (t2 – t1) = (1.2 x 8.314)(1)(15) = 150 KJ

A supply of 50 kg of chicken needs at 6oC contained in a box is to be frozen to -18oC in a freezer. Determine the amount of heat that needs to be removed. The latent heat of chicken is 247 KJ/kg, and its specific heat is 3.32 KJ/kg-oC above freezing and 1.77 KJ/kg-oC below freezing. The container box is 1.5 kg, and the specific heat of the box material is 1.4 Kj/kg-oC. Also the freezing temperature of chicken is -2.8oC. *A. 15,206.4 KJ B. 50.4 KJ C. 15,156 KJ D. 1,863 KJ SOLUTION: Qchicken = 50 [3.32(6 + 2.8) = 247 1.77(-2.8 + 18)] = 15,156 KJ Qbox = 1.5(1.4)(6 + 8) = 50.4 KJ Q = 15,156 + 50.4 = 15, 206.4 KJ Water is being heated in a closed pan on top of a range while being stirred by a paddle wheel. During the process, 30 KJ of heat is transferred to the water, and 5 KJ of heat is lost to the surrounding air. The paddle wheel work amounts to 500 N-m. Determine the final energy of the system if its initial energy is 10 KJ. *A. 35.5 KJ B. 45.5 KJ C. 25.5 KJ D. 14.5 KJ SOLUTION: Final energy = Qa + ΔU – Qloss + W = 30 + 10 – 5 + 0.50 = 35.5 KJ A classroom that normally contains 40 people is to be air- conditioned with window airconditioning units of 5 KW cooling capacity. A person at rest may be assumed to dissipate heat at a rate of about 360 KJ/hr. There are 10 light bulbs in the room, each with a rating of 100 watts. The rate of heat transfer to the classroom through the walls and the windows is estimated to be 15,00 KJ/hr. If the room air is to be maintained at a constant temperature of 21oC, determine the number of window air- conditioning units required. 1 unit *B. 2 units C. 3 units D. 4 units SOLUTION: Q = total head load = 40(360/3600) + 10(0.100) + 15,000/3600 = 9.167 KW No. of air-conditioning = 9.167/5 = 1.833 = 2 units A 4m x 5m x 6m room is to be heated by a baseboard resistance heater. It is desired that the resistance heater be able to raise the air temperature in the room from 7 to 23oC within 15 minutes. Assuming no heat losses from the room and an atmospheric pressure of 100 kpa, determine the required power of the resistance heater. Assume constant specific heats at room temperature. 2.34 KW *B. 1.91 KW C. 4.56 KW D. 6.34 KW SOLUTION: w = P / R T = 100 / (0.287)(7 +273) = 1.244 kg / m3 m = 1.244 (4 x 5 x 6) = 149.28 kg Q = m cv (t2 – t1 ) = 149.28 (0.7186)(23 – 7) = 1,716.36 KJ Power = 1,716.36 / (15 x 60) = 1.91 KW

A student in a 4m x 6m x 6m dormitory room turns on her 150 watts fan before she leaves the room on a summer day, hoping that the room will be cooler when she comes back in the evening. Assuming all the doors and windows are tightly closed and disregarding any heat transfer through the walls and the windows, determine the temperature in the room when she comes back 10 hours later. Use specific heat values at room temperature, and assume the room to be at 100 kpa and 15 oC in the morning when she leaves 28.13oC B. 38.13oC C. 48.13oC *D. 58.13oC SOLUTION: w = P / R T = 100 / (0.287)(15 + 273) = 1.2098 kg / m 3 m = 1.2098(4 x 6 x 6) = 174.216 kg Q = m cv (t2 – t1) 0.15(10 x 3600) = 174.216 (0.7186)(t2 – 15) t2 = 58.13oC A piston cylinder device whose piston is resting on top of a set stops initially contains 0.50 kg of helium gas at 100 kpa and 25oC. The mass of the piston is such that 500 kpa of pressure is required to raise it. How much heat must be transferred to the helium before the piston starts rising? 1557.13 KJ B. 1657.13 KJ C. 1757.13 KJ *D. 1857.13 KJ SOLUTION: For helium: cv = R / (k-1) = (8.314 / 4) (1.667 – 1) = 3,116 KJ/ kg-K T2 = (25 + 273)(500 / 100) = 1,490oK T1 = 25 + 273 = 298o Q = m cv (T2 – T1) = 0.50(3.116)(1490 – 298) = 1857.13 KJ In order to cool 1 ton (100kg) of water at 20oC in an insulated tank, a person pours 80 kg of ice at -5oC into the water. Determine the final equilibrium temperature in the tank. The melting temperature and the hat of fusion of ice at atmospheric pressure are 0 oC and 333.7 kJ/kg, respectively. *A. 12.43oC B. 14.43oC C. 16.43oC D. 18.43oC SOLUTION: Qwater =Qice 1000(4.187)(20 –te) = 80(2.09)(0 + 5) + 80(333.7) + 80(4.187)(te – 0) te = 14.43oC A fan is powered by a 0.5 hp motor and delivers air at a rate of 85 m 3/min. Determine the highest value for the average velocity of air mobilized by the fan. The density of air to 1.18 kg/m3.

18.23 m/s *B. 21.12 m/s C. 25.34 m/s SOLUTION: P=wQh 0.50(0.746) = ( 1.18 x 0.00981)( 85 / 60) (h)

D. 32.23 m/s

h = 22.74 m v = = 21.12 m/s An Ocean – Thermal Energy Conversion power plant generates 10,000 KW using a warm surface water inlet temperature of 26oC and a cold deep- water temperature of 15oC. ON the basis of a 3oC drop in the temperature of the warm water and a 3oC rise in the temperature of the cold water due to removal and addition of heat, calculate the power required in KW to pump the cold- deep water to the surface and through the system heat.Assume a Carnot cycle efficiency and density of cold water to be 1000 kg/m3. 108 *B. 250 C. 146 D. 160 SOLUTION: e = (Th – TL)/ TH = [(26 + 273) – (15 + 273)] / (26 + 273) = 0.0679 e =W / Qa 0.03676 = 10, 000 / QA QA = 271, 612. 99 KW QR = Qa – W = 271,812.99 – 10, 000 = 261,813 KW QR = m cp (Δt) 261, 813 = m (4.187)(3) M = 20, 643.32 kg/s Q = 20,843.32 kg/s or 20,843.32 li/s = 20.843 m3/s H = P / w = 12 / 9.81 = 1.223 m Wp = w Q h = 9.81(20.843)(1.223) = 250.12 KW A plate – type solar energy collector with an absorbing surface covered by a glass plate is to receive an incident radiation of 800 W/m2. The glass plate has a reflective of 0.12 and a transmissivity of 0.85. The absorbing surface has an absorptivity of 0.90. The area of the collector is 5m2. How much solar energy in watts is absorbing by the collector? 2500 B. 2880 C. 3510 *D. 3060 SOLUTION: Q = heat absorbed from sun Q = (800 W/m2)(5 m2)(0.85)(0.9) = 3,060 watts A tank contains liquid nitrogen at -190oC is suspended in a vacuum shell by three stainless steel rods 0.80 cm in diameter and 3 meters long with a thermal conductivity of 16.3 W/m2-C. If the ambient outside the vacuum shell is 15oC, calculate the magnitude of the conductive heat flow in watts along the support rods. *A. 0.168 B. 0.0587 C. 0.182 D. 0.176 SOLUTION: Q = h A (Δt) = 16.3 (π/4 x 0.0082)(15 – (-190)) = 0.168 watts An elastic sphere containing gas at 120 kpa has a diameter of 1.0 m. Heating the sphere causes it to expand to a diameter of 1.3 m. During the process the pressure is proportional to the sphere diameter. Calculate the work done by the gas in KJ. 41.8 B. 50.6 *C. 87.5 D. 35.4 SOLUTION:

PαD P=kD 120 = k(1) K = 120 P = 120 D V = 4/3 π(D/d)2 = 4/24 π D3 dV = (12/24) π D2 dD W = D3dD W = 87.47 KJ An ideal gas with a molecular weight of 7.1 kg/kg mol is compressed from 600 kpa and 280oK to a final specific volume of 0.5 m3/kg. During the process the pressure varies according to p = 620 + 150v + 95v2 where p is in kpa and v in m3/kg. Calculate the work of compression in KJ/kg? 32.8 B. 28.7 C. 35.6 *D. 33.3 SOLUTION: V1 = R T / P = (8.314 / 7.1)(280) / (600) = 0.546 m3/kg W=

One cubic meter container contains a mixture of gases composed of 0.02 kg-mol of oxygen and 0.04 kg-mol of helium at a pressure of 220 kpa. What is the temperature of this ideal gas mixture in degrees Kelvin? *A. 441 B. 350 C. 400 D. 450 SOLUTION: V = V1 + V2 VT = 1= T = 441 oK Methyl alcohol (CH3OH) is burned with 25% excess air. How much unburned oxygen in kg-mol-oxygen / kg-mol fuel will there be in the products if the combustion is complete? 0.35 B. 0.45 *C. 0.37 D. 0.65 SOLUTION: CH3OH + O2 + (3.76)N2 = CO2 + H2O + (3.76)N2 CH3OH + 1.5O2 + 1.5(3.76)N2 = 1 CO2 + 2H2O + 1.5(3.76)N2 Consider 25% excess air: CH3OH + 1.25(1.5)O2 + 1.25(1.5)(3.76)N2 = 1CO2 + 2H2O + 1.25(1.5)(3.76)N2 + 0.25(1.5)O2 Unburned O2 = 0.25(1.5) = 0.375 A 12 DC electrical motor draws a current of 15 amps. How much work in KJ does this motor produce over a 10 minute period of operation?

*A. 108.0 B. 129.6 C. 216.0 D. 318.2 SOLUTION: W=E=QV W = (15 x 10 x 60)(12) = 108,000 J = 108 KJ A 4 liter (2-liter per revolution at standard pressure and temperature) spark ignition engine has a compression ratio of 8 and 2200 KJ/kg heat addition by the fluid combustion. Considering a cold air-standard Otto cycle model, how much power will the engine produce when operating at 2500 rpm? *A. 166.53 hp B. 73.12 hp C. 97.4 hp D. 148 hp SOLUTION: W = 1.2 kg/m3 (standard density of air) M = 2 li/rev x 2500 rev/min x 1.2 kg/m3 x 1 m3/ 1000 li x 1 min/ 60 sec = 0.10 kg/s e = W / QA e = 1 -1 / 8 1.4 -1 = 0.5647 0.5647 = W / 2200 W = 1,242.34 KJ/kg (0.10 kg/s) = 124.23 KW = 166. 53 hp A simple Rankine cycle produces 40 MW of power, 50 MW of process heated and rejects 50 MW of heat to the surroundings. What is the utilization factor of this cogeneration cycle neglecting the pump work? 50% B. 60% *C. 64% D. 80% SOLUTION: QA = WT + W processW p = 40 + 50 + 50 = 140 KW UF = = 64 % The rate of heat transfer to the surroundings’ from a person at a rest is about 400 KJ/hr. Suppose that the ventilation system fails in an automobile in an auditorium containing 120 people and assuming that the energy goes into the air of volume 1500 m 3 initially at 300oK and 101 kpa, calculate the rate in oC/min of air temperature change. 0.81 B. 0.53 *C. 0.63 D. 1.0 SOLUTION: Q = m cv Δt PV = m RT 101(1500) = m (0.287)(300) m = 1,759.58 kg Q = m cv Δt 120(400 / 60) = 1759.58 (0.7186)(Δt) Δt = 0.633 oC / min An insulated box containing helium gas falls from a balloon 4.5 km above the earth’s surface. Calculate the temperature rise in oC of the helium when box hits the ground. 15.2 B. 12.6 C. 25.3 *D. 14.1 SOLUTION: Cv of helium = 3118.9 j/kg-C m g h = m cv Δt

m (9.81)(4500) = m (3118.9) Δt Δt = 14.15 oC Consider two Carnot heat engines operating in series. The first engine receives heat from the reservoir at 2500oK and rejects the waste heat to another reservoir at temperature T. The second engine receives heat by the first one, convert some of it to work, and rejects the rest to a reservoir at 300oK. If thermal efficiencies of both engines are the same, determine the temperature T. *A. 849oK B. 578 oK C. 763 oK D. 976 oK SOLUTION: et = e 2 T = 849oK An ideal gas mixture consists of 2 kmol of N2 and 6 kmol of CO2. The mass fraction of CO2 is: 0.175 B. 0.250 *C. 0.825 D. 0.750 SOLUTION: 6(44) Mass fraction of CO2 = = 0.825 6(44) + 2(28) An ideal gas mixture consists of 2 kmol of N2 and 6 kmol of CO2. The apparent gas constant of mixture is: *A. 0.208 B. 0.231 C. 0.531 D. 0.825 Solution: M= (2/8)(28) + (6/8)(44) = 40 R= 8.314/M = 8.314/40 = 0.208 KJ/kg-K A Carnot cycle operates between the temperature limits of 300OK and 1500OK, and produces 600 KW of net power. The rate of entropy changes of the working fluid during the heat addition process is: A. 0 B. 0.4KW/K *C. 0.5KW/K D.2.0KW/K Solution: W= ( s) (TH – TL) 600 = ( s) (1500 – 300) ( s)= 0.50 KW/K Air in an ideal Diesel cycle is compressed from 3L to 0.15L and then it expands during the constant pressure heat addition process to 0.3L. Under cold air standard conditions, the thermal efficiency of this cycle is: rk = 3/0.15= 20 rc= 0.3/0.15= 2 1 e=1rkk-1

rck- 1 k(rc – 1)

1 =1201.4-1

21.2 - 1 = 0.6467 1.4 (2-1) =64.67%

Helium gas is an ideal Otto cycle is compressed from 20oC and 2L to 0.25L and its temperature increases by an additional 800oC during the addition process. The temperature of helium before the expansion process is: *A. 1700oC B. 1440oC C. 1240oC D.880oC Solution: rk =2/0 25 = 8 T2 = (20+273) (8)1.667-1 =1,172K T3 = T2 + 800 = 1172 + 800 = 1972oK t3 = 1699oC = 1700oK In an ideal Brayton cycle has a net work output of 150KJ/kg and backwork ratio of 0.4. If both the turbine and the compressor had an isentropic efficiency of 80%, the net work output of the cycle would be. A. 50KJ/kg *B. 75KJ/kg C. 98KJ/kg D.120KJ/kg Solution: Backwork ratio = W O/WT 0.40 = W O/W T WO = 0.40 W T Wnet = W T - W O 150 = W T – 0.4 W T WT =250 KJ/kg WT ‘=250(0.8) = 200KJ/kg WP = 0.40(200) =100KJ/kg WP’ = 100/0.80 =125 KJ/kg Wnet= WT’ – W C’= 200 – 125 = 75 KJ/kg Air entered a turbojet engine at 200 m/s at a rate of 20 kg/s, and exists at 800 m/s relative to the aircraft. the thrust developed by the engine is: A. 6KN *B. 12KN C.16KN D. 20KN Solution: Thrust developed = m(v2 – v1) = 20(800 – 200) = 12,000N = 12KN A thermal power has a net power 10MW. The backwork ratio of the plant is 0.005. Determine the compressor wor. A. 50.15KW B. 50.35KW *C.50.25KW D. 50.45KW Solution: Wnet= WT + WP BW= WP / W T 0.005 =WP / W T WP= 0.005WT Wnet= WT - WP 10,000 = W T – 0.005WT WT= 10,050.25 KW WC= 0.005(10,050.25) = 50.25KW A heat engine receives heat from a source at 1200oK at a rate of 500KJ/s and rejects the waste heat to a sink at 300oK. If the power output of the engine is 200KW, the second law efficiency of the heat engine is: A.35% B.40% *C.53% D.75% Solution:

ea= 200/500 = 0.40 et= (TH – TL)/TH = (1200 – 300)/1200 = 0.75 es= 0.40/0.75= 53.33% A water reservoir contains 100,000 kg of water at an average elevation of 60 m. The maximum amount of electric power that can be generated from this water is: A.8KWh *B.16KWh C.1630KWh D.58, 800KWh Solution: P= m h = (100,000 x 0.00981)(60)= 58,860 KJ P= 58,860 KJ x KWh/3600 KJ = 16.35KWh A house is maintained at 22oC in winter by electric resistance heaters. If the outdoor temperature is 5oC, the second law of efficiency of the resistance heaters is: A.0% *B.5.8% C.34% D.77% Solution: ea= 100% of resistance heaters et= (22 – 5)/(22 + 273) = 5.8% es= 5.8/100 = 5.8% A thermoelectric refrigerator that resembles a small ice chest is powered by a car battery, and has a COP of 0.10. If the refrigerator cools at 0.350L canned drink from 20OC to 4OC in 30 min. determine the average electric power consumed by the thermoelectric refrigerator. *A.130 watts B.110 watts C.120 watts D.140 watts Solution: (1 x 0.35) Q= m cp(t2 – t1) = (4.187)(20 – 4) = 13 watts 30 x 60 COP= RE/Wc 0.10= 13/0.10= 130 watts A Carnot refrigerator operates in a room in which the temperature is 25OC and consumes 2 kW of power when operating. If the food compartment of the refrigerator is to be maintained at 3OC, determine the rate of heat removal from the food compartment. *A.1504.8 kJ/min B.12.86 kJ/min C.1625 kJ/min D.9.57 kJ/min Solution: COP= TL /TH– TL = (3 + 273)/ (25 + 273) – (3+273) =12.54 QL= COP x W = 12.54 x 2(60) = 1504.8 kJ/min A household refrigerator with EER 8.0 removes heat from the refrigerated space at a rate of 90 kJ/min. Determine the rate of heat transfer to the kitchen air. A.101.25 kJ/min B.63.05 kJ/min *C.128.46 kJ/min D.80 kJ/min Solution: COP= EER /3.412 = 8/ 3.412 = 2.34 COP= QL /QH – QL = 2.34 = 90 / QH – 90 QH=128.46Kj/min An air-conditioning system is used to maintain a house at 75OF when the temperature outside is 95OF. The house is gaining heat through the walls and windows at a rate of 1250 Btu/min, and the heat generation rate within the house from people, lights and

appliances amounts to 350 Btu/min. Determine the minimum power input required for this air-conditioning system. A.10.06 hp B.1.36 hp *C.1.41 hp D.7.94 hp

Solution: QL= 1250 + 350 = 1600 Btu/min COP= TL / TH – TL= (75 + 460)/(95 + 460) - (75+460) = 26.75 W= QL /COP = (1600 / 26.75) / 42.4 = 1.41 hp A refrigeration system is to cool bread loaves with an average mass of 450 g from 22 OC to -10OC at a rate of 500 loaves per hour by refrigerated air. Taking the average specific and latent heats of bread to be 2.93 kJ/kg, OC and 109.3 kJ/kg, respectively, determine the product load. A.541.7 kJ/min B.351.6 Kj/min *C.761.5 kJ/min D.409.9 kJ/min Solution: Mbread= (500 breads/h) (0.45 kg / bread) = 225 kg/h Qtotal= Qbread + Qfreezing = (mcp∆T)breadl (mhlatent)bread = (225)(2.93)[22-(-10)] l (225)(109.3) Qtotal= 45,688.5 kJ/h = 761.5 kJ/min A house that was heated by electric resistance heaters consumed 1200 kWh of electric energy in a winter month. If this house were heated instead by a heat pump that has an average performance factor, PF of 2.4, determine how much money the homeowner would be saved that month. Assume a price of 0.085$/kWh for electricity. A. $42.5 *B. $59.50 C.$109 D.$97.75 Solution: W= QH/PF = 1200kWh / 2.4 = 500 kWh $ Savings per month = (1200 – 500) (0.085) = $59.50 An ammonia simple saturation cycle operates with a suction pressure of 291.6 kPa and a condenser pressure of 1204 kPa develops 15 tons of refrigeration. Determine the theoretical horsepower of the compressor. The following enthalpies have been found: condenser entrance = 1653 kJ/kg, exit =346.6 kJ/kg, compressor entrance = 1450.2 kJ/kg, exit= 1653kJ/kg. A.7.23 hp *B.13 hp C. 15 hp D.8.23 hp Solution: m= Qe/ (h1 – h4) = (15 x 3.52) / (1450.2 – 346.6) = 0.0478 kg/s W= m (h2 – h1) = (0.0478)(1653 – 1450.2) / 0.746 = 13 hp An ammonia ice plant operates between a condenser temperature of 35 OC and evaporator of -15OC. It produces 10 metric tons of ice per day from water at 30OC to ice at -5OC. Assuming simple saturation cycle, determine the horsepower of the motor if the adiabatic efficiency of the compressor ηc=0.85 and mechanical efficiency ηm=0.95. The specific heat of ice is 2,094kJ/kg. OC and the latent heat is 335kJ/kg. From the table for ammonia the following enthalpies are: condenser entrance = 1703 kJ/kg, exit= 366.1 kJ/kg; compressor entrance= 1443.9kJ/kg, exit = 1703kJ/kg A.17.68 hp B.18.61 hp C.15.5 hp *D.21.9 hp Solution:

qe=cpa (te – tf) lhlatentlcph(tf – ts) = (4.187) (30 – 0) + 335 + (2.094) [(0- (-5)] = 471.08 kJ/kg Qe= (10 x 1000) (471.08) / 24 = 196,283.33 kJ/hr= 54.523kJ/s m= Qe / (h2 – h4) = (54.523)/ (1443.9 – 366.1) = 0.05059kg/s W= m (h2 – h1) = (0.0509) (1703 – 1443.9)/ 0.746 = 17.68 hp Wmotor= 17.68 / (0.85) (0.95) =21.9 hp A Freon 22 air conditioning under standard operating conditions of 35OC is condensing and 5OC evaporating temperatures. The volume flow rate entering the compressor is 23.72 L/s. determine the refrigerating capacity if the refrigerating effect is 164 kJ/kg. From the table for R22 the specific volume at the compressor entrance is 40.36L/kg. A.393.3 TR B.79.3 TR C.96.4 TR *D.27.4 TR Solution: m= V1/v1= 23.72 / 40.36 = 0.5877 kg/s Qe= m (qe) = 0.5877 (164)/ 3.52 = 27.4 TR

The refrigerant volume flow rate at the entrance of compressor were obtained from a test on a twin cylinder single acting 15 cm x 20 cm, 320 rpm compressor ammonia refrigerating plant 33 L/s. Determine the volumetric efficiency of the compressor. A.77.65% *B.87.6% C.97.6 TR D.65.65% Solution: VD = (π D2 L/4) N= (π /4) (0.15)2(0.2) (320) (2) = 2.26 m3/min nv =V1/VD = 33/2.26 (1000/60) = 0.876 or 87.6 % A twin cylinder ammonia compressor with volume displacement of 14,726 cm 3 operates at 300 rpm. Condenser and evaporator pressure are 1200 kPa and 227 kPa respectively. Specific volume of refrigerant at the entrance of compressor is 528.26 L/kg. Compression process is polytrophic with a 1.20 and clearance factor of compressor is 2%. Determine horsepower required. A.60hp B.70hp C.80hp D.90hp Solution: nv= 1+c-c(p2/p1)1/n= 1 + 0.02 – 0.02(1200/227)1.20= 0.8725 VD= (π D2 L/4) N= (0.014726) (320) (2) = 9.424 m3/min V1= (nv) VD- (0.8725) (9.424) - 8.2224 m3/min W= (n p1 V1 /1-n) [(p2/p1) n-1/n-1] = [(1.20 x 2.27 x 8.2224)/ (1-1.20)] [(1200/2.27)1.2-1/1.2 -1] = 3582 kJ/min W= 3582/ (60) (0.746) = 80 hp A reversed Carnot cycle has a refrigerating COP of 2.5. Determine the ratio T H/TL? A.1.4 B.1.5 C.1.25 D.1.2 Solution: COP= TH/ (TH-TL) 1/COP = (TH-TL)/TL= TH/TL-1 TH/TL =1 + 1/COP =1 +1/2.5 =1.4

Three thousand cubic feet per minute of air are circulated over an air-cooled condenser. If the load on the condenser is 64,800 Btu/hr, compute the temperature rise of the air passing over the condenser. Specific volume of standard air (13.34ft3/lb) A.10OF B.15OF C.20OF D.25OF Solution: Qc= m c ∆t ∆t = Qc/ m c = 64,800/ [3000 (60) / (13.34)] (0.24) = 20OC Saturated vapor ammonia at -16OC (h1 = 1442.60 kJ/kg) leaves the evaporator and enters the compressor at -6OC (h1 = 1465kJ/kg. The refrigerant leaves the condenser as saturated liquid at 40OC (h4= 390.6 kJ/kg) and enter the expansion valve at 35OC (h5 =366.1kJ/kg). Heat rejected from the compressor amount to 50kW. The work to compressor is 208kJ/kg, while the heat loss from compressor is 33kJ/kg. If 95kJ/kg of heat are lost in the piping between the compressor discharge and condenser inlet, determine the refrigerating capacity of the system. A.49.5 TR B.46.61 TR C.12.88 TR D.13.24 TR Solution: Solving for the enthalpy at the exit of compressor using energy balance about the compressor h2 = h1 + w - qwc = 1465 + 208 – 33= 1640 kJ/kg Solving for the enthalpy at the entrance of condenser using energy balance about piping from compressor exit to condenser entrance h3= h2 – q2-3= 1640 – 95 = 1545kJ/kg Solving for heat rejected in condenser using energy balance about the condenser qc = h3 – h4 = 1545 – 390.6 = 1154.4kJ/kg m = Qc/qc = 50/1154.4 = 0.0433kg/s Solving for refrigerating effect using energy balance about the evaporator h5 – h4 = 366.1kJ/kg qe= h1 – h5 = 1442.6 – 366.1 = 1076.5kJ/kg Therefore, the refrigerating capacity Qe= m qe = [(0.0433) (1076.5)] / 3.52 = 13.24 TR An actual refrigerating cycle using R12 as working fluid, the refrigerant flow rate is 0.05 kg/s. Vapor enters the expansion valve at 1.15 MPa, 40OC (h = 238.5kJ/kg) and leaves the evaporator at 175 kPa, -15OC (h = 345kJ/kg). The electric input to motor driving the compressor is measured and found 3.0 kW. Motor efficiency at this load is 92% and mechanical efficiency 82%. Determine the actual coefficient of performance for this cycle. A.1.58 B.2.36 C.1.78 D.1.34 Solution: Qe= (0.05) (345 - 238.5) = 5.325kW W = (3) (0.92) (0.82) = 2.26kW COPactual= 5.325 / 2.26 = 2.36

An ammonia refrigeration system the temperature in the evaporator is -12OC and the ammonia at the evaporator entry is 0.1511 dry while at exit is 0.95 dry. If the rate of ammonia circulation is 5.64 kg/min, determine the refrigerating capacity of the system. Enthalpy of saturated liquid and vapor at -12OC is 144.929kJ/kg and 1447.74kJ/kg respectively. A.17.82 B.34.82TR C.27.82TR D.4.82TR Solution: h4= hf4 + x4 (hg4 – hf4) = 144.929 + (0.1511) (1447.74 – 144.929) = 341.78 h1= hf1 + x1 (hg1 – hf1) = 144.929 + (0.95) (1447.74 – 144.929) = 1382.6kJ/kg Qe = m (h1 – h4) = (5.64) (1382.6 – 341.78) / 211 = 27.82 TR A two stage cascade vapor compression refrigeration system uses ammonia in the lowtemperature loop and R-12 in the high-temperature loop. The ammonia provides 15 tons cooling. If the high-loop temperature requires 10.12 kW compressor power and low-loop 15.93 kW, determine the COP of the system. A.2.027 B.5.22TR C.3.314TR D.9.1TR Solution: Wtotal= 10.12 + 15.93 = 26.05 kW COP = 15 (3.52) /26.05 = 2.027 When a man returns to his well-sealed house on a summer day, he finds that the house is at 32OC. He turns on the air conditioner, which cools the entire house to 20 OC in 15min. If the COP of the air-conditioner system is 2.5, determine the power drawn by the air conditioners. Assume the entire mass within the house is equivalent to 800 kg of air for which c= 0.72kJ/kg OC A.7.68Kw B.3.07kW C.19.2kW D.12.03kW Solution: Qe = m c (t1 – t2) / ∆time = (800) (0.72) (32 – 20) / (15 x 60) = 7.68kW W= Qe / COP = 7.68 / 2.5 = 3.07kW It is desired to double the COP of a reversed Carnot engine for cooling from 5.0 by raising the temperature of the heat addition while keeping the temperature of the heat rejection constant. By what percentage must the temperature of heat addition be raised? A.10.1% B.9.1% C.8.1% D.7.1% Solution: Percent increase of the temperature = (TL – TL) TL = TL / TL – 1 of heat addition Original COP: 5 = TL / (TH – TL); (TH – TL) TL = 1/5; TH / TL -1 = 1/5 = 02; TH / TL = 1.2 eq.1 Doubling the COP: 10 = TL / (TH – TL); (TH – TL) TL = 1/10; TH / TL – 1 =1/10 = 0.1; TH/TL=1.1 Dividing eq.1 and eq.2: (TH/TL) (TH/TL) = 1.2 / 1.1 = 1.091 Percent Increase of heat addition = 1.091 – 1 = 0.091 or 9.1%

An ammonia water-cooled compressor receives the refrigerant at specific volume 62 L/kg. It has a piston displacement rate of 3m3/min. If a squirrel cage motor is running at 1200 rpm drives the compressor and average piston speed is 490m/min, calculate size of cylinder bore. A.20.4 cm B.26.0 cm C.16.13 cm D.13.6 cm Solution: Piston speed = 2LN = 490 = 2(L) (1200); L= 0.204 m = 20.4 cm Vp = (π D2 L/4) N = (π/4) D2 (0.204) (1200) =5 D = 0.1613m = 16.13cm If the initial volume of an ideal gas is compressed to one-half its original volume and to twice its temperature, the pressure: A. Doubles B. Quadruples C. Remains constant D. Halves Solution: P1V1/T1 = P2V2/T2 P1V1/T1= P2 (1/2 V1) / (2T1) P2 = 4 P 1 If the gage pressure of a medium is 30kPa (vacuum) and the atmospheric pressure is 101.3kPa, the absolute pressure will be: A.131.3kPa B.-71.3kPa C.71.3kPa D.-131.3kPa Solution: Pabs = Patm - Pvac = 101.3kPa - 30kPa = 71.3kPa If a particle has a velocity of 4 meters per second and a kinetic energy of 144 Joules, then the mass, in kilograms of this particle must be: A.44 B.16 C.18 D.24 Solution: KE = (1/2) (m) (V2) / 2k 144 – (1/2) (m) (4)2 / [(2) (1)] m = 18kg A condenser vacuum gauge reads 715 mm Hg when the barometer stands at 757 mm Hg. State the absolute pressure in the condenser in kN/m or kPa. A.5.6kPa B.5.9kPa C.6.5kPa D.5.2kPa Solution: Pabs = Patm – Pvac = 757 – 715 = 42 mm Hg (101.325kPa/760 mm Hg) = 5.60kPa Determine the force in Newton in a piston of 465 mm2 area with a pressure of 0.172MPa. A.65N B.72N C.80N D.111 Solution: F= PA – (0.172MPa) (105 Pa/MPa) (465 mm2) (m2 / 105 mm2) F = 79.98N One piston of a hydraulic press has an area of 1cm2. The other piston has an area of 25cm2. If a force of 130N is applied on the smaller piston, what will be the total force on the larger piston is both piston surfaces are the same level? A.6N B.175N C.3750N D.4250N Solution:

F1/A1 = F2/A2 (150/1) = (F2/ 25) F2 = 3750N If the pressure of a confined gas at a constant temperature is tripled, what will happen to the volume? A. The volume will be tripled B. The volume will remain C. The volume will be reduced to one-third of its original value D. The volume is constant Solution: P1V1 = P2V2 P1V1 = (3P1) V2 V2 = (1/3) V1 The work done on air is 10.86kJ/kg, determine the compressor power if it is receiving 272 kg/min if air. A.36.72 hp B.49.23 hp C. 29.54 hp D.66 hp Solution: W= (10.86) (272.60) = 49.232 kJ/s or kW = 49.232 kW (1 hp/ 0.746kW) = 65.99 hp A water tank of 18 ft long and 4 ft high, calculate the pressure at the bottom of the tank. A.1.733psi B.1.999psi C.2.337psi D.3.773psi Solution: P= (62.4 lbf/ft3) (4ft) (1ft2 / 144 in2) = 1.733psi The pressure of 750 mm Hg in kN/m2 A.90 B.100 C.103 D110 Solution: P= 750 mm Hg (101.325 kPa/ 760 mm Hg) = 99.99 kPa A double purpose tank 18 ft wide, 24 ft long and 4 ft depth is filled with water. What is the weight of water in the tank in long tons? A.49tons B.48tons C.54tons D.50tons Solution: W = (62.4 lb/ft2) [(18) (24)(4)] ft3 = 107,827.2 lbf = 107,827.2 lb (1ton/ 2200 lb) = 49tons Oil flow through a 16 tubes on a single cooler with a velocity of 2 m/s, the internal diameter of the tube is 30mm and oil density is 0.85gm/ml. Find the volume flow in liters per sec. A.22.62 B.32.22 C. 62.22 D.42.62 Solution: Volume flow rate = (3.1416) (0.015)2 (2) (16) = 0.02262 m3/s or 22.62 liters/s A substance temperature was 620OR. What is the temperature in OC? A.50.7 B.45.54 C.71.11 Solution:

D.94.44

T, OC = [(620 – 460) – 32] (5/9) = 71.11 Unknown volume of container gas of gas of 1 atmosphere is allowed to expand to another container of 10 m3 volume at 500 mm Hg at constant temperature. Find the unknown volume. A.6.58 m3 B.6.75 m3 C.5.67 m3 D.7.65 m3 Solution: P1V1 – P2V2 (760) V1 = (500) (10) V1= 6.58 m3 An iron block weighs 5 Newton and has volume of 200 cm 3. What is the density of the block? A. 2458kg/m3 B. 2485 kg/m3 C. 2584 kg/m3 D. 3 2549 kg/m Solution: Density = specific weight [at sea level or near the surface of the earth] = (5N/200 cm) (10 cm3/m3) (1kg/9.8066N) = 2549.30 kg/m3 If air is at a pressure of 22.22 psia and at temperature of 800 OR, what is the specific volume? A.11.3 ft3/lbm B.33.1 ft3/lbm C.13.3 ft3/lbm D.31.3 ft3/lbm Solution: Pv = RT v = (53.34) (800) / [(22.22) (144)] = 13.33 ft3/lbm The specific gravity of mercury is 13.55. What is the specific weight of mercury? A.123.9 kN/m3 B.139.2 kN/m3 C.139.9 kN/m3 D.193.2 kN/m3 Solution: ? = (13.55) (9.8066) = 132.88 kN/m3 The equivalent weight of mass 10 kg at a location where the acceleration of gravity is 9.77 m/sec2 A. 97.7N B.79.7N C.77.9N D.977N Solution: Weight = mg/k = [(10) (9.77)] /1 =97.7N A transportation company specializes in the shipment of pressurized gaseous materials. An order is received from 100 liters of a particular gas at STP (32OF and 1 atm). What minimum volume tank is necessary to transport the gas at 80OF and maximum pressure of 8 atm? A.16 liters B.14 liters C.10 liters D.12 liters Solution: P1V1/ T1 = P2V2/ T2 [(1) (100)/ (32 + 460)] / [(8) (V2)/ (80 + 460) V2 = 13.72 liters

100 g of water are mixed with 150 g of alcohol (density = 790 kg/m 3). What is the specific volume of the resulting mixtures, assuming that the fluids mixed completely? A.0.82x10-3 cu.m/kg B.0.88x10-3 cu.m/kg C.0.63x10-3 cu.m/kg D.1.16x10-3 cu.m/kg Solution: Mass of mixtures = 100 + 150 =250 g Volume of mixture = [(0.100) / (1000)] + [(0.150)/ (790)] = 0.00029 m 3 Specific volume of mixture = (0.00029)/ (0.250) = 1.16x10-3 cu.m/kg How much does 30 lbm weigh on the moon? (gmoon = 5.47 ft/s2 A.20 lbr B.3.2 lbr C.3.4 lbr D.5.096 lbr Solution: Weight = mg/k = {[(30) (5.42)]/32.174} = 5.1 lbf A 10 kg block is raised vertically 3 meters. What is the change in potential energy? A.320 J B.350 kg-m2/s2 C.294 J D.350 N-m Solution: PE = mgs/k = {[(10) (9.8066) (3)]/1} = 294.2 J How many cubic meters is 100 gallons of liquid? A.3.7850 cu.m B.0.1638 cu.m C.0.3785 cu.m D.1.638 cu.m Solution: 100gal (785 liters/gal) (1m3/1000 liters) = 0.3785 m3 Steam turbine is receiving 1014 lbm/hr of steam, determine the horsepower output of the turbine if the work done by steam is 251 Btu/lbm A.100 hp B.462.7 hp C.200 hp D.6002.7 hp Solution: W= (251 Btu/lbm) (1014 lbm/hr) 1hp/2545 Btu/hr) = 100 hp What is the resulting pressure when one pound of air at 15 psia and 200 OF is heated at constant volume to 800OF? A.52.1 psia B.15 psia C.28.6 psia D.36.4 psia Solution: P1/T1 = P2/T2 P2 = [(800+460) / (200+460)] (15) = 28.64 psia A bicycle tire has a volume of 600 cm3. It is inflated with carbon dioxide to pressure of 551.43kPa at 20OC. How many grams of CO2 are contained in the tire? Rcoz = 0.18896 kJ/kg.K A.5.98 g B.6.43 g C.4.63 g D.3.83 g Solution: m = rv/RT = [(551.43) (600)/ (102)] / [(0.18896) (20+273)] = 0.00598 kg or 5.98 g Is the most common dryer used which consist of rotating cylinder inside which the materials flow while getting in contact with hot gas. A. Tower dryer B. Centrifugal dryer C. Tray dryer D. Rotary dryer

ANSWER: D Is the ratio of the mass water-vapor in air and the mass of air if it is saturated is called: A. Humidity ratio B. Mass ratio C. Vapor ratio D. Relative humidity ANSWER: D The hands feel painfully cold when the skin temperature reaches: A.8OC B.10OC C.12OC D.14OC ANSWER: B The refrigerant used in steam jet cooling is: A. Steam B.R-11 C. Ammonia D. Water ANSWER: D The total heat of the air is a function of A. WB temperature B.DP temperature C.DB temperature D.WB depression ANSWER: A Boiling point of Freon-12 at atmospheric pressure is: A. 21OF B.15OF C.5OF D.28OF ANSWER: A Which of the following is NOT a type of water cooled condenser in refrigeration? A. Double pipe B. Double shell C. Shell and coil D. Shell and tube ANSWER: B Component of absorption refrigeration system in which the solution is cooled by cooling water: A. Rectifier B. Generator C. Evaporator D. Absorber ANSWER: D Cascade refrigeration cycle is often used in industrial process where objects must be cooled to temperature below: A. -46OC B. -56OC C. -66OC D. -76OC ANSWER: A Type of refrigerant control designed to maintain a pressure difference while the compressor is operating. A. Thermostatic expansion valve B. Automatic expansion valve C. Using low side float flooded system D. Capillary tube ANSWER: D As a rule of thumb, for a specified amount of compressed air, the power consumption of the compressor decreased by for each 3OC drop in the temperature inlet air to the compressor. A. 1% B. 1.5% C. 2% D. 2.5% ANSWER: A Modern way of detecting air compressor leak is by using A. Soup and water B. Air leak detector C. Acoustic leak detector D. Ammonia leak detector ANSWER: C For foundation of stacks, the maximum pressure on the soil is equal to the pressure due to the weight and the .

A. Soil movement B. Wind movement C. Ground movement D. Engine movement ANSWER: B Foundation bolts of specified size should be used and surrounded by a pipe sleeve with an inside diameter of at least A. 3 times the diameter of engine bolt B. 3 times the diameter of anchor bolt C. 2 times the diameter of engine bolt D. 2 times the diameter of anchor bolt ANSWER: B For multi stage compression of an ideal Brayton cycle, the backwork ratio will A. Increase B. Decrease C. Remain the same D. none of these ANSWER: B Type of turbine that has a specific speed below 5 A. Impulse turbine B. Propeller turbine C. Francis turbine D. Deriaz turbine ANSWER: A A high discharge type of turbine A. Impulse turbine B. Francis turbine C. Propeller turbine D. Deriaz turbine ANSWER: C Use to minimize the speed rise due to a sudden load rejection A. needle valve B. wicket gate C. Shut-off valve D. jet deflector ANSWER: D Is the speed of a turbine when the head on the turbine is one meter? A. specific speed B. rated speed C. utilized speed D. unit speed ANSWER: D Is a fluid property which refers to the intermolecular attraction by which the separate particles of the fluid arc held together? A. Cohesion B. Adhesion C. Surface tension D. hypertension ANSWER: A Which of the following is NOT the cause of black smoke in diesel engine? A. fuel valve open too long B. high compression pressure C. carbon in exhaust pipe D. overload on engine BB94-BB102 Which of the following is not a method of starting a diesel engine? Manual rope, crank and kick Electric (battery) Compressed air Using another generator ANSWER D Two-stroke engine performs to complete one cycle

Suction and discharge stroke Power and exhaust stroke Compression and power stroke Suction and exhaust stroke ANSWER C A type of geothermal plant used when there is a presence of brine extracted from underground Dry geothermal plant Double-flesh geothermal plant Single-flash geothermal plant Binary geothermal plant ANSWER D Is the most important safety device on the power boiler Check valve Gate valve Safety valve Globe valve ANSWER C During hydrostatic test, the safety valves should be Removed Open Closed Partially closed ANSWER A Where deaerating heaters are not employed, it is recommended that the temperature of the feed less than 197°C 102°C 104°C 106°C ANSWER A Is a reaction during which chemical energy is released in the form of heat? Cosmic reaction Ethnic reaction Endothermic reaction Exothermic reaction ANSWER D By reheating the steam in an ideal Rankine cycle the heat rejected will: Increase Decrease Remains the same None of these above ANSWER A By increasing the boiler pressure in Rankine cycle the moisture content at boiler exit will: Increase

Decrease Remains the same None of these above ANSWER A Presently the highest steam temperature allowed at the turbine inlet is about 340°C 520°C 620°C 1020°C ANSWER C The most common gases employed in Stirling and Ericsson cycles are: Air and helium Oxygen and helium Hydrogen and helium Nitrogen and helium ANSWER C In most common designs of Gas turbine, the pressure ratio ranges from 10 to 12 11 to 16 12 to 18 15 to 20 ANSWER B In Brayton cycle, the heat is transformed during what process? Constant temperature Isentropic process Isobaric process Isochoric process ANSWER C The fuel injection process in diesel engine starts when piston Is at the TDC Leaving TDC Approaches TDC Halfway of the stroke ANSWER C If the cut-off ratio of diesel cycle increases, the cycle efficiency will Decrease Increase Remains the same None of these ANSWER A The fuel used in a power plant that is used during peak periods Gas Solid Liquid None of these ANSWER C

Typical compression ratio of Otto cycle is 6 8 10 12 ANSWER B If Joule Thompson coefficient is equal to zero, then the process will become Isentropic Isenthalpic Isobaric Isothermal ANSWER D If the fluid passed through a nozzle its entropy will: Increase Decrease Remains the same None of these ANSWER C Refrigerants consisting of mixtures of two or more different chemical compounds, often used individually as refrigerants for other applications Suspension Compound reaction Blends Mixing of refrigerants ANSWER C Pairs of mating stop valves that allow sections of a system to be joined before opening these valves or separated after closing them Check valve Gate valve Safety valve Companion valve ANSWER D An enclosed passageway that limits travel to a single path Corridor Hallway Lobby Tunnel ANSWER A For Immediate Dangerous to Life or Health (IDHL) the maximum concentration from which unprotected persons have time to escape within without escape-impairing symptoms or irreversible health 16 minutes 1 minute 20 minutes 30 minutes ANSWER D

The volume as determined from internal dimensions of the container with no allowance for the volume of internal parts Internal allowance volume Internal gross volume Internal interference volume Internal fots volume ANSWER B A waiting room or large hallway serving as a waiting room Terrace Test room Compound room Lobby ANSWER D A continuous and unobstructed path of travel from any in a building or structure to a public way Average of aggress Mean o aggress Hallway of aggress Pathway of aggress ANSWER B Any device or portion of the equipment used to increase refrigerant pressure Pressure relief device Pressure-imposing element Pressure lift device Pressure limiting device ANSWER B The quantity of refrigerant stored at some point is the refrigeration system for operational, service, or standby purposes Pressure vessel Pumpdown charge Liquid receiver Accumulator ANSWER B Secondary refrigerant is a liquid used for the transmission of heat, without a change of state, and having no flash point or a flash point above as determined from ASTM 150°F 160°F 180°F 200°F ANSWER A A service valve for dual pressure-relief devices that allows using one device while isolating the other from the system maintaining one valve in operation at all times Three-way valve Two-way valve One-way valve Four-way valve

ANSWER A Tubing that is unenclosed and therefore exposed to crhsing, abrasion, puncture, or similar damage after installation Protected tubing Bare tubing Open tubing Unprotected tubing ANSWER D Refers to blends compromising multiple components of different volatile that, when used in refrigeration cycles, change volumetric composition and saturation temperature as they evaporate (boil) or condense at constant pressure Zeoline Blending Composition Zertropic ANSWER D Is a premises or that portion of a premise from which, because they are disabled, debilitated, or confined, cooperate cannot readily leave without the assistance of others Institutional occupancy Public assembly occupancy Residential occupancy Commercial occupancy ANSWER A Is one in which a secondary coolant is in direct contact with the air or other substance to be cooled or heated Double indirect open spray system Indirect closed system Indirect open spray system Indirect vented closed system ANSWER B Refrigerant number R-744 is: Butane Carbon monoxide Propane Carbon dioxide ANSWER D Refrigerant number R-1150 is: Propylene Ethane Ethane Methyl formale ANSWER B Refrigerant number R-40 is: Chlorodifluoromethane Difluoromethane Ammonia

Chloromethane ANSWER D When the air duct system serves several enclosed spaces, the permissible quantity of refrigerant in the system shall not exceed the amount determined by using the total volume of those spaces in which the airflow cannot be reduced to less than of its maximum when the fan is operating One-quarter One half-quarter Three-quarter One fourth-quarter ANSWER A The space above a suspended ceiling shall not be include 1 in calculating the permissible quantity of refrigerant in the system unless such space is continuous and is part of the air return system Partition Plenum Separator Plate divider ANSWER B Which of the following is NOT a possible location of service valve? Suction of compressor Discharge of compressor Outlet of liquid receiver Outlet of condenser ANSWER D A cool to serve with evaporator that is use to prevent the liquid refrigerant entering the compressore Accumulator Liquid superheater Drier loop Liquid suction heat exchanger ANSWER C A type of valve connected from discharge of compressore directly to suction that is normally closed and will open automatically only if there is high discharge pressure Check valve Solenoid valve King valve Relief valve ANSWER B Use to increase the capacity of condenser Water regulating valve Desuperheating coils Liquid-suction heat exchanger Condenser heating coils ANSWER B Is use to subcooled the refrigerant from the condenser

Liquid subcooler Condenser subcooler Desuperheating coils Liquid receiver ANSWER A Which of the following is NOT a part of low pressure side in refrigeration system? Liquid line Refrigerant flow control Evaporator Suction line ANSWER A Which of the following is NOT a part of high pressure side in refrigeration system? Compressor Condenser Liquid line Suction line ANSWER D Which of the following is NOT a part of condensing unit? Compressor Discharge line Condenser Liquid line ANSWER D By subcooling the refrigerant in refrigerant system, the compressor power per unit mass will Increase Decrease Remains the same None of these ANSWER C Superheating the refrigerant in refrigeration system without useful cooling, the refrigeration effect per unit mass will Increase Decrease Remains the same None of these ANSWER C Which of the following is NOT a type of air-cooled condenser? Shell and tube Natural draft Forced draft Induced draft ANSWER A A type of refrigerant control typically used in household refrigeration Thermostatic expansion valve Automatic expansion valve

Capillary tube High side float ANSWER C Type of condenser that operates like cooling tower Air-cooled condenser Evaporative condenser Shell and tube condenser Water-cooled condenser ANSWER B The major problem of heat pump is Refrigerant used Outside air Supply air Frosting ANSWER D Dominant refrigerant used in commercial refrigeration system R-11 R-22 R-12 R-502 ANSWER D Cascade refrigeration system are connected in Series Parallel Series-parallel Parallel-series ANSWER A Is use to heat up the solution partially before entering the generator in absorption refrigeration system Rectifier Absorber Regenerator Pump ANSWER C The COP of actual absorption refrigeration system is usually Less than 1 Less than 2 Less than 3 Less than 4 ANSWER A Sight glass is often located at: Discharge line Liquid line Between condenser and liquid receiver Suction line ANSWER B

Use to detects a vibration in current caused by the ionization of decomposed refrigerant between two opposite-charged platinum electrodes Electronic detector Halide torch Bubble method Pressurizing ANSWER B Joints and all refrigerants containing parts of a refrigerating system located in an air duct carrying conditioned air to and from an occupied space shall be constructed to withstand a temperature of without leakage into the airstream 550°F 600°F 650°F 700°F ANSWER D Refrigerant piping crossing an open space that affords passageway in any building shall be less than above the floor unless the piping is located against the ceiling of such space and is permitted by the authority having jurisdiction 2.2 m 3.2 m 4.2 m 5.2 m ANSWER A Methyl chloride shall not be in contact with Aluminum Zinc Magnesium All of these ANSWER D Shall not be in contact with any halogenated refrigerants Aluminum Zinc Magnesium All of these ANSWER C Are suitable for use in ammonia system Copper Aluminum and its alloy Plastic Cast iron ANSWER B If a pressure-relief device is used to protect a pressure vessel having an inside dimension of 6 in or less, the ultimate strength of the pressure vessel so protected shall be sufficient to withstand a pressure at least the design pressure 2 times 3 times

4 times 5 times ANSWER B Seats and discs shall be limited in distortion, by pressure or other cause, to a set pressure change of not more than in a span of five years 1% 5% 10% 50% ANSWER B Liquid receivers, if used, or parts of a system designed to receive the refrigerant charge during pumpdown charge. The liquid shall not occupy more than of the volume when temperature of the refrigerant is 90°F 80% 85% 90% 95% ANSWER C The discharge line (B4) shall be vented to the atmosphere through a fitted to its upper extremity Nozzle Convergent-divergent nozzle Pipe Diffuser ANSWER D Convert fossil fuels into shaft work Nuclear power plant Gas turbine power plant Dendrothermal power plant Thermal power plant ANSWER D Ultimate strength drops by 30% as steam temperature raises from for unalloyed steel 300 to 400°C 400 to 500°C 600 to 700°C 700 to 800 °C ANSWER B Recent practice limits steam temperature to 438°C 538°C 638°C 738°C ANSWER A In a closed feed water heater, the feed water pass through Inside the tube

Outside the tube Inside the shell Outside the shell ANSWER A In use if extracted steam upon condensation gets subcooled Trap Deaerator Filter Drain cooler ANSWER D Needs only single pump regardless of number of heaters Open heater Closed heater Mono heater Regenerative heater ANSWER B Is also known as deaerator Open heater Closed heater Reheat heater Regenerative heater ANSWER A Dissolve gases like makes water corrosive react with metal to form iron oxide O2 and N2 O2 and CO O2 and CO2 N2 and SO2 ANSWER C A cycle typically used in paper mills, textile mills, chemical factories, sugar factories and rice mills Cogeneration cycle Combined cycle By-product cycle Cascading cycle ANSWER A When process steam is basic need and power is byproduct, this cycle is known as Cogeneration cycle Combined cycle By-product cycle Cascading cycle ANSWER C A type of turbine employed where steam continuously extracted for process heating Back pressure turbine Gas turbine Steam turbine Passout turbine

ANSWER D Which of the following is used for Binary cycle power generation for high temperature application Mercury Sodium Potassium All of these ANSWER D critical temperature of mercury is 1160°C 1260°C 1360°C 1460°C ANSWER D Critical pressure of mercury is 100 Mpa 108 Mpa 128 Mpa 158 Mpa ANSWER B Method used in converting heat directly to electricity by magnetism Electronic induction Magnetodynamic Magnetohyrdodynamic Thermoelectric ANSWER C Which of the following is NOT a material used for thermoelectric elements Bismuth telluride Lead telluride Zinc telluride Germanium ANSWER C A type of coal formed after anthracite Lignite Bituminous pear graphite ANSWER D Which of the following is lowest grade of coal? Peat Lignite Sub-bituminous Bituminous ANSWER B Which of the following helps in the ignition of coal? Moisture

Ash Fixed carbon Volatile matter ANSWER D is the ratio of fixed carbon and volatile matter Air-fuel ratio Fuel ratio Combustion ratio Carbon-volatile ratio ANSWER B A suspension of a finely divide fluid in another Filtration Floatation Emulsion Separation ANSWER C Contains 90% gasoline and 10% ethanol Gasohol Gasonol Gasothanol Gasethanol ANSWER A Process used commercially in coal liquefaction Tropsch process Fisher process Fisher-Tropsch process Mitch-Tropsch process ANSWER C Is an organic matter produced by plants in both land and water Bio-ethanol Biomass Petroleum Biodegradable ANSWER B In thermal power plant, induced draft fans are located at the Exit of furnace Foot if the stack Above the stock Top of the stock ANSWER B In thermal power plant, forced draft fans are installed at the Foot of the stack Top of the stack Exit of the preheater Inlet of the preheater ANSWER D

Known as drum less boiler La Mont boilers Fire tube boiler Forced circulation boiler Once-through boiler ANSWER D Reduces the steam temperature by spraying low temperature water from boiler drum Reheater Preheater Desuperheater Superheater ANSWER C Carbon dioxide can be removed by: Deaeration Aeration Evaporation Vaporization ANSWER B Is often used to absorb silica from water Sorbent Rectifier Silica gel Magnesium hydroxide ANSWER D Presence of excess hydrogen ions makes the water Acidic Alkalinity Base Hydroxicity ANSWER A PH of water varies with Pressure Temperature Density Volume ANSWER B Ph value of is usually maintained for boiler water to minimized corrosion 8.5 9.5 10.5 11.5 ANSWER C What type of turbine that has a degree of reaction of? Impulsive turbine Reaction turbine Rarsons turbine

Deriaz turbine ANSWER C The cooling water is made to fall in series of baffles to expose large surface area for steam fed from below to come in direct contact Spray condenser Surface condenser Jet condenser Barometric condenser ANSWER D Show the variation of river flow (discharge) with time Hydrograph Hyetograph Mass curve Flow duration curve ANSWER A Is an open channel erected on surface above the ground Canal Tunnel Penstock Flume ANSWER D Type of turbine used up to 300 m head Impulsive turbine Francis turbine Propeller turbine Deriaz turbine ANSWER D A turbine that has a diagonal flow Impulsive turbine Francis turbine Propeller turbine Deriaz turbine ANSWER D Oil is optimized either by air blast or pressure jet at about 60 bar 70 bar 80 bar 90 bar ANSWER B Type of solid injection that use single pump supplies fuel under high pressure to a fuel header Common rail injection Individual pump injection system Distributor system Single rail injection

ANSWER A Water flow in diesel engine that is caused by density differential Thermosiphon cooling Thermostat cooling Pressurized water cooling Evaporating cooling ANSWER A Type of lubrication system in diesel engine in which oil from pump is carried to a separate storage tank outside the engine cylinder and used for high capacity engine. Mist lubrication system Wet pump lubrication system Splash system Dry sump lubrication system ANSWER D Produces extreme pressure differentials and violent gas vibration Vibration Detonation Explosion Knocking ANSWER B In a spark ignition engine, the detonation occurs near the End of combustion Middle of combustion Beginning of combustion Beginning of interaction ANSWER A In a compression ignition engine, the detonation occurs near the End of combustion Middle of combustion Beginning of combustion Beginning of interation ANSWER C Morse test is use to measure the of multi-cylinder engine Brake power Indicated power Friction power Motor power ANSWER B Ignition delay can be minimized by adding to decrease engine knocking Ethel ether Ethyl chloride Ethyl nitrate Ethyl oxide ANSWER C For the submerged plane surface, the point on the surface where the resultant force acts is called the

Center of buoyancy Center of gravity Center of pressure Center of attraction ANSWER C At any point in fluid at rest, the pressure is the same in all direction. This principle is known as: Bernoulli Principle Archimedes Principle Pascal’s Law Torricelli’s Law ANSWER C The hot-wire manometer is used to measure Pressure in gases Pressure in liquids Wind velocities at airports Gas velocities ANSWER D The pitot static tube measures The static pressure The gage pressure The total pressure The dynamic pressure ANSWER A The terminal velocity of a small sphere setting in a viscous fluid varies as the First power of its diameter Inverse of fluid viscosity Inverse square of the diameter Inverse of the diameter ANSWER B Pressure drag results from Skin friction Deformation drag Breakdown of potential flow near the forward stagnation point Occurrence of wake ANSWER A The pressure coefficient is the ratio of pressure forces to Viscous forces Inertia forces Gravity forces Surface tension force ANSWER B Tranquil flow must always occur Above the normal depth Above the critical depth Below the normal depth

Below the critical depth ANSWER B Which of the following head loss coefficient among the following types of entrance? Bell mouth Square edge Reentrant It depends ANSWER C What waste water treatment method involves of algae from stabilization pond effluents? Sedimentation Floatation Filtration Microscreening ANSWER D The number of nozzles will depend on the quantity of steam required by the turbine. If nozzle occupy the entire arc of the ring, the turbine is said to have Partially full peripheral admission One-half full peripheral admission Maximum peripheral admission Full peripheral admission ANSWER D Tandem compound units may also have two low pressure castings that produces Single flow Double flow Triple flow Quadruple flow ANSWER C A type of turbine used for driving pumps, fans, and other auxiliaries in power plant commonly operate at exhaust pressure approximating atmospheric Tandem compound turbine Passout turbine Cross-compound turbine Back pressure turbine ANSWER D A governor with 0% regulation is termed as: Isochronous governor Synchronous governor Isenchronous governor Isobarnous governor ANSWER A The speed of regulation for most turbine-generators is adjustable from 2 to 6% 4 to 8% 6 to 10 % 8 to 12% ANSWER A

Poppet valves of steam turbine are used for extraction pressures of 20 to 120 psig 20 to 150 psig 20 to 130 psig 20 to 140 psig ANSWER B When both bearings of steam engine are on one side of the connecting rod, the engine is referred to as Center-crank engine Side crank engine Under crank engine Standard crank engine ANSWER B When the valve in steam engine is in mid-position of its travel, it will cover the steam port by an amount known as Steam lap Partial lap Full lap Angular lap ANSWER A A type of governor in steam engine that do not control the actual admission of steam to the cylinder but controls the pressure of the steam Flyball governor Variable cut-off governor Throttling governor Shaft governor ANSWER C By inter-cooling using two stage compressor of brayton cycle, the backward ratio will Increase Decrease Remains the same None of these ANSWER B On dynamic similitude, the relation which represents the ratio of inertia force to pressure force is Froude number Cauchy number Euler number Raynolds number ANSWER C What is the maximum velocity in a sewer flowing full? 0.6 m/sec 0.9 m/sec 1.2 m/sec 1.8 m/sec ANSWER A

A temporary structure constructed to exclude water from the site of the foundation during its excavation and construction is called: Caisson Retaining wall Coffer dam Earth dam ANSWER C Which is not a physical characteristics of water? Total suspended and dissolved solids Turbidity Color Hardness ANSWER D Which dam is best for weak foundation? Gravity Arch Buttress Earth ANSWER C What is the volume of water which will drain freely the aquifer? Specific yield Reservoir yield Safe yield Secondary yield ANSWER A What is the line defined by water level in a group of artesian walls? Water table Peizometric surface Specific yield All of the above ANSWER B Select the one that is positive indication of pollution of a river Acidity Oxygen content Chloride content Nitrite content ANSWER C Which instruments is used to measure humidity of the atmosphere continuously? Barograph Thermograph Hydrograph Thermo-hydrograph ANSWER C Entrance losses between tanks and pipe or losses through elbows, fittings and valves are generally expresses as a function of Kinetic energy

Pipe diameter Friction factor Volume flow rate ANSWER A The air that contains no water vapor is called Zero air Saturated air Dry air Humid air ANSWER C In psychometric chart, the constant enthalpy lines coincide with constant temperature lines at temperature Above 50°C Below 40°C Below 50°C Above 10°C ANSWER C The amount of moisture in air depends on its Pressure Volume Temperature Humidity ANSWER C The deep body temperature of healthy person is maintained constant at 27°C 37°C 47°C 48°C ANSWER B Air motion also plays important role in Surroundings Cooling Human comfort None of these ANSWER C During simple heating and cooling process has a humidity ratio Increasing Decreasing Constant None of these ANSWER C The follows a line of constant wet bulb temperature on the psychometric chart Evaporative cooling process Condensing process Direct cooling process None of these

ANSWER A A vapor which is not about to condense is called a Mixture of vapor and liquid Critical vapor Superheated vapor None of these ANSWER C Passing from the solid phase directly into vapor phase is called Condensation Fusion Sublimation None of these ANSWER C Robert Boyle observed during his experiments with a vacuum chamber that the pressure of gases is inversely proportional to their Temperature Pressure Volume None of these ANSWER C Is energy in transition Heat Work Power None of these ANSWER A Is the mode of energy transfer between solid surface and the adjacent liquid or gas which is in motion, and it involves combine effects of conduction and fluid motion Conduction Convection Radiation None of these ANSWER B Radiation is usually considered as Surface phenomenon Surface interaction Surface corrosion None of these ANSWER A Work is between the system and the surroundings Work interaction Energy interaction Heat interaction None of these ANSWER B Is a process during which the system remains in equilibrium at all times

Quasi-equilibrium Static equilibrium Dynamic equilibrium None of these ANSWER A In the absence of any work interactions between a system and its surroundings, the amount of net heat transfer is equal To the change in total energy of a closed system To heat and work Energy interactions None of these ANSWER A The constant volume and constant pressure specific heats are identical for Compressible substance Incompressible substance Compressible gas None of these ANSWER B The velocity of fluid is zero at the wall and maximum at the center because of the Velocity effect Viscous effect Temperature effect None of these ANSWER B For steady flow devices, the volume of the control volume is Increase Decrease Constant None of these ANSWER C The work done in a turbine is since it is done by the fluid Positive Negative Zero None of these ANSWER A Reheating process in Brayton cycle, the turbine work will Increase Decrease Remains the same None of these ANSWER A Which of the following is the chemical formula of ethanol C7H16 C2H6O C7H8

C6H12 ANSWER B Which of the following is the chemical formula of Heptane? C7H16 C2H6O C7H8 C6H12 ANSWER D Which of the following is the chemical formula of Toluene? C7H16 C2H6O C7H8 C6H12 ANSWER C As the air passes through a nozzle, which of the following will increase? Temperature Enthalpy Internal energy Mach number ANSWER D As the air passes through a diffuser, which of the following will decrease? Temperature Enthalpy Internal energy Mach number ANSWER D As the air passes through a nozzle, which of the following will decrease? Entropy Velocity Internal energy Mach number ANSWER C As the air passes through a diffuser, which of the following will increase? Density Entropy Mach number Velocity ANSWER A As the air passes through a diffuser, which of the following will NOT be affected? ANSWER B After passing through a convergent-divergent nozzle, the temperature of air will: Increase Decrease Remains the same None of these ANSWER B

After passing though a convergent-divergent nozzle, the density of air will: Increase Decrease Remains the same None of these ANSWER B After passing through a convergent-divergent nozzle, the match number of air will: Increase Decrease Remains the same None of these ANSWER B By increasing the temperature source of Carnot cycle, which of the following will not be affected? Efficiency Work Heat added Heat rejected ANSWER D By decreasing the temperature source of Carnot cycle, which of the following will not be affected? Efficiency Work Heat added Heat rejected ANSWER C By superheating the refrigerant in vapor compression cycle with useful cooling, which of the following will increase? (Use per unit mass analysis) Condenser pressure Evaporator pressure Quality after expansion Heat rejected from condenser ANSWER D By superheating the refrigerant in vapor compression cycle with useful cooling, which of the following will decrease? (Use per unit mass analysis) Refrigerant effect COP Compressor power Mass flow rate ANSWER D By superheating the refrigerant in vapor compression cycle without useful cooling, which of the following will decrease? (Use per unit mass analysis) Heat rejected COP Compressor power Specific volume at suction

ANSWER B By superheating the refrigerant in vapor compression cycle without useful cooling, which of the following will increase? (Use per unit mass analysis) Refrigerant effect COP Compressor power Mass flow rate ANSWER C By superheating the refrigerant in vapor compression cycle without useful cooling, which of the following will not be affected? (Use per unit mass analysis) Refrigerant effect COP Compressor power Mass flow rate ANSWER C By sub-cooling the refrigerant in vapor compression cycle at condenser exit, which of the following will increase? (Use per unit mass analysis) Refrigerating effect Specific volume at suction Compressor power Mass flow rate ANSWER A By sub-cooling the refrigerant in vapor compression cycle at condenser exit, which of the following will decrease (Use per unit mass analysis) A. Coefficient of performance B. Heat rejected from condenser C. Refrigerating effect D. Mass flow rate ANSWER: D By increasing the vaporizing temperature in vapor compression cycle, which the following will increase? (Use per unit mass analysis) A. mass flow rate B. COP C. specific volume D. compressor work ANSWER: B By increasing the vaporizing temperature in vapor compression cycle, which the following will decrease? (Use per unit mass analysis) A. Refrigerating effect B. COP C. evaporator temperature D. temperature difference between evaporator and compressor ANSWER: D By increasing the condenser pressure in vapor compression cycle, which of the following will increase? (Use per unit mass analysis) A. Refrigerating effect B. COP C. Specific volume at suction D. Compressor power ANSWER: D By increasing the condenser pressure in vapor compression cycle, which of the following will decrease? (Use per unit mass analysis)

A. moisture content after expansion B. compressor power C. heat rejected in the condenser D. mass flow rate ANSWER: A If the pressure drop in the condenser increases in a vapor compression cycle, which of the following will increase? (Use per unit mass analysis) A. mass flow rate B. compressor power C. heat rejected in the condenser D. specific volume at suction ANSWER: C If the pressure drop in the condenser increases in a vapor compression cycle, which of the following will decrease? (Use per unit mass analysis) A. Refrigerating effect B. mass flow rate C. heat rejected in the condenser D. Compressor power ANSWER: B If the pressure drop in the condenser increases in a vapor compression cycle, which of the following will not be affected? (Use per unit mass analysis) A. compressor power B. mass flow rate C. heat rejected in the condenser D. COP ANSWER: A If the pressure drop in the evaporator increases in a vapor compression cycle, which of the following will increase? (Use per unit mass analysis) A. Refrigerating effect B. vaporizing temperature C. heat rejected in the condenser D. COP ANSWER: C If the pressure drop in the evaporator increases in a vapor compression cycle, which of the following will decrease? (Use per unit mass analysis) A. specific volume at suction B. compressor power C. heat rejected in the condenser D. COP ANSWER: D By lowering the condenser pressure in Rankine cycle, which of the following will decrease? (Use per unit analysis) A. Pump work B. turbine work C. heat rejected D. cycle efficiency ANSWER: C By increasing the boiler pressure in Rankine cycle, which of the following will decrease? (Use per unit analysis) A. heat rejected B. pump work C. cycle efficiency D. moisture content ANSWER: A By superheating the steam to a higher temperature in Rankine cycle, which of the following will decrease? (Use per unit analysis) A. moisture content at the turbine exhaust B. turbine work C. heat added D. heat rejected ANSWER: A

By superheating the steam to a higher temperature in Rankine cycle, which of the following will increase? (Use per unit analysis) A. moisture content at the turbine exhaust B. pump work C. condenser pressure D. cycle efficiency ANSWER: D, B By reheating the steam before entering the second stage in Rankine cycle, which of the following will decrease? A. Turbine work B. moisture content after expansion C. heat added D. heat rejected ANSWER: B What Rankine cycle is modified with regeneration, which of the following will increase? A. turbine work B. heat added C. heat rejected D. cycle efficiency ANSWER: Is the combination of base load and peaking load? A. rated load B. intermediate load C. combine load D. over-all load ANSWER: B Sum of the maximum demand over the simultaneous maximum demand? A. use factor B. capacity factor C. demand factor D. diversity factor ANSWER: D Regenerative with feed heating cycle with infinite number of feed water heaters thus efficiency is equal to? A. otto cycle B. stirling cycle C. erricson cycle D. carnot cycle ANSWER: D A type of turbine used in desalination of sea water. A. back pressure turbine B. passout turbine C. peaking turbine D. reaction turbine ANSWER: A State that when conductor and magnetic field move relatively to each other, an electric voltage is induced in the conductor. A. Maxwell’s law B. Kirchhoff’s law C. Faraday’s law D. Newton’s law ANSWER: C Transfers heat directly to electrical energy by utilizing thermionic emissions. A. thermionic motor B. thermionic generator C. thermionic converter D. thermionic cell ANSWER: B Is the largest group of coal containing 46-89% of fixed carbon and 20% to 40% volatile matter. A. anthracite B. sub-anthracite C. bituminous D. subbituminous ANSWER: C When 1 gram of coal is subjected to a temperature of about 105C for a period of 1 hour, the loss in weight of the sample gives the:

Volatile matter B. ash C. fixed carbon D. moisture content ANSWER: D When 1 gram of sample of coal is placed in a crucible and heated 950C and maintain at the temperature for 7 minutes there is a loss in weight due to elimination of: A. volatile matter and moisture B. ash C. fixed carbon D. moisture content ANSWER: A Consist of hydrogen and certain hydrogen carbon compounds which can be removed from coal by heating. A. moisture content B. product of combustion C. ash D. volatile matter ANSWER: D By heating 1 gram of coal in an uncovered crucible until the coal is completely burned, the ____ will formed. A. volatile matter and moisture B. ash C. fixed carbon D. moisture content ANSWER: B Caking coal are use to produce coke by heating in a coke oven in the absence of ____ with volatile matter driven off. A. air B. oil C. oxygen D. nitrogen ANSWER: A Gindability of standard coal is: A. 80 B. 90 C. 100 D. 110 ANSWER: C Major constituent of all natural gases is: A. ethane B. methane C. propane D. cethane ANSWER: B Two types of fans are: A. centrifugal and axial B. reciprocating and axial C. centrifugal and rotary D. tangential and rotary ANSWER: A Enthalpy of substance at specified state due to chemical composition. A. Enthalpy of reaction B. enthalpy of combustion C. enthalpy of formation D. enthalpy of product ANSWER: C A type of boiler used for super critical pressure operation. A. La Mont boiler B. Once- through-circulation boiler C. Force circulation boiler D. Natural circulation boiler ANSWER: B Economizer in a water tube boiler is heated by: A. electric furnace B. electric current C. incoming flue gas D. outgoing flue gas ANSWER: D Receives heat partly by convection and partly by radiation.

A. radiant superheater B. desuperheater C. convective superheater D. pendant superheater ANSWER: D Regenerative superheater is a storage type of heat exchangers have an energy storage medium called: A. matrix B. regenerator C. boiler D. recuperator ANSWER: A Stirling cycle uses a ____ as working fluids. A. incompressible gas B. incompressible fluids C. compressible refrigerant D. compressible fluids ANSWER: D In stirling process the heat added is added during? A. isobaric process B. isentropic process C. isothermal process D. heat process ANSWER: C Brayton cycle is also known as: A. carnot cycle B. joule cycle C. diesel cycle rankine cycle ANSWER: B Is applied to propulsion of vehicles because of certain practical characteristics. A. diesel cycle B. otto cycle C. carnot cycle brayton cycle ANSWER: D

D.

D.

Heat exchangers typically involve A. no work interactions B. no heat interactions C. no energy interactions D. none of these ANSWER: A A device that is used to convert the heat to work is called A. adiabatier B. regenerator C. heat engines D. none of these ANSWER: C The objective of a heat pump is to maintain a heated space at A. Low temperature B. high temperature C. medium temperature D. none of these ANSWER: B A device that violates the second law of thermodynamics is called A. perpetual motion machine of second kind B. perpetual motion machine of third kind C. perpetual motion machine of first kind D. none of these ANSWER: A A process is called ____ if no irreversibility’s occur outside the system boundaries during the process. A. externally reversible B. internally reversible C. reversible

D. none of these ANSWER: A An energy interaction which is not accompanied by entropy transfer is A. energy B. heat C. work D. none of these ANSWER: C A ____ is used in aircraft engines and some automotive engine. In this method, a turbine driven by the exhaust gases is used to provide power to compressor or blower at the inlet. A. discharging B. turbo charging C. supercharging D. scavenging ANSWER: B The only devices where the changes in kinetic energy are significant are the A. compressor B. pumps C. nozzles and diffusers D. none of these ANSWER: C The distance between TDC and BDC in which the piston can travel is the A. Right extreme position B. displacement stroke C. stroke of the engine D. swept stroke ANSWER: C In compression-engine, the combustion of air-fuel mixture is self-fuel ignited as a result of compressing the mixture above its A. self developed temperature B. mixing temperature C. self feed temperature D. self ignition temperature ANSWER: D The thermal efficiency of an ideal Otto cycle depends ____ of the working fluid. the pressure ratio of the engine and the specific heat ratio the temperature ratio of the engine and the specific heat ratio the moles ratio of the engine and the specific heat ratio the compression ratio of the engine and the specific heat ratio ANSWER: D As the number of stages is increased, the expansion process becomes A. isentropic B. isothermal C. isometric D. polytropic ANSWER: B Aircraft gas turbines operate at higher pressure ratio typically between A. 6 to 8 B 12 to 24 C. 10 to 18 D. 10 to 25 ANSWER: D The first commercial high-pass ratio engines has a bypass ratio of A. 1 B. 3 C. 5 D. 7 ANSWER: C The single-stage expansion process of an ideal Brayton cycle without regeneration is replaced by a multistage expansion process with reheating the samepressure limits. As a result of modification, thermal efficiency will: A. increase B. decrease C. remain constant D. none of these

ANSWER: B Which of the following is/are the application of Brayton cycle. A. propulsion system B. automotive turbine engine C. aircraft turbine engine D. all of these ANSWER: D It is used as working fluid in high-temperature applications of vapor cycles. A. helium B. deuterium C. mercury D. water ANSWER: C The superheated vapor enters the turbine and expands isentropically and produces work by the rotating shaft. The ____ may drop during the process. A. density B. viscosity of fuel C. temperature and pressure D. none of these ANSWER: C Only ____ of the turbine work output is required to operate the pump. A. 0.01% B. 0.02% C. 0.03% D. 0.04% ANSWER: D Superheating the steam to higher temperatures decreases the moisture content of the steam at the ____. A. turbine inlet B. compressor inlet C. compressor exit D. turbine exit ANSWER: D Regeneration also provides a convenient of dearating the feedwater to prevent? A. boiler explosion B. boiler scale production C. boiler corrosion D. compressor damage ANSWER: C Can be apply steam turbine cycle(rankine). Gas turbine cycle(brayton) and combined cycle. A. hydroelectric plant B. nuclear power plant C. cogeneration plant D. tidal power plant ANSWER: C In a rankine cycle with fixed turbine inlet conditions. What is the effect of lowering the condenser pressure, the heat rejected will: A. increase B. decrease C. remains the same D. none of these ANSWER: B In an ideal rankine cycle with fixed boiler and condenser pressure. What id the effect of superheating the steam to a higher temperature, the pump work input will. A. increase B. decrease C. remain the same D. none of these ANSWER: C The fact that total energy in any one energy system remains constant is called the principle of ____? A. conversion of energy B. second law of thermodynamics C. conservation of mass D. zeroth law of thermodynamics ANSWER: A A process for which the inlet and outlet enthalphies are the same

A. isntropic ANSWER: C

B. enthalphy conservation C. throttling D. steady state

The sum of energies of all the molecules in system, energies that in several complex forms. A. kinetic energy B. internal energy C. external energy D. flow work ANSWER: B A system that is completely impervious to its surrounding. Neither mass nor energy cross its boundaries. A. Open system B. closed system C. adiabatic system D. isolated system ANSWER: D A device used to measure small and moderate pressure difference. A. manometer B. bourdon gage C. barometer D. piezometer ANSWER: A A vapor having a temperature higher that the saturation temperature corresponding to its pressure. A. superheated pressure B. saturated vapor C. super saturated vapor D. subcooled vapor ANSWER: A The energy or stored capacity for performing work possessed by a moving body, by virtue of its momentum. A. internal energy B. work C. gravitational potential energy D. kinetic energy ANSWER: D A thermodynamic process wherein temperature is constant and the change in internal energy is zero. A. isobaric process B. isometric process C. isothermal process D. polytropic process The science terminology concerned with precisely measuring energy and enthalpy A. thermodynamics B. chemistry C. calorimetry D. none of these ANSWER: C The rate of doing work per unit time A. torque B. power C. force D. moment ANSWER: B A vapor having a temperature higher than the saturation temperature corresponding to the existing pressure. A. superheated vapor B. saturated vapor C. wet vapor D. none of the above ANSWER: A It is the work done in pushing a fluid across a boundary, usually or out of a system. A. mechanical work B. nonflow work C. flow work D. electrical work ANSWER: C

A liquid that has a temperature lower that the saturation temperature corresponding to the existing pressure. A. subcooled liquid B. saturated liquid C. unsaturated liquid D. water ANSWER: A In this type of boiler, the water passes through the tubes while the flue gases burn outside the tubes. A. water-tube boiler B. fire-tube boiler C. steam generator D. electric boiler ANSWER: A It shows the water level in the boiler drum. A. water column B. try cocks C. gauge glass D. all of the above ANSWER: C It prevents damage to the boiler by giving warning of low water. A. safety valve B. fusible plug C. relief valve D. try cocks ANSWER: B it is heat exchanger which utilizes the heat of the flue gases to preheat air needed for combustion. A. economizer B. feedwater heater C. reheater D. air preheater ANSWER: D It is the subject that deals with the behavior of moist air. A. psychrometer B. psychometry C. refrigeration D. pneumatics ANSWER: B Air whose condition is such that any decreases in temperature will result in condensation of water vapor into liquid. A. saturated air B. unsaturated air C. saturated vapor D. moist air ANSWER: A It is the warm water temperature minus the cold water temperature leaving the cooling tower A. approach B. terminal difference C. cooling range D. LMTD ANSWER: C The surrounding air ____ temperature is the lowest temperature to which water could possibly cooled in a cooling tower. A. dry-bulb B. wet-bulb C. dew-point D. saturated temperature ANSWER: B PRE ELEMENTS Which of the following compressors are so widely used for today’s refrigeration system? Ans. Centrifugal A valve sometimes known as the magnetic valve. Ans. Solenoid valve Which of the following valve use to regulate the flow of refrigerant to the evaporator? Ans. Expansion valve Which of the following use control the flow of refrigerant gas from the evaporator coil. This type of control valve is also known as back pressure regulator of an evaporator.

Ans. Suction line regulator Which of the following refrigerant control is used to limit the flow of gas to the compressor to prevent surge of excessive load from overloading the compressor. Ans. Hold-back valve Which of the following refrigerant control is used in the liquid suction or discharge to interrupt the flow on demand from any one of the several types of temperature or pressure sensing devices. Ans. Solenoid valve What is the pressure drop from the receiver to the expansion valve? Ans. 5 psi If any of the electricity controlled device in a Freon system malfunction, which following valves will also automatically shut off? Ans. Solenoid valve The relief valve on a CO2 machine is located: Ans. On the discharge pipe between the compressor and the discharge valve. When checking zinc plates in a condenser, one should: Ans. Clean the plate and renew worn-out ones. A refrigerant should have a Ans. High latent heat The mechanical energy of a device is the ratio of: Ans. Mechanical energy input to the mechanical energy output When the winding or circuit is open, what is the resistance reading? Ans. Infinity If the meter scale of the ohmmeter reads 0 ohms and the range adjustment is R 10, what is the resistance reading? Ans. 100 What is the resistance reading of an electric motor in good condition? Ans. With resistance The basic unit of electrical pressure is: Ans. Volt What is the most commonly used conductor? Ans. Copper Which of the following is the effect of superheating the refrigerant? Ans. Increase is COP Which of the following statement is true in ice making capacity? Ans. Ice making capacity is always proportional to the refrigerating effect Which of the following has the largest heat load in cold storage room? Ans. Product heat load Which of the following is a halocarbon refrigerant? Ans. Methyl chloride A refrigerant system in which pressure- imposing element is mechanically operated? Ans. Compressor Which of the following compressors have the compressing element and drive seals in a single housing? Ans. hermitically sealed compressor

Which of the following consists of two mating helically grooved male and female grooves? Ans. Helical rotary-screw What is the another name of helical-rotary screw compressors? Ans. Lysholm type At what head is the helically-rotary compressor are designed to operate? Ans. High head pressure The bulk quantity of oil separated from the refrigerant in the helically-rotary compressor falls by? Ans. Gravity to a sump If the temperature in the icebox is too high, the trouble could be: Ans. Automatic control not functioning properly If any of the electrically controlled devices in a Freon system malfunction, which of the following valves also automatically shut-off? Ans. Solenoid valve Natural ice in an ice box was used in the early Ans. 19th century The amount of matter present in a quantity of any substance is called: Ans. Mass What pressure is indicated by a barometer? Ans. Atmospheric pressure The basic unit measurement for heat is: Ans. BTU The force that acts upon a unit of area is termed: Ans. Pressure What is the physical state of a refrigerant entering the flow control? Ans. Sub-cooled What method of heat transfer does not depend on molecular motion? Ans. Radiation Where s the filter drier is located? Ans. After the condenser A motor drive device which removes the heat-laden vapor refrigerant from the evaporator is the: Ans. Compressor The function of the compressor is to: Ans. Squeeze out refrigerant from the evaporator In which part of the compressor is the piston attached to the crankshaft? Ans. Connecting rod This is a compressor which has an off center-rotor. Ans. Rotary Which of the following compressor is commonly used in household refrigerators? Ans. Hermetic What is the length of the tubing extended from the part to be swaged Ans. ½ inch The best method of making leak proof connection is: Ans. Silver brazing

The refrigerant that should not be used copper or brass tubing: Ans. R-717 The color of the hose connected to suction line or process be Ans. Blue Where is the compound gauge installed in a refrigerant system? Ans. Suction line A gauge used to measure pressure both below and above atmospheric pressure Ans. Compound gauge What is the valve position if the valve stem is turned all the way in? Ans. Front-seated What is the valve position if the valve stem is turned all the way out? Ans. Back-seated Ohm’s law states that resistance is inversely proportional to: Ans. Current To prevent corrosion within the flue gas, what must be prevented from condensing? Ans. Vapor Ans. Evaporative cooling # An adiabatic saturation process. This process can be produced with _____, which is essentially a regular thermometer with its bulb wrapped in wet cotton or gauze..... Ans. Sling psychrometer # A high velocity gas is defined as a gas moving with a velocity in excess off approximately Ans. 300ft/s/ 100m/s # The theoretical maximum velocity is achieved when..... Ans. All internal and pressure energies are converted to kinetic energies # If the gas flow is adiabatic and frictionless (i.e. reversible) Ans. Both A and B # A property by which the refrigerant remains at its original chemical form/original condition is imposed by the operation..... Ans. Stability of refrigerant # The maximum temperature at which a gas is condense into liquid. Above temperature, a vapor imposed by the operation..... Ans. Stability of refrigerant # Which of the following is the name given for halogenated hydrocarbon? Ans. Halocarbon # Which of the following refrigerants are produced by carrier corporation? Ans. Carrene # Which of the following refrigerants is produced by E.L.DU FONT? Ans. Freon #Which of the following is the other name of R 718? Ans. Water # Energy per unit volume stored in a deformed material is called: Ans. Strain energy # The ratio of ultimate failure strain to yielding strain is known as:

Ductility # One ton of refrigeration is equal to: Ans. 3.5KW # With what cycle does air refrigeration works? Ans. Bell-coleman cycle # What cycle does air refrigeration works? Ans. Coal # What is produced by fission process? Ans. Radiation # Loss of power is due to: Ans. Low injection pressure # A branch system of pipes to carry waste emissions away from the combustion chamber. Ans. Exhaust manifold # The type of filter where the filtering elements is replaceable. Ans. Metal-edge filter # When four events takes place in one revolution of a crankshaft of an engine, the engine is called Ans. Two stroke engine # Which of the following does not belong to the group. Ans. Time injection system # The function of a super heater is to: Ans. Exchange heat to increase energy to the flow during an adiabatic, internally reversible process # What is true about change in entropy? Ans. it is always zero # How does an adiabatic process compare to an isentropic process? Ans. Both heat transfer = Q, isentropic reversible # Adiabatic heat transfer within a vapour cycle refers to : Ans. The transfer of energy from one stream to another in a heat exchanger that the energy of the input streams equal to the energy of output. # All of the following process are irreversible except: Ans. An isentropic compression of a perfect fluid # Refers to the system where the evaporator coils are placed in the ice box Ans. Direct system # What is the usual dehydrating agent in Freon system? Ans. Activated alumina # Refers to the system where the evaporator coils are indicated in a brine solution and brine is pumped thru the ice box Ans. Indirect system # Is the most common dryer which consist of rotating cylinder inside which the materials flow while getting in contact with hot gas. Ans. Rotary dryer # Is the ratio of the mass of water-vapor in air and mass of air if it is saturated is called:

Ans. Relative humidity # The hands feel painfully cold when the skin temperature reaches Ans. 10°C # The total heat of air is a function of Ans. WB temperature # Boiling point of Freon-12 at atmospheric pressure is: Ans. 21°F # Which of the following is NOT a type of water cooled condenser in refrigeration? Ans. Double shell # Component of absorption refrigeration system in which the solution is cooled by cooling water. Ans. Absorber # Cascade refrigeration cycle is often used in industrial process where objects must be cooled to temperature below: Ans. -46°C # Type of refrigerant control designed to maintain a pressure difference while the compressor is operating. Ans. Capilliary tube # As a rule of thumb, for a specified amount of compressed air, the power consumption of the compressor decreased by ____________for each 3°C drop in the temperature inlet air to the compressor. Ans. 1 percent # Modern way of detecting air compressor leak is by using Ans. Acoustic leak detector # For foundation of stacks, the maximum pressure on the soil is equal to the pressure due to the weight and the _______. Ans. Wind movement # Foundation bolts of specified size should be used and surrounded by a pipe sleeve with an inside diameter of at least Ans. 3 times the diameter of anchor bolt # For multi stage compression of an ideal Brayton cycle, the back work ratio will Ans. decrease # Type of turbine that has a specific speed below 5. Ans. Impulse turbine # A high discharge type of turbine Ans. Propeller turbine # Use to minimize the speed rise due to a sudden load rejection Ans. jet deflector # Is the speed of a turbine when the head on the turbine is one meter. Ans. Unit speed # Is a fluid property which refers to the intermolecular attraction by which the separate particles of the fluid arc held together. Ans. Cohesion # Which of the following is NOT the cause of black smoke in diesel engine?

Ans. high compression pressure # Which of the following is not a method of starting a diesel engine? Ans. Using another generator # Two-stroke engine performs _____ to complete one cycle. Ans. compression and power stroke # A type of geothermal plant used when there is a presence of brine extracted from undergroungd Ans. Binary geothermal plant # Is the most important safety device on the power boiler. Ans. Safety valve # During hydrostatic test, the safety valves should be Ans. removed # Where deaerating heaters are not employed, it is recommended that the temperature of the feed water be not less than _______ Ans. 197°C # What is the possible cause of too much suction pressure? Ans. Shortage refrigerant Where is the oil separator located? Ans. Between the compressor and condenser # What is the purpose of the low pressure cutout switch? Ans. To cut compressor in and out at present pressure # Freon unit will tend to short cycle when operating under Ans. Light loads # What cause hot suction line? Ans. Insufficient refrigerant # When changing a Freon system which valve is not used? Ans. King valve # Where is the scale trap located? Ans. Between the king valve and expansion valve # Where is the solenoid valve located? Ans. Between the scale trap and thermal expansion valve # Water tube boilers have how many fusible plugs? Ans. 4 # What is the main cause of air pollution as a result of burning fuel? Ans. Nitrogen dioxide # Refers to the increase of enthalpy of a substance when it undergoes some phase change at constant pressure and temperature. Ans. Heat of vaporization # Which of the following keeps moisture from passing thru the system? Ans. Dehydrator # What is the lowest temperature to which water could possibly be cooled in a cooling tower? Ans. Temperature of adiabatic saturation #What do you call the intake pipe to a hydraulic turbine from a dam? Ans. Penstock # An ideal fluid is one that

Ans. is frictionless and incompressible # What principal was used by McLeod gauge which is used for low pressure measurement/ Ans. Boyles Law #The difference between the brake and hydraulic power is Ans. Friction power # Product of motor pump efficiency is known as Ans. Over-all efficiency # The most efficient cross-section of an open channel is the one that Ans. All of the above #Refers to the structure across an open channel over which water flows. Ans. Weir # When can we say that a trapezoidal weir is a “cipoletti” weir? Ans. When side slope 4 vertical and 1 horizontal # A weir is broad-crested if: Ans. The weir length is greater than half of the head # A feature of a dam over where water is discharged is called: Ans. Spillway # Laminar flow occurs when the Reynolds number is approximately less than Ans. 2100 # Turbulent flow occurs when the Reynolds number exceeds: Ans. 4000 # Laminar flow is typical when Ans. All of the above # When the Reynolds number is between 2100 to 4000, the flow is said to be in Ans. Critical zone # Which of the following characterizes a laminar flow? Ans. All of the above # Without a pump and turbine, which of the following rules is true in a frictionless environment about energy and hydraulic gradient? Ans. All of the above # What is the latent heat of vaporization of water? Ans. 970 Btu # A dry bulb temperature at which water starts to condense but when moist appears in a constant pressure process Ans. Dew point temperature # A mixture of dry air and saturated vapour is known as: Ans. Saturated air # What is another term for “humidity ratio”? Ans. Specific humidity # The water vapour mass is often reported in Ans. Grains of water # Approximately how many grains of water are there in 1 pound? Ans. 7000 # The ratio of actual humidity ratio to the saturated humidity ratio at the inlet temperature and p

Ressure is known as Ans. Degree of saturation # What is another term for “degree of saturation”? Ans. Percentage of humidity # The partial pressure of the water vapour divided by the saturation pressure? Ans. Relative humidity # The point where sonic velocity has been achieved (M=1) is known as: Ans. Critical point # The ratio of the thrust in equal mass flow rate is known as... Ans. Effective exhaust velocity # Is the velocity of gas dropped from supersonic to subsonic, gas will experience Ans. Shock wave. # An adiabatic flow with friction, which would be approximately as flow through an insulated duct... Ans. Fan no flow # What is an inevitable by product of the combustion of hydro carbon fuel? Ans. Water vapour # A compound pressure gauge is used to measure: Ans. Positive and negative pressure # Which of the following best describes a Curtis turbine? Ans. It is a velocity pressure compound turbine # What causes low head pressure? Ans. Too much cooling water and in sufficient refrigeration gas # What causes high head pressure? Ans. All of the above # A commercial pipe which has the characteristics of being immune to electrolysis and corrosion, light in weight weak structurally. Ans. Asbestos cement pipe # A commercial pipe which is durable, water tight, low maintenance and smooth interior. Ans. Concrete pipe # A commercial pipe which has the characteristics of being corrosion, and scour and erosion Resistance. Ans. Vitrified clay pipe # A commercial pipe which that is ductile, high strength, shock resistant, very smooth internally. Ans. Steel pipe a # A commercial pipe that is chemically insert, resistant to corrosion, very smooth, light weigth and very cheap Ans. Plastic (PVC and ABS) # Which commercial pipe that is primarily used for water condensate, and refrigerant lines easier to bent by hand, good thermal conductivity Ans. Brass and copper pipe

# A fluid pathway that expose parts of the fluid to the atmosphere is the: Ans. Open channel # The ratio of the area in flow to the width of the channel at fluid surface in an open channel is known as: Ans. Hydraulic depth # The most efficient cross-section for an open channel is known as: Ans. Semi-circular section # The most efficient section rectangular section has: Ans. A width twice the depth # What is the maximum power that the motor can provide Ans. Rated horsepower # Which of the following is a possible cause of cavitation Ans. All of the above # Is a reaction during which chemical energy is released in the form of heat # When forebay is not part of the generating plant design, it will be desirable to produce a ________ in order to relieve the effect of rapid changes in flow rate. Ans. Surge chamber #What is used to keep the turbine in a hydroelectric generating plant up to 15 feet above the tail water surface? Ans. Draft tube # If a draft tube is not employed, water may be return to the tail water by way of a chamber known as the: Ans. Tail race # An impulse turbine consists of a rotating shaft on which buckets of blades are mounted. What is the term used for rotating shaft? Ans. Turbine runner # Which of the following refrigerant that belong to group B refrigerants and harmful or toxic refrigerants? Ans. All of the above # Which of the following refrigerants are combustible? Ans. All of the above # Which of the following refrigerants is used for frozen foods and ice cream display cases Warehouses and food freezing plants, medium temperature display cases truck refrigeration and heat pumps? Ans. R-502 # The refrigerant that was formerly the most widely used for air conditioning and refrigeration. It was the principal refrigerant for automotive air conditioning. Ans. R-12 # How do we know that there is shortage of refrigerant considering that we are in the liquid line? Ans. Vapor bubbles in the sight glass appear # The amount of power actually entering a fluid is known as:

Ans. All of the above #The input power to the pump will be: Ans. Equal to the output power of the electric motor driving the pump # A bent or curved carries fluid from a container at a high elevation to another container at a lower elevation. Ans. Siphon # A water path, usually a large diameter pipe used to channel water around or through an obstruction feature. Ans. Culvert # Which of the following principles governs the distributions of flow between the two branches in a pipe system. Ans. All of the above # A method used to determine the network flow in a multi-hoop piping system. Ans. Hardy-cross method # A device used to measure velocity by determining the cooling effect of fluid flowing over an electrically heated tungsten. Ans. Hot-wire anemometer # Which of the following is used for R-12? Ans. All of the above # Which of the following tubing (pipes) usually used in ammonia refrigerant? Ans. Ferrous pipe # Which of the following tubing (pipes) usually used in halo-carbon refrigerant? Ans. K and L copper tubing # Properties of superheated vapour can be found in? Ans. Superheated table # Properties of non-reacting gas mixture are given by: Ans. Volumetric weight for molecular weight and density and geometric weighing for all the properties except entropy. # The relation between the total volume of a mixture of a non-reacting gases for initial volume is given by: Ans. Amagat’s law # Which of the following is the first definition of enthalpy? Ans. The amount of useful energy in a system. # A consists press thermodynamics process obeys: Ans. Charles law # The volume of an ideal is halved, while it’s temperature is double, what happens to the pressure? Ans. Pressure is multiplied by 4 # A liquid boils when its pressure equal. Ans. Ambient pressure # A system composed of ice and water at zero °C is said to be: Ans. All of the above # A heat of fusion for a pure substance is ________. Ans. The energy required to melt the substance. #The heat of vaporization involves the change of enthalpy due to:

Ans. The change in phase from liquid to gas. # The heat of sublimation involves the change of enthalpy due to: Ans. The change in phase from solid to gas. # The unit for absolute viscosity which is dyne-sec/cm3 is also known as: Ans. Poise # What is the unit of absolute viscosity? Ans. Pa-second # The ratio of absolute viscosity to mass density is also known as: Ans. Kinetic viscosity # What is the kinetic viscosity? Ans. m2/s # Stroke is a unit of kinetic viscosity which is equivalent to: Ans. cm2/s # What are volatile liquids? Ans. The liquid that vaporizes immediately # When does boiling occur? Ans. When the liquid temperature is increased to the point that the vapour present is equal to the local ambient pressure. # The liquid boiling pressure is dependent on: Ans. Both A and B # Referred as the mass flow the rate divided by the generator output in kilowatts Ans. Water rate of stream rate # The power after the auxiliary loads has been removed is known as: Ans. Net electrical output # _________ of a turbine is the ratio of actual to ideal energy extraction? Ans. Both A and B # What is the approximate maximum practical metallurgical limit on superheat? Ans. 1150°C and 625°C # What is used to increased the mean effective temperature at which heat is added without producing significant expansion in the liquid-vapor region. Ans. Superheat # If some of the heat energy from these waste product is recovered and used for space heating or cooling the process is called Ans. Cogeneration # In cogeneration, the recovered heat Ans. Is used as heat # The ratio of useful energy to the energy input is called? Ans. Fuel utilization # The ratio of the energy of a turbine to the recovered heat is known as: Ans. Power to heat ratio # If the recovered heat is used to vaporized water in a vapour cycle this is called: Ans. Combined cycle # What is the best coolant for high temperature gas reactor? Ans. Helium (He) # What is the liquid metal most frequently used as coolant, in liquid metal reactor?

Ans. Sodium (Na) # Combustion power cycles differ from vapour power in that: Ans. The combustion product cannot be returned into their initial condition for reuse. # A closed system using a fixed amount of ideal air as the working fluid? Ans. Air standard cycle # Which of the following is not a specific compound but is a mixture of octane and lighter hydrocarbon? Ans. Gasoline # A series of process that eventually brings the system back to its original condition is known as? Ans. Cycle # The _____ of a power cycle is defined as the ratio of usual work output to the supplied input energy? Ans. Thermal efficiency # Is use to heat up the solution partially before entering the generator in absorption refrigeration system. Ans. regenerator # The COP of actual absorption refrigeration system is usually Ans. less than 1 # Sight glass is often located at: Ans. liquid line # Use to detects a vibration in current caused by the ionization of decomposed refrigerant between two opposite-charged platinum electrodes. Ans. Electronic detector # The ability of oil to mix with refrigerants Ans. miscibility # For the submerged plane surface, the point on the surface where the resultant force acts is called the Ans. center of pressure # At any point in fluid at rest, the pressure is the same in all directions. This principle is known as: Ans. Pascal’s Law # The hot-wire manometer is used to measure Ans. gas velocities # The pitot static tube measures Ans. the static pressure # The terminal velocity of a small sphere setting in a viscous fluid varies as the Ans. inverse of fluid viscosity # Pressure drag results from Ans. skin friction # The pressure coefficient is the ratio of pressure forces to: Ans. inertia forces # Tranquil flow must always occur Ans. above the critical depth # Which of the following head loss coefficient among the following types of entrance?

Ans. re-entrant # What waste water treatment method involves of algae from stabilization pond effluents? Ans. microscreening # The number of nozzles will depend on the quantity of steam required by the turbine. If the nozzles occupy the entire arc of the ring, the turbine is said to have: Ans. full peripheral admission # Tandem compound units may also have two low-pressure casting that produces: Ans. triple flow # By inter-cooling using two stage compressor of Brayton cycle, the backwork ratio will: Ans. decrease # On dynamic similitude, the relation which represents the ratio of inertia force to pressure force is: Ans. Euler number # What is the maximum velocity in a sewer flowing full? Ans. 0.6 m/sec # A temporary structures constructed to exclude water from the site of the foundation during its excavation and construction is called: Ans. coffer dam # Which is not a physical characteristics of water? Ans. hardness # Which dam is best for weak foundation? Ans. buttress # What is the volume of water which will drain freely from the aquifer? Ans. specific yield # What is the line defined by the water level in a group of artesian wells? Ans. peizometric surface # Select the one that is a positive indication of pollution of a river. Ans. chloride content # Which instruments is used to measure humidity of the atmosphere continuously? Ans. hydrograph # Entrance losses between tank and pipe or losses through elbows, fittings and valves are generally expresses as a function of: Ans. kinetic energy # The air that contains no water vapour is called: Ans. dry air

# What effectively states that it is impossible to built a cylindrical engine that will have a thermal efficiency of 100%? Ans. Kelvin-Planc statement of second law # Refers to the maximum possible work that can be obtained from a cycle. Ans. Availability # The difference between the maximum and the actual work output is known as? Ans. Process inversibility # The study of the property of atmospheric air?

Ans. Psychrometry # The temperature of the air that has gone through an adiabatic saturation process is known as? Ans. Wet-bulb temperature # How often should compressor oil be changed? Ans. At least semi-annually # Which of the following is the possible effect of the weak solution of brine in a refrigeration system? Ans. Corrosion # In Freon refrigeration system, where is the scale trap located? Ans. On the suction side of the compressor # At what pressure is the high pressure cut-off in the Freon (R-12) Ans. 125-150 psi # What is the other name for brine refrigeration system? Ans. Indirect refrigeration system # What is the indication that there is air in the refrigeration system? Ans. Unusual high head pressure # A Freon-12 leaking can be detected by halide torch. What color will it turn to in the presence of a refrigerant if this torch has a normal blue flame? Ans. Green # All of the following process are irreversible except Ans. Inelastic tension and release of steel bar # Which of the following is a point function except Ans. Work # All heat transfer process require medium of energy exchange except Ans. Radiation # Thermal conduction is described by: Ans. Fourier Law # Convection is described by: Ans. Newton’s Law # Radiant heat transfer is described by: Ans. The Stefan-Boltzman Law # The equivalence of ratios of emissive power to absorptivity equilibrium is described by Ans. Kirchoff’s Law # The temperature potential between temperature difference. Ans. The logarithmic temperature difference # A thermodynamic process whose deviation from equilibrium is infinitesimal at all times. Ans. In quasi-equilibrium # A thermodynamic property best describes the molecular activity of a substance? Ans. Internal energy # The combination of conditions that best describes thermodynamic process is given: Ans. Has successive states thru which the system passes # A substance whose properties are uniform throughout is called a: Ans. Pure substance # All of the following are thermodynamic properties except:

Ans. Modulus # A process that is adiabatic and reversible is also called: Ans. Isentropic # The first law of thermodynamics for a closed system is Q= ∆U + W, the sign convention is: Ans. Q positive in, W positive out and ∆U negative for decreased internal energy # The heat transfer term in the first law of thermodynamics may be due to any of the ff. except: Ans. Internal heat generation # A system that experiences no mass crossing the system boundaries called: Ans. Closed system # A substance in which a substance is allowed to enter and have is most properly called: Ans. Open system # First and second law of thermodynamics are: Ans. Energy equations # A constant temperature thermodynamic process obeys: Ans. Boyles Law # A series of process that eventually bring the system back to the original condition is called a: Ans. Cycle # If the refrigerant leaves the evaporator with a quality of < 1, the cycle is known as: Ans. Wet vapor compression cycle # For even lower temperature, are required as in commercial freezing equip. What is used as refrigerant? Ans. Ammonia # What is another name for Air-Refrigeration cycle? Ans. Brayton cycle # What is the major disadvantage of Air-Refrigeration cycle? Ans. High power consumption # What refrigeration is practically used when a large quantity of waste-inexpensive heat is avail? Ans. Heat-driven refrigeration cycle # For an absorption cycle, how many working fluids are required? Ans. Two (2) # In absorption cycle, what is needed in NH3 water system to remove any remaining traces of absorbents from the refrigerant and is placed between generator and condenser. Ans. Rectifier # Which is a “TEMA” stands for? Ans. All of the above # The acronym “ TEMA” stands for? Ans. Tubular Exchangers Manufacturing Association # The acronym “API” stands for? Ans. American Petroleum Institute

# What occurs when the exit temperature of the cold fluid is above the exit temperature of the hot fluid? Ans. Temperature cross # Are heat exchangers whose purpose is to heat with condensing stream. Ans. Closed feedwater heater # Refers to the corrosion, precipitation of compound in solution, setting of particular solid and biological activity that adhere to a heat transfer Ans. Fouling # Fouling in a heat exchanger industries is commonly known as: Ans. Silent thief # An operation with 1 fluid flow rate substantially known as: Ans. Down turn # Indicate the false statement? Ans. Duo to viscosity, liquid cannot resist instantaneously change velocity # Ideal liquid are assumed to be? Ans. Newtonian fluids # Which of the following is a Newtonian fluids? Ans. All of the above # Thermal radiation heating body can be absorb, reflected or transmitted, this is known as Ans. Radiation Conservation law # The rate of thermal emitted per unit area of a body is known as: Ans. Emissive power # States that for any two bodies in thermal equilibrium, the ratio of emissive power to absorptivity are Equal. Ans. Kirchoff’s Radiation law # Bodies that radiate at absorptivity of 1 are known as: Ans. Black bodies or ideal radiators # Which of the following does not radiate at the ideal level? Ans. Real body # The ratio of actual to ideal emissive power is known as: Ans. Emissivity # What is the refrigerator’s main function? Ans. To cool this low temperature area # The rate of energy removal from the low temperature area is known as: Ans. Either of the refrigeration capacity or refrigerating effect # The refrigeration capacity is measured in refrigeration ton where one ton is _____ heat remove Ans. All of the above # The refrigeration ton is derived from the heat flow required to melt the ton of ICE in: Ans. 24 hours # In psychrometric chart, the constant- enthalpy lines coincide with constanttemperature lines at temperature Ans. below 50°C # The amount of moisture in air depends on its

Ans. temperature # The deep body temperature of healthy person is maintained constant at Ans. 37°C # Air motion also plays important role in Ans. human comfort # During simple heating and cooling process has a_____ humidity ratio. Ans. constant # The ________ follows a line of constant wet-bulb temperature on the psychrometric chart. Ans. evaporative cooling process # A vapor which is not about to condense is called a Ans. superheated vapor # Passing from the solid phase directly into vapor phase is called Ans. sublimation # Robert Boyle observed during his experiments with a vacuum chamber that the pressure of gases is inversely proportional to their Ans. volume # _____ is energy in transition. Ans. Heat # Is the mode of energy transfer between a solid surface and the adjacent liquid or gas which is in motion, and it involves combine effects of conduction and fluid motion. Ans. convection # Radiation is usually considered as Ans. surface phenomenon # Work is _____ between the system and the surroundings. Ans. energy # Is a process during which the system remains in equilibrium at all times Ans. Quasi-equilibrium # In the absence of any work interactions between a system and its surrounding, the amount of net heat transfer is equal Ans. to the change in the total energy of a closed system # The constant volume and constant pressure specific heats are identical for Ans. incompressible substance # The velocity of fluid is zero at the wall and maximum at the center because of the Ans. viscous effect # For steady flow devices, the volume of the control volume is Ans. constant # The work done in a turbine is _____ since it is done by the fluid. Ans. positive # Reheating process in Brayton cycle, the turbine work will Ans. increase # As the air passes through a nozzle, which of the following will increase? Ans. mach number # As the air passes through a diffuser, which of the following will decrease? Ans. mach number # As the air passes through a nozzle, which of the following will decrease?

Ans. internal energy # As the air passes through a diffuser, which of the following will increase? Ans. density # As the air passes through a diffuser, which of the following will NOT be affected? Ans. entropy # After passing through a convergent-divergent nozzle, the temperature of air will: Ans. decrease # After passing through a convergent-divergent nozzle, the density of air will: Ans. decrease # After passing through a convergent-divergent nozzle, the mach number of air will: Ans. decrease # By increasing the temperature source of Carnot cycle, which of the following will not be affected? Ans. heat rejected # By decreasing the temperature sink of Carnot cycle, which of the following will not be affected? Ans. heat added Ans. Exothermic reaction # By reheating the steam in an ideal Rankine cycle the heat rejected will: Ans. increase # By increasing the boiler pressure in Rankine cycle the moisture content at boiler exit will: Ans. increase # Presently the highest steam temperature allowed at the turbine inlet is about ______. Ans. 620°C # Two most common gases employed in Stirling and Ericson cycles are: Ans. Hydrogen and helium # In most common designs of Gas turbine, the pressure ratio ranges from: Ans. 11 to 16 # In Brayton cycle, the heat is transformed during what process? Ans. isobaric process # The fuel injection process in diesel engine starts when the piston_______. Ans. approaches TDC # If the cut-off ratio of diesel cycle increases, the cycle efficiency will: Ans. decrease # The fuel used in a power plant that is used during peak periods. Ans. liquid # Typical compression ratio of Otto cycle is Ans. 8 # If Joule Thompson coefficient is equal to zero, then the process will become Ans. isothermal # If the fluid passed through a nozzle its entropy will: Ans. remains the same # Which of the following is NOT a possible location of service valve? Ans. outlet of condenser

# A coil in series with evaporator that is use to prevent the liquid refrigerant entering the compressor. Ans. Drier loop # A type of valve connected from discharge of compressor directly to suction that is normally closed And will open automatically only if there is high discharge pressure. Ans. solenoid valve # Use to increase the capacity of condenser. Ans. desuperheating coils # Is use to subcooled the refrigerant from the condenser. Ans. liquid subcooler # Which of the following is NOT a part of low pressure side in refrigeration system? Ans. liquid line # Which of the following is NOT a part of high pressure side in refrigeration system? Ans. suction line # Which of the following is NOT a part of condensing unit? Ans. liquid line # By subcooling the refrigerant in refrigeration system, the compressor power per unit mass will Ans. remains the same # Superheating the refrigerant in refrigeration system without useful cooling, the refrigeration effect Per unit mass will Ans. remains the same # By subcooling the refrigerant in refrigeration system, the specific volume at compressor suction will Ans. remains the same # Pressure loss due to friction at the condenser, the compressor power per unit mass will Ans. remains the same # Which of the following is NOT a type of air-cooled condenser? Ans. shell and tube # A type of refrigerant control typically used in household refrigeration. Ans. Capilliary tube # Type of condenser that operates like a cooling tower. Ans. evaporative condenser # The major problem of heat pump is Ans. frosting # Dominant refrigerant used in commercial refrigeration system # The Carnot refrigeration cycle is: Ans. All of the above # The vapour compression cycle is essentially a reverse of: Ans. Rankine vapor cycle # What can be considered to be theoretically exact and can be used to determine local thermal film coefficient at a distance X from the leading edge of an isothermal plate in laminar flow?

Ans. Pohlhausen Solution # Which one is exact and can be derived from the Pohlhausen correction by setting cube root of Prandtl number equal to 108 and is useful in gases and water? Ans. Blausius Solution # What another term for skin friction coefficient? Ans. Fanning friction factor # The Darcy friction factor is ________. Ans. 4 times # The ______ for submerged bodies in the sum of the skin friction coefficient is? Ans. Drag coefficient # What can predict the average film coefficient along the entire length of laminar flow? Ans. Labarsky-Kauffman Correlation # When can we say that the laminar flow is “Fully Developed”? Ans. When the difference between the surface (wall) and the mean fluid temperature is Constant # Where is the accumulator usually located? Ans. Before the suction line # A device used to remove moisture and foreign particles inside the refrigeration system. Ans. Filter drier # What is the standard value of the atmospheric pressure at the atmospheric sea level? Ans. 14.7 psi # Requirements are classified according to: Ans. Manner of absorbing heat # A push or pull on any thing is called: Ans. Force # Anything that takes up space has mass Ans. Matter # Factor use in calculating the overall heat transfer through the tube walls of the condenser tube of Another heat transfer surface. Ans. Fouling factor # What could be the possible cause of the abnormal discharge temperature? Ans. All of the above # Where is the relief valve located? Ans. At the discharge side of the compressor # What will you do before starting a refrigeration system? Ans. Vent the condenser # What is the purpose of the expansion valve bypass? Ans. To control the refrigerant to the evaporator in case the automatic valve fails. # How many feed water lines are connected to the boiler? Ans. 2 # Exhaust gases from the engine possesses what type of energy?

Ans. Kinetic energy # What is the instrument used to measure density? Ans. Hydrometer # Engine overall efficiency is ________. Ans. Brake thermal efficiency # What is the ratio of the fuel usage rate to the power generated? Ans. Specific fuel consumption # What is the ratio of the air mass that enters the engine to each mass of fuel? Ans. Air-fuel ratio # The ohmmeter is an instrument used to measure Ans. Resistance # The ammeter is an electrical instrument used to measure Ans. Current # The electrical pressure that pushes the electrical current or objections. Ans. Voltage # The voltmeter is an instrument used to measure: Ans. Voltage # When the winding is shorted, what is the resistance reading? Ans. Zero ohm # Which tools are used to enlarge the end of the tubing to connect a fitting? Ans. Flaring tools # The tubing is never used when the fluid temperature goes beyond 100 to 175 mm? Ans. Plastic # The tubing to be swaged is damped in a Ans. Flaring block # The hand tools used to lightened and loosen the fitting are Ans. Wrenches # In soldering process, What substance is used to prevent oxidation? Ans. Flux # The process of applying molten metal to heated metals is called: Ans. Soldering # What is the physical state of the refrigerant? Ans. Gas # Which of the following appliances uses a finned-coil evaporator? Ans. Window-type airconditioner # The operation of condenser is the opposite of that of Ans. Evaporator # Which of the following devices maintains the pressure difference between the evaporator and the condenser? Ans. Flow control # What device is ideal for measuring the flow of liquid metal? Ans. Magnetic flow meter # The frictional force that acts parallel but opposite to the direction of motion. Ans. Drag # A pump that transform kinetic energy into fluid static pressure. Ans. Kinetic pump

# Due to friction losses between the fluid and the pump and mechanical losses in the pump itself. Ans. The brake pump power will be greater than hydraulic power # The net energy actually transferred to fluid per unit time is: Ans. Hydraulic power # The input power delivered by motor to pump. Ans. Brake pump power # Boiler tubes are specified by Ans. Outside diameter # The maximum size of the boiler down lines valves and fittings shall not exceed Ans. 2.5 inches # The purpose of an expansion tank in a hot water heating system is to provide for the expansion of Ans. Water # Heavy accumulation of a soot in the boiler will result in: Ans. Loss of boiler efficiency # How are tubes secured in a fired tube boiler will result in: Ans. Rolled and beaded over # A tube in a fire tube boiler is surrounded by: Ans. Water # What is the formula for the factor of evaporation? Ans. H-b/970.3 # What can cause a super heater tube to overheat? Ans. Dirty tube # How many square feet of heating surface does a 3-in fire tube 20 ft long with a 3/16 in . wall have? Ans. 13.7 # In a high temperature hot water boiler; what could cause a tube to over heat? Ans. Poor circulation # A down corner is: Ans. Found in a water tube boiler # Extreme firebox temperature changes cause: Ans. Spalling # Low water cu. off: Ans. Shuts the burner when the boiler is low on water # Steam coming from the bottom by cock would indicate: Ans. Low water level # Why is the method to cool water called condenser not practical? Ans. Water is expensive # A device use to collect liquid refrigerant entering the compressor to prevent trouble in compressor is Ans. Accumulator # A disiccant is a material that has high affinity to water. Which is the common used dissicant Ans. Silica gel # The only means of preserving food in its original fresh state is _____________.

Ans. Refrigeration # PVC means Ans. Poly Vinyl Chloride # What does ABS means? Ans. Acrylonitrite Butadiene Styrene # What measures the average roughness of imperfection inside the pipe? Ans. Specific roughness # Darcy factor, friction factor is not constant but decreases as the Reynolds number increases up to a certain point known as: Ans. Full turbulent flow # What presents the friction factor graphically as 6 functions of Reynolds number and relative roughness. Ans. Moody diagram # If the flow in truly laminar, and fluid flowing in a circular pipe then which of the equation is appropriate to use? Ans. Hagen Poisenille Equation # Which is true about Hagen Williams equation? Ans. It is primarily use for water # For highly turbulent flow, what causes shear stress? Ans. Momentum Effect # In a circular pipe laminar flow momentum flux is: Ans. Maximum at the pipe wall # If the head pressure is too high Ans. The high pressure cut-out switch should operate # In a combustion engine cycle, what is the ratio of the net output power to the input? Ans. Thermal efficiency # In a reciprocating engines, what is the ratio of actual to ideal volumes of entering gases? Ans. Volumetric efficiency # In a combustion engine, what is the ratio of the actual power developed to the ideal power developed? Ans. Mechanical efficiency # The equivalent of ratio of emissive power to absorptivity for both thermal equilibrium is described by: Ans. Kirchoffs law # The ideal cycle based on the concept that the combination process in both diesel and gasoline in the combination of heat transfer process that is constant pressure and constant volume. Ans. Dual cycle # The ratio of the volume at the end of heat addition to the volume at the start of heat addition is called: Ans. Cut-off ratio # A theoretical body which when heated to incandescence would emit continuous light ray spectrum.

Ans. Blackbody # Which of the following is the reason for insulating the pipes? Ans. Heat loss from the surface is minimize # Heat transfer due to density differential Ans. Convection # A process involving a gas that cannot, even in principle be reserved with being done. Ans. An isobaric compression # The maximum amount of heat that can be converted into mechanical energy Ans. Depends on the intake and exhaust temperature # The maximum amount of mechanical energy converted into heat Ans. 100% # The work output of energy heat engine. Ans. Equals the difference between its heat intake and heat exhaust # To increase the output of a centrifugal pump, you must Ans. Speed of rotation # By superheating the refrigerant in vapor compression cycle with useful cooling, which of the following will increase? (Use per unit mass analysis) Ans. Heat rejected from condenser # By superheating the refrigerant in vapor compression cycle with useful cooling, which of the following will decrease? (Use per unit mass analysis) Ans. Mass flow rate # By superheating the refrigerant in vapor compression cycle with useful cooling, which of the following will decrease? (Use per unit mass analysis) Ans. COP # By superheating the refrigerant in vapor compression cycle with useful cooling, which of the following will increase? (Use per unit mass analysis) Ans. Compressor power # By superheating the refrigerant in vapor compression cycle with useful cooling, which of the following will not be affected? (Use per unit mass analysis) Ans. Compressor power # By sub-cooling the refrigerant in 110apour compression cycle at condenser exit, which of the following will increase? (Use per unit mass analysis) Ans. Refrigerating effect # By sub-cooling the refrigerant in 110apour compression cycle at condenser exit, which of the following will decrease? (Use per unit mass analysis) Ans. Mass flow rate # By increasing the vaporizing temperature in vapor compression cycle, which of the following will increase? (Use per unit mass analysis) Ans. COP # By increasing the vaporizing temperature in 110apour compression cycle, which of the following will decrease? (Use per unit mass analysis) Ans. temperature difference between evaporator and compressor # By increasing the condenser pressure in 110apour compression cycle, which of the following will increase? (Use per unit mass analysis) Ans. Compressor power

# By increasing the vaporizing temperature in 111apour compression cycle, which of the following will decrease? (Use per unit mass analysis) Ans. moisture content after expansion # If the pressure drop in the condenser increases in a vapor compression cycle, which of the following will increase? (Use per unit mass analysis) Ans. heat rejected in the condenser # If the pressure drop in the condenser increases in a 111apour compression cycle, which of the following will decrease? (Use per unit mass analysis) Ans. mass flow rate # If the pressure drop in the condenser increases in a 111apour compression cycle, which of the following will not be affected? (Use per unit mass analysis) Ans. compressor power # If the pressure drop in the evaporator increases in a 111apour compression cycle, which of the following will increase? (Use per unit mass analysis) Ans. heat rejected in the condenser # If the pressure drop in the condenser increases in a 111apour compression cycle, which of the following will decrease? (Use per unit mass analysis) Ans. COP # By lowering the condenser pressure in Rankine cycle, which of the following will decrease? (Use per unit mass analysis) Ans. heat rejected # By increasing the boiler pressure in Rankine cycle, which of the following will decrease? (Use per unit mass analysis) Ans. heat rejected # By superheating the steam to a higher temperature in Rankine cycle, which of the following will decrease? (Use per unit mass analysis) Ans. moisture content at the turbine exhaust # By superheating the steam to a higher temperature in Rankine cycle, which of the following will increase? (Use per unit mass analysis) Ans. cycle efficiency # By superheating the steam to a higher temperature in Rankine cycle, which of the following will decrease? (Use per unit mass analysis) Ans. pump work # What type of boiler incorporates furnace water cooling in the circulatory system? Ans. Integral-furnace boiler #The main components of a combined cycle power plant are: Ans. Gas turbine and waste heat boiler # Which of the following indicators is used to determine the anti-knocking characteristics of gasoline? Ans. Octane number # Indicate the false statement Ans. The heat transfer cannot exceed the work done. # In fluid flow, linear momentum is: Ans. A vector quantity equal to the product of mass and velocity # A fact that a fluid’s velocity increases as the cross-sectional area of the pipe through which it flow decreases due to:

Ans. The continuity equation # The coefficient of contraction is the ratio of: Ans. Area of vena contracta to the orifice area # The coefficient of discharge is the ratio of: Ans. Actual discharge to the theoretical discharge # The coefficient of velocity is the ratio of: Ans. Actual velocity divided the theoretical velocity # Flow measuring devices include all of the following except: Ans. Magnetic dynamometers # In the series pipe systems, all of the following parameters vary from section to section except Ans. Mass flow # The coefficient of velocity accounts for the: Ans. Small effect of friction and turbulence of the orifice # Expansion factors take into account the Ans. Effects of compressibility # The water hammer phenomenon is primarily what kind of fluid mechanics? Ans. Dynamic (a time-dependent phenomena) # All of the following are forms of drug on a body moving through a fluid except: Ans. D’Alembert’s paradox drug # The function of a turbine is to: Ans. Extra energy from the flow # The fact that there is no bodies moving through an ideal fluids is known as: Ans. D’Alembert’s paradox # Liquids and gases takes the following characteristics of their contents. Ans. Shapes # All of the following dimensionless parameters are applicable to fluid flow problem excepts Ans. Bolt number # All of the following can be characteristics of fluids except: Ans. Hysteresis # The most common method for calculating frictional energy loss for laminar flowing fluid is non- circular pipes is: Ans. The Darcy equation # For computation convenience, fluids are usually classed as Ans. Real and ideal # Which of the following is not a characteristics of real fluids? Ans. Experience of eddy currents and turbulence # Property of a fluid whereby its own molecules are attracted is known as: Ans. Cohesion # The term subsonic flow refers to a flowing gas with a speed. Ans. Less than the local speed of sound # The difference between stagnation pressure and total pressure Ans. None of the terms are interchangeable # The presence of friction in the hydraulic grade line will always cause the line to slope. Ans. Down in the direction of the flow

#The presence of minor loss in the energy grade line will cause the line to slope Ans. Down in the direction of the flow # If the Mach number is greater than 1 but less than 5 Ans. Supersonic # The flow is called sonic when mach number is Ans. Equal to 1 # The flow is sub-sonic when mach number is Ans. Less than 1 # To check water level in the gauge glass of a steam boiler Ans. Use the # The flame failure control is tested by? Ans. Shutting off the fuel supply to the burner # During purge cycle what is the minimum amount air volume changes required? Ans. Four (4) # The purge cycle time for gas compared to oil is: Ans . The same #A high (CO) Carbon monoxide reading, indicates: Ans. None of these # The induced draft fan is located: Ans. In the breaching # A balanced draft boiler required: Ans. Both forced and induced draft # What percent would expect in a well maintain boiler burning number 6 oil? Ans. 15% # Laminar friction factor of fluid is flowing through a pipe is a function of all the following except: Ans. Pipe roughness # The stream function is a useful parameter in describing: Ans. Conservation of mass # The study of the practical law of fluid and the resistance of open pipes and channels is the _____ Ans. Hydraulics # The most common methods of calculating frictional energy loss for laminar flow fluids in non circular pipes: Ans. Equation # The parameter in the expression for head loss is: Ans. Darcy friction factor # The characteristic length of Reynolds number use to calculate the friction in noncircular full running pipes is based on the: Ans. Hydraulic diameter # The hydraulic radius of a non-circular pipe is: Ans. The ratio of flow area to wetted parameter: # An electromagnetic radiation wavelength is in the 0.1 to 10.000 mm range: Ans. Thermal radiation # What characteristics makes the difference between a Newtonian and non-newtonian fluids?

Ans. Their viscous behaviour # What instrument use to measure salt solution: Ans. Salimeter # A measure of fluid resistance to flow? Ans. Viscosity # What is another for absolute viscosity? Ans. Dynamic # What is the reciprocal of viscosity? Ans. Fluidity # The density of fresh water is a ship will float. Ans. Lower in fresh water than in sea warer # Bernoulli’s equation is based on? Ans. Conservation of energy # An express train goes past a station flatform at high speed a person standing in the edge of the flatforms tend to be: Ans. Attracted to the train # The volume of fluid flowing per second out of an orifice at the bottom. A tank does not depend on: Ans. Density of fluid # Addition of detergent to water? Ans. Decreases its surface tension # Water neither rise or falls in silver capillary. This suggest that the contact angle between the water and silver is: Ans. 90 degrees # Most pressure in the Freon system have two dials or graduation on 1 gage what does the two dial represent: Ans. Temperature and pressure # The flame detector is directed: Ans. Into the furnace area #Which of the following statement is true? Ans. All of the above #Which of the following is a pseudo plastic fluid? Ans. All of the choices # A type of fluids which are capable of indefinitely resisting a small shear stress but move easily when the stress becomes large: Ans. Bingham fluids # Which of the following is an example of Bingham fluids? Ans. All of the above # The Bingham fluids will become pseudo plastic when: Ans. The stress increases # A type of fluids which exhibits viscosities that increases with an increasing velocity gradient. Ans. Dilatant fluid # The viscosity decreases with time the fluids is said to be: Ans. Thixotropic fluid # Viscosity increases with time the fluids is said to be:

Ans. Rneopectic fluids # What is the dominating cause of viscosity in fluids: Ans. Molecular cohesion # In a compressor, the piston is said to be at _____ when it is at its most petracted position in the stroke, Ans. Bottom dead center # Reciprocating compressors are characterized by their Ans. Clearance volume # The gases remaining in the clearance volume after the discharge in the dead center are known as the: Ans. Residual gases # The ratio of the clearance volume to the swept volume is known as: Ans. Percent clearance or just clearance # The work done in an adiabatic (isentropic, process is _____ the work done by the isothermal process; Ans. Less than # The work done in an isentropic compressor is ____ the work done by the isothermal compressor; Ans. Greater than # The term used to describe a partially compressed gases withdrawn cooled and compressed further; Ans. Intercooling # A perfect intercooling refers to the case where the gases cooled to: Ans. The original inlet temperature # Multi stage blowers can reach pressure up to ____ of water; Ans. 100 inches # What represents the ratio of the coefficient of the friction at the expanded in pumping a fluid; Ans. Fried heat transfer efficiency factor # What is the proper oil storage temp for No. 6 oil? Ans. 20 ºF above pour point # To prime a pump Ans. Fill casing with water # Balance draft means; Ans. Almost atmosphere pressure in the furnace # A gallon of No. 6 oil contains: Ans. 152,000 Btu # What is the efficiency of a steam plant with a heat rate of 12,000 Btu/KW? Ans. 28.4% # What type of fuel must be preheated to burn properly? Ans. Heavy oil # The fuel oil for mechanical atomizing burners is usually heated Ans. 150 ºF # Fuel suppliers specify a minimum flash point for their oil because a low flash point oil is; Ans. Dangerous

# How many cubic feet of furnace volumes are needed to burn one gallon of No. 6 oil? Ans. 10 # A fuel oil is heated its viscosity Ans. Decreases # Only ____ of the turbine work output is required to operate the pump. Ans. 0.04% # Superheating the steam to higher temperature decreases the moisture content of the steam at the __ Ans. Turbine exit # Regeneration also provides a convenient means of dearating the feedwater to prevent. Ans. Boiler corrosion # Can be apply in Steam turbine cycle (Rankine), Gas turbine cycle (Brayton) and Combined cycle. Ans. Cogeneration plant # In a Rankine cycle with fixed turbine inlet conditions. What is the effect of lowering the condenser pressure, the heat rejected will; Ans. Decrease # In an ideal Rankine cycle with fixed boiler and condenser pressures. What is the effect of superheating the steam to a higher temperature, the pump work input will; Ans. Remains the same # How do the following quantities change when the simple ideal Rankine cycle is modified with regeneration? The heat rejected will; Ans. Decreases # During a combustion process, the components which exist before the reaction are called Ans. Reactants # Is an obvious reason for incomplete combustion. Ans. Insufficient oxygen # Higher heating value when H2O in the product of combustion is in Ans. Liquid form # Device which transfer heat from low temperature medium to a high temperature one is a Ans. Heat pump # A rule of thumb is that the COP improves by _____ for each °C the evaporating temperature is raised or the condensing temperature is lowered. Ans. 2 to 4% # Are generally more expensive to purchase and install than other heating systems, but they save money in the long run, Ans. Heat pumps # The most widely used absorption system is the ammonia-water system, where ammonia serves as refrigerant and H2O as the Ans. Transport medium # Known as drum less boiler. Ans. Once-through boiler # Reduces the steam temperature by spraying low temperature water from boiler drum.

Ans. Desuperheater # Carbon dioxide can be removed by; Ans. Aeration # Is often used to absorb silica from water. Ans. Magnesium hydroxide # Presence of excess hydrogen ions makes the water Ans. Acidic # PH of water varies with Ans. Temperature # PH value of ____ is usually maintained for boiler water to minimize corrosion. Ans. 10.5 # What type of turbine that has a degree of reaction of 1/2? Ans. Rarsons turbine # The cooling water is made to fall in series of baffles to expose large surface area for steam fed from below to come in direct contact. Ans. Barometric condenser # Show the variatiom of river flow (discharge) with time. Ans. Hydrograph # The pressure at the bottom of a vessel filled with liquids does not depend on the; Ans. Area of the liquid surface # A person stands on a very sensitive scale, inhales deeply the reading on the scale: Ans. Depends on the expansion of the person’s chest, relative to the volume inhaled. # Buoyancy occurs because, with increasing depth in a fluid: Ans. The pressure increases # In order for an object to sink when placed in water its average specific gravity must be; Ans. More than 1 # The average solar energy heating the outer edge atmosphere is approximately 442 Btu per hour-foot squared of 1.41KW per meter squared is known as: Ans. Solar constant # The ratio of total dissolve solids in the recirculating water to the total solids in the make-up water. Ans. Cycles of concentration # What is the another term for Bryton cycle: Ans. Joule cycle # The process of transferring heat from low temperature area to high temperature area; Ans. Refrigeration #The device to remove heat from water: Ans. Chiller # What device which transfer heat from low temperature area to operate on refrigeration system: Ans. Heat pump # Refers to the process of one substance mixing with another substance: Ans. Diffusion # Which of the following statement about Newtonian fluid is most accurate:

Ans. Shea stress is proportional to the rate of strain # Which is not characteristic of fluids? Ans. Hysteresis # The relationship between pressure and altitude in the atmosphere: Ans. Barometric height relationship # Flash point means: Ans. Ignition temperature # Fire point means: Ans. None of the above # Bernoulli’s equation is a Ans. Conservation of energy equation # The pressure at given depth due to several immiscible liquid is: Ans. The sum of individual pressure # The relationship between pressure and altitude in the atmosphere is given by the Ans. Barometric height relationship # The fact that the buoyant force on a floating object is equal to the weight of the water displaced Ans. Archimedes principle # Which of the following terms does not appear in the steady flow energy equation in the extended Bernoulli’s equation Ans. Hysteresis losses # The pitot tube can be use to measure fluid velocity as described by the Bernouilli equation and the relationship between; Ans. Kinetic energy and static pressure # The difference between stagnation pressure and total pressure is; Ans. None. The terms are interchangeable # Fully turbulent flow in a pipe is characterized by all the following except: Ans. Parabolic velocity profile # What is the bulk modulus of the water is; Ans. 300,000 psi # Atmospheric pressure does not correspond to approximately; Ans. 98 N/m2 # Hydraulic press is able to produce a mechanical advantage because; Ans. An external pressure extended on a fluid is transmitted uniformly through its volume # In the operation of a hydraulic press, it is impossible for the output piston to exceed the input piston Ans. Work # If one of the management legs is inclined it is known as: Ans. Draft gauge # Why are manometer tube generally large in diameter? Ans. To avoid significant capillary effect # Bernoulli’s equation is an energy conservation based on several reasonable assumptions: Ans. All of the above # Body that emits a constant emissivity, regardless of the wavelength;

Ans. Gray body # What gives the total emissive power from a black body Ans. Stefan-Boltzmann law of the fourth power # What accounts for the spatial arrangement of the body and their emissivity; Ans. Emissivity factor # The gray body shape factor is the product of the Ans. Black body shape factor and emissivity factor # The product of the area and the shape factor is known as; Ans. Geometric flux # Surfaces that re-radiates absorb thermal radiation are known as; Ans. Refractor materials of refractories # Pitot tube used to measure Ans. Velocity # Liquid gases take the following characteristics of the containers: Ans. Shape # For computational convenience usually classed as: Ans. Real and ideal # Which of the following statement about Newtonian fluid is more accurate? Ans. Shear stress is proportional to the rate of strain # Which of the following is not a characteristic of a real of fluid? Ans. Experience of eddy current and turbulence # One could expect the possibility of Reynold’s number similarity in all of the following cases except Ans. Closed pipe turbulent flow # One could expect the possibility of Froude number similarity in all of the following cases except Ans. Subsonic airflows # The absolute viscosity of a fluid varies in pressure and temperature as a function of; Ans. Shear and angular deformation rate # Turbulent flow of a fluid in a pipe. All of the following except; Ans. Reynold’s number will be less than 2300 # Flow measuring devices include all of the following except; Ans. Magnetic dynamometer # Flow measuring devices include all of the following except; Ans. Mercury barometer # Flow measuring devices include all of the following except; Ans. Precision tachometer # The following are all examples of indirect (secondary) measurements to measure flow rates using obstruction meters except; Ans. Volume tanks # The following are all examples of indirect (secondary) measurements to measure flow rates using velocity meters except; Ans. Weight and mass scales # The following are all examples of indirect (secondary) measurements to measure flow rates using flow meters except; Ans. Positive displacement meters

# In series pipe systems which of the following parameters vary from section to section except; Ans. Mass flow # Venturimeter, pitot static gauges, flow nozzles, manometer vary with; Ans. Flow velocity and pressure # Other name for R 729? Ans. Air # In an indirect refrigeration system, which of the following is used to measure the density of brine? Ans. Hydrometer # Which of the following is the result of opening a hand expansion valve too much in a refrigeration system? Ans. Evaporator will “freeze back” to compressor If one of the manometer legs is inclined, it is known as: Ans. Draft gauge Why are monometer tubes generally large in diameter? Ans. To avoid significant capillary effect Bernoulli’s equation is an energy conservation based on several reasonable assumptions: Ans. All of the above Body that emits a constant emissivity, regardless of the wavelength: Ans. Gray body What gives the total emissive from a black body: Ans. Stefan-boltzman law of the fourth power What accounts for the spatial arrangements of the body and their emissivity? Ans. Emissivity factor The gray body shape factor is the product of the Ans. Black body shape factor and emissivity factor The product of the area and the shape factor is known as: Ans. Geometric flux Surfaces that re-radiates absorb thermal radiation are known as: Ans. Refractory materials of refractories Pito- tube used to measure Ans. Velocity Liquid gases take the following characteristic of the containers: Ans. Shape For computation convenience usually classed as: Ans. Real and ideal Which of the following statement about Newtonian fluid is more accurate? Ans. Shear stress is proportional to the rate of strain Which of the following is not a characteristic of a real fluid? Ans. Experience of eddy current and turbulence One could expect the possibility of Reynold’s number similarity in all of the following cases except Ans. Closed pipe turbulent flow

One could expect the possibility of Froude number similarity in all of the following cases except Ans. Subsonic airflows Difference between energy grade line (friction) and the energy line friction loss is: Ans. Friction and minor losses The absolute viscosity of a fluid varies in pressure and temperature as a function of: Ans. Shear and angular deformation rate Turbulent flow of a fluid in a pipe. All of the following are true except: Ans. Reynold’s number will be less than 2300 Flow measuring devices include all of the following except: Ans. Mercury barometer Flow measuring devices include all of the following except: Ans. Precision tachometer The following are all examples of indirect (secondary) measurements to measure flow rates using obstruction meters except: Ans. Volume tank The following are all examples of indirect (secondary) measurements to measure flow rates using velocity meters except: Ans. Weight and mass scales The following are all examples of indirect (secondary) measurements to measure flow rates using flow meters except: Ans. Positive displacement meter In series pipe systems which of the following parameters vary from section to section except: Ans. Mass flow Venturimeter, pitot static gauges, flow nozzles, manometers vary with: Ans. Flow velocity and pressure Other name for R 729? Ans. Air In an indirect refrigeration system, which of the following is used to measure the density of the brine? Ans. Hydrometer Which of the following is the result of opening a hand expansion valve too much in a refrigeration system? Ans. Evaporator will “freeze back” to compressor Types of turbine used up to 300m head. Ans. Deriaz turbine A turbine that has a diagonal flow. Ans. Deriaz turbine Oil is atomized either by air blast or pressure jet at about Ans. 70 bar Type of solid injection that use single pump supplies fuel under high pressure to a fuel header. Ans. Common rail injection Water flow in diesel engine that is caused by density differential. Ans. Thermosiphon cooling

Type of lubrication system in diesel engine in which oil from pump is carried to a separate storage tank outside the engine cylinder and used for high capacity engine. Ans. Dry sump lubricating system Produces extreme pressure differentials and violent gas vibration Ans. Detonation In a spark ignition engine, the detonation occurs near the______________. Ans. End of combustion In a compression ignition engine, the detonation occurs near the _____________. Ans. Beginning of combustion Morse test is use to measure the _________ of multi-cylinder engine. Ans. Indicated power Ignition delay can be minimized by adding ______ to decrease engine knocking. Ans. Ethyl nitrate The work done in throttling valve is Ans. Zero Stagnation enthalpy represents the enthalpy of a fluid when it is brought to rest ______. Ans. Adiabatically Represents the temperature an ideal gas attains when it is brought to rest adiabatically. Ans. Stagnation temperature After passing through a nozzle the density of fluid decreases as the fluid velocity. Ans. Increases The volume flow passes through a venture meter will Ans. Constant A converging-diverging nozzle is the standard equipment in Ans. Supersonic aircraft Nozzles efficiencies range from Ans. 90%-99% By reheating the steam before entering the second stage in Rankine cycle, which of the following will increase? Ans. Heat rejected Contains 90% gasoline and 10% ethanol Ans. Gasohol In an evaporator section in refrigerating unit, which of the following is the function of evaporator? Ans. Absorption of latent heat of vaporization Which of the following may not be the type of bulb in R-12 refrigeration system? Ans. Duplex Which of the following shuts down the compressor motor when discharge pressure rises to a predetermined point, it cuts in motor when pressure drops to predetermined pressure: Ans. High pressure cut-off switch Which of the following shuts down the compressor motor when discharge pressure rises to a predetermined pressure and shuts down the compressor motor when there is decrease in pressure in the suction line. Ans. Low pressure cut-off switch The P.O. service pump must have:

Ans. A means of shutting down from outside the machinery space The relief valve on the P.O. transfer pump discharge Ans. Back to the transfer pump discharge When heating hot water, feed pump should at least be how many mm to prevent vapor lock? Ans.1220 mm The water column should be provided with a valve drain at least: Ans. 15 mm pipe size Lubricating oils have flash points from Ans. 375-800 degree Fahrenheit A device receives information in one form of instrument signal and transmits an output in another form; Ans. converter Refinement petroleum Naphtha which by its composition is suitable for use in internal combustion engine; Ans. Gasoline It is possible for water to get into the fuel oil by: Ans. All of the above Air chambers are attached to the: Ans. Discharge side of the reciprocating pumps On automatic combustion control: Ans. Reduce the number of burners when the oil pressure approaches minimum specified The volumetric efficiency of the compressor increases as the suction pressure: Ans. Increase As the discharge pressure increases, the volumetric efficiency: Ans. Increases When the suction And discharge pressure are varied in such a direction that the compression ratio is increased, the volumetric efficiency of the compressor: Ans. Decrease For a compressor of any given clearance, the volumetric efficiency Ans. Varies with compression ratio The useful energy transfer in Btu/hr divided by input power in Watts; Ans. Energy efficiency ratio If air is humidified by injecting steam of by pressuring the air through a hot water spray, the dry bulb temperature and enthalpy of air Ans. Increase Air passing through a solid or absorbed bed, such as silica gel or activated alumina will: Ans. Decrease in humidity A device that passes air through dense spray of recirculating water; Ans. Air washer The mass of water sprayed to the mass of air passing through the washer per unit time. Ans. Spray ratio When the cooling tower is used to provide cold water for the condense of a refrigerator system the water circulation system will be approximately: Ans. 3 gal/min-ton Water lost in small droplets and carried away by the air flow:

Ans. Drift The ratio of total dissolved solids in the circulating water to the dissolved solids in the make-up water Ans. Ratio of concentration Through windage removes some solids, most must be removed by bleeding some of the water. Ans. Bleed-off A dry cooling tower where stream travels through large diameter trunks to a cross-flow heat exchanger where it is condensed and cooled by air; Ans. Direct condensing tower An economizer is used to: Ans. Heat the feedwater before it enters the boiler A manometer measures: Ans. Air pressure A pyrometer measures: Ans. Stack pressure An attemperator is another name for a Ans. Desuperheater Waterwalls are used to: Ans. Carry of excess heat from the furnace walls The studs on waterwalls tubes are to: Ans. Give added surface area to tubes The heats remove from the cold area Ans. Use for energy transfer On the upstroke of the piston, the low pressure vapor is first compressed and then discharged as high pressure vapor through the discharged valves into the: Ans. Head of the compressor The vapor that remains the clearance space at the end discharge stroke is called Ans. Clearance vapor Which of the following steps would you take if you found an accumulation of oil on the furnace floor? Ans. Open all registers Which of the following method can be used to clean up the inside of the boiler? Ans. All of the above Another term for the oil discharge strainer is the: Ans. Hot strainer In a controlled feed water system the power water level maintained by: Ans. The fluctuation water level in the boiler The difference between the enthalpy of the product at a specific state and the enthalpy of reactants at the same state for a complete reaction: Ans. Enthalpy of reaction The amount of heat released during a steady flow combustion process when I kg of fuel burned completely at a specific temperature and pressure. Ans. Enthalpy of combustion The enthalpy of a substance at a specific state due to its chemical composition Ans. Enthalpy of formation

The inlet side of condenser tube are rolled and flared to: Ans. Allows a smoother entrance flow of circulating water The purpose of inspection plates on the main condenser is to: Ans. Check tubes without removing condenser heads A pump that has one steam cylinder and two liquid cylinders is called a: Ans. Tandem pump A mixture of dry air and superheated water vapor Ans. Unsaturated air When the air is unsaturated, what is the relation between the dry bulb temperature and wet bulb temperature? Ans. Less than The difference between the dry bulb temperature and wet bulb temperature Ans. Wet bulb depression The relative ratio of water vapor to dry air is: Ans. Humidity ratio Also known as humidity ratio: Ans. Specific humidity The ratio of the actual humidity ratio to the saturated humidity ratio: Ans. Degree of saturation Also known as saturation ration: Ans. Percentage humidity The partial pressure of the water vapor divided by the saturation pressure: Ans. Relative humidity Rapidly increasing the temperature through the air of a chord will cause the water in the gauge to Ans. Evaporate Since the increase in the water vapor a latent heat contents equals the decrease in the air’s sensible heat, the total enthalpies before and after adiabatic saturation is: Ans. Equal An adiabatic saturation process follows a line of constant Ans. Dew point temperature Which of the following used with adiabatic saturation process? Ans. Saturation efficiency The saturation efficiency of a large commercial air washer is typically Ans. 80%-90% The presence of oil or scale on the boiler tubes causes: Ans. Overheating What would you do before giving a boiler bottom blow off? Ans. Raise the water level When installing a new gauge glass in water gauge, you should secure the bolts from? Ans. Center, alternately, toward each end One of the main purpose of refractories in the boiler furnace is to: Ans. Prevent excessive furnace heat losses A fuel oil service pump steam valves are fitted with: Ans. Each rod leading to a location outside the fire room Water tube boilers have

Ans. No fusible plugs The valve that prevents water from backing out of the boiler in the feed water line: Ans. Feed check valve The fuel oil heater is located Ans. On the discharge side of the service pump In the forced draft system: Ans. One fan supplies air to all furnaces In the closed fire room system Ans. The fire room is supplied with air from one fan The air cock on a boiler is located at the: Ans. Highest point of the steam and water drum Soot blowers should be used in proper sequence so that Ans. The soot will be upright toward the uptake What is the first thing you would check on taking over a watch? Ans. The water level A boiler with a fan/blower located in the uptake is operating on Ans. Induced draft A fire room that is isolated(closed) operates on: Ans. Forced draft How many feed water lines are connected to the boiler? Ans. Two If the water in the gauge glass has not been blown for a period of time, the level of water in the glass will be: Ans. less than that in the steam and water drum The enthalpy of fluid when it is brought to rest adiabatically Ans. Stagnation enthalpy In the absence of any heat and work and any changes in potential energy, the stagnation enthalpy of a fluid during a steady flow process: Ans. remains constant During the stagnation process, which of the following is converted to enthalpy? Ans. Kinetic energy The cross section of a nozzle at the smallest flow area is called: Ans. Throat The properties of a fluid at a location where the Mach number is unity are called: Ans. Critical properties What is the average fuel oil temperature range of the oil in the discharge line? Ans. 180-200 degree F Which of the following is a common type of oil burner? Ans. All of the above The boiler gauge glass should be blown down. Ans. At the beginning of every watch Gauge pressure of 200 pounds is equivalent to what absolute pressure? Ans. 215 In a fuel oil service system, the quick closing fuel oil shut-off valve is located between the: Ans. Master fuel oil shutoff valve and the boiler

A fuel oil meter placed between the fuel oil service pumps and the fuel oil heaters. Ans. Cold-type meter The presence of oil or scale on boiler tubes causes: Ans. Overheating What would you do before giving a boiler bottom blow off: Ans. raises the water level When installing a new gauge glass in a water gauge you should secure the bolts from: Ans. Center, alternately towards each ends One of the main purpose of refractories in the boiler furnace is to: Ans. Prevent excessive furnace heat losses A fuel oil service pump steam valves are fitted with: Ans. Each rod leading to a location outside the fire room Water tube boilers have Ans. No fusible plugs The valve that prevents water from backing out of the boiler in the feed water line: Ans. Feed check valve The fuel oil heater is located Ans. On the discharge side of the service pump In the forced draft system: Ans. One fan supplies air to all furnaces In the closed fire room system Ans. The fire room is supplied with air from one fan The air cock on a boiler is located at the: Ans. Highest point of the steam and water drum Soot blowers should be used in proper sequence so that Ans. The soot will be upright toward the uptake What is the first thing you would check on taking over a watch? Ans. The water level A boiler with a fan blower located in the uptake is operating on Ans. Induced draft A fire room that is isolated (closed) operates on: Ans. Forced draft How many feed water lines are connected to the boiler? Ans. Two If the water in the gauge glass has not been blown down for a period, the level of water in the glass will be: Ans. Less than that in the steam and water The enthalpy of fluid when it is brought to rest adiabatically. Ans. Stagnation enthalpy In the absence of any heat and work and any changes in potential energy, the stagnation enthalpy of a fluid during a steady flow process; Ans. Remains constant During the stagnation process, which of the following is converted to enthalpy? Ans. Kinetic energy The cross section of a nozzle at the smallest flow area is called: Ans. Throat

The properties of a fluid at a location where a Mach number is unity are called: Ans. Critical properties What is the average fuel oil temperature range of the oil in the discharge line? Ans. 180-200 degree Fahrenheit Which of the following is a common type of burner? Ans. All of the above The boiler gauge glass should be blown down. Ans. At the beginning of every watch Gauge pressure of 200 pounds is equivalent to what absolute pressure? Ans. 215 In a fuel oil service system, the quick closing fuel oil shut-off valve is located between the: Ans. Master fuel oil shutoff A fuel oil meter placed between the fuel oil service pump and fuel oil heater Ans. Cold-type meter When securing a scotch boiler Ans. The belly plug must be removed If the cooling coils temperature is between the airs dew point, the moisture will Ans. Condense What occurs when the air passes through a water spray in an air washer? Ans. Evaporative cooling process To prevent ice buildup, the cooled air temperature should be kept from dropping Ans. Below the freezing point of water What precaution must be taken before using steam soot blowers? Ans. Drain thoroughly Which of the following would you do before blowing the tubes with steam soot blower? Ans. All of the above Which of the following would cause a flareback? Ans. Trying to relight from hot Sputtering oil burners might indicate Ans. Water in the fuel oil The boiler feed water in the feed water heater is heated by: Ans. Auxiliary exhaust steam If the boiler is smoking white smoke, a possible cause would be: Ans. Too much air The Edwards-type air pump has: Ans. One set of valve A scoop condenser is a: Ans. Single pass condenser The purpose of the steam to baffle in a condenser Ans. Prevent the steam from hitting directly on the cooler tubes The condenser vacuum feed valve is open and the water in the reserve tank is dry. The result is: Ans. A loss of vacuum The division plate in a two-pass-condenser

Ans. Prevents the circulating water from passing directly to overload discharge The holes in Kinghorn-valve disks Ans. B The equilibrium temperature that a regular thermometer measure if exposed to atmospheric air: Ans. Dry bulb temperature The temperature of air that has gone through an adiabatic saturation process Ans. Wet bulb temperature If the vapor pressure equals the saturation process, the air is said to be: Ans. Saturated When the air is saturated, the dry bulb, wet bulb and the dew point temperature are Ans. Equal On taking over a watch, one should check Ans. All of the above When you are cleaning fuel oil burner tips, use a Ans. Brass knife The amount of steam generated by a boiler is dependent upon Ans. All of the above A boiler operated at a pressure not exceeding 1.055 kg/cm 2 gauge steam of temperature not exceeding 120oC. Ans. Low pressure heating boiler No part of the steam generator should be closer than how many from the wall? Ans. 1 m Steam generator should be mounted over a suitable foundation or concrete not less than 305 mm thick and with sufficient area at base to be supported the bearing capacity of the soil with a factor of safety of Ans. Not less than 4 No smoke stacks should be closer than how many millimeters from the exposed woodwork or framing Ans. 305 mm The air on an extinguisher fire should be Ans. Kept closed Too low F.O. temperature will cause Ans. Poor combustion and smoky fires -when securing a scotch boiler Ans. The belly plug must be remove -if the cooling coils temperature is between the airs dew point, the moisture will Ans. Condense -what occurs when the air passes through a water spray in an air washer? Ans. Evaporative cooling process -to prevent ice build up, the cooled air temperature should be kept from dropping Ans. Below freezing point of water -What precaution must be taken before using steam soot blowers? Ans. All of the above -Which of the following would cause a flareback? Ans. Trying to relight from hot brickwork

-Sputtering oil burners might indicate Ans. Water in the fuel oil -The boiler feed water in the feed water heater is heated by: Ans. Auxiliary exhaust steam -If a boiler is smoking white smoke, a possible cause could be: Ans. Too much air -The Edwards-type air pump has; Ans. One set of valve -A scoop condenser is a: Ans. Single pass condenser - The purpose of the steam baffle in a condenser is to: Ans. Prevent the steam from hitting directly on the cooler tubes -The condenser vacuum feed valve is open and the water in the reserve tank is dry. Ans. A loss of vacuum -The division plate in a two-pass-condenser Ans. Prevents the circulating water from passing directly to overload discharge -The holes in kinghorn-valve discs Ans. B -The equilibrium temperature that a regular thermometer measure if exposed to atmospheric air; Ans. Dry bulb temperature -The temperature of air that has gone through an adiabatic saturation process Ans. Wet bulb temperature -if the vapor pressure equals the saturation process the air is said to be Ans. Saturated -When the air is saturated, the dry bulb, wet bulb and dew point temperature are Ans. Equal -On taking over a watch, one should check Ans. All of the above -When you are cleaning fuel oil burner tips, use a Ans. Brass knife -The amount of steam generated by a boiler is dependent upon Ans. All of the above -A boiler operated at pressure not exceeding 1.055 gauge steam of steam temperature not exceeding 120oC. Ans. Low pressure heating boiler -No part of the steam generator should be closer than how many from the wall? Ans. 1m -Steam generator should be mounted over a suitable foundation or concrete of not less than 305 mm thick and with sufficient area at base to be supported the bearing capacity of the soil a factor of safety of Ans. Not less than 4 -No smoke stacks should be closer than how many millimeters from exposed wood work or framing? Ans. 305 mm -The air register on an extinguisher fire should be

Ans. Kept closed -Too low F.O. temperature will cause Ans. Poor combustion and smoky fires -When the fuel-oil temperature is too high it causes Ans. Carbon deposits on the fuel-oil heater coils -A dry cooling tower where steam is condensed by cold water jets Ans. Indirect condensing dry cooling tower -The hot condensate is pumped to cross heat exchangers whose it is cooled by. Ans. Air -which of the following is the refrigerant “of choice” in entering air conditioning Ans. R-22 -In new equipment, which of the following replaces R-11 Ans. R-12 -The super heater is used to Ans. Remove moisture from steam -Water tube boiler are Ans. Rolled in -The recirculating valve is used to Ans. Recirculate and heat the fuel oil prior to lighting off -The fusible plug in a scotch boiler are found in the Ans. Fire tubes -Which of the following valves are arrange in a Y-branch fitting? Ans. Boiler-drum safety valves -The coils in the feedwater heater are secured by. Ans. Union fitting -Natural draft towers can cool the water to within. Ans. 10 to 12oF Force draft towers can cool the water to within Ans. 5 to 8oF -When you light a fire in the burner, you must always Ans. Use a torch -The recirculating line on the burner valves returns the oil to the Ans. Suction side of service pump -The valve which shuts off liquid line with the least amount of resistance to flow. Ans. Gate valve -The process applied to iron pipe which retards corrosion is called Ans. Galvanizing -Listed in sequence, iron pipe series are; Ans. ¼ ,3/8, ½, ¾ -The two main purpose of the main condenser are; Ans. Convert exhaust steam to water -Pyrometer is a Ans. High temperature thermometer -The temperature and humidity of the air to be used for comfort cooling shall be maintained at effective temperature at air movement within the range from Ans. 0.0762 to 0.127m/s

-As the air enters the living zone the air motion in such occupied space and which the only source of contaminants shall have a velocity of not more than Ans. 0.294m/s -In air conditioning and ventilation standards, as the air enters into living zone, the distance above the floor is Ans. 1.603mm -Carbon dioxide concentration in air when measured 910 mm above the floor shall be estimated Ans. 1000ppm -The useful refrigerating effect equals to 211kj/mm is Ans. Tons of refrigeration -Another name for the fuel oil suction strainer; Ans. Cold strainer -In a manually controlled feed water heater system the proper water level is maintained by Ans. Operating the feed check valve -The actual refrigerating capacity of the compressor Ans. Always less than the theoretical capacity -The ratio of the actual displacement of the compressor to its piston displacement Ans. Total or real volumetric efficiency -Cyclone super heater are located in the Ans. Steam and water drum -The purpose of the blow valve is Ans. Removes oil and scum from surface of water -The purpose of the salinometer cock is to Ans. Obtain a sample of boiler water for testing -Salt in boiler water usually contains from Ans. Leaky condenser tubes -Before blowing down a boiler always Ans. Open skin valves -The mass flow rate produced by the compressor is Ans. Equal to the mass of the suction vapor that the compressor takes in the entire suction inlet unit time -In order to increase back pressure, one must Ans. Close in on the back pressure valve -The actual steam that condenses in the feedwater heater Ans. Discharge to the hotwell -The excess steam pressure in the back pressure system exhausts to the Ans. Atmosphere through a relief valve -The air pressure supplied to the boiler is measure in Ans. Inches of water -The condensate from the coils in the fuel oil heater to the fuel oil heaters goes to the Ans. Observation tank The valve between the fuel oil heaters and the burner valve is called the Ans. Root valve -The safety device located in the crown sheet of a scotch boiler is called a

Ans. Fusible plug -Tube retarders in scotch boilers are used to Ans. Slow down the combustion gases -In Order for oil to burn properly it must Ans. Al of the above -The steam that is discharge from safety valves goes to the Ans. Atmospheric line -The polytrophic exponent approaches one with the change in the process____approaches zero Ans. Temperature -The change in kinetic energy of a process ___ as the change in temperature of the process path increases Ans. Increases -The final temperature of an isobaric process if the system work in a _____Quantity and the Surrounding heat transfer to the system Ans. Positive -As the volume of an isothermal expansion process approaches its first value the rate of change in the system pressure. Ans. Decreases -As a fluid flow through a pipe , their potential energy Ans. Decreases -The heat of compression___ as suction vapor becomes super heated Ans. Increases -The heat content of the refrigeration cycle____ when a liquid- suction line heat exchanger is installed Ans. Remains the same -The COP of refrigeration cycle___ with subcooling Ans. Increases -Which defrost method is commonly used small commercial application where the forms on the evaporator surfaces? Ans. Hot gas -Which of the following liquid-chilling evaporator types incorporates overflow heat exchangers? Ans. Baudelot -Which of the following is not a unit of energy? Ans. Watt -The unit of special heat are: Ans. Btu/lbm.oF -Phase change process are constant________process. Ans. None of the above -Which of the following sequences accurately indicates the responses that occur when heat is transferred from a gas? Ans. Sensible heat of vapors, latent heat of vaporization, sensible heat of liquids, latent heat of fusion, sensible heat of fluids -Btu/lbm are units of

Ans. Specific enthalpy -As a liquid changes phase to vapor, its enthalpy Ans. Increases - As a liquid changes phase to a vapor , its entropy Ans. Decreases -The vaporization process that occurs at temperatures below the triple point of a substance is_____ Ans. Sublimation -The Vaporization process that occurs when the vapor pressure of a substance is equal to the atmospheric pressure is: Ans. Boiling -As the pressure of a vapor increases, the amount of work increases and its enthalpy__ Ans. Increases -The entropy of R-134a in a saturated liquid phase at 40 psi is approximately Ans.21 BTU/lb-oF -The latent heat of vaporization of R-134a in its saturated vapor phase at 0 degree C is approximately. Ans.196.7Kj/Kg -The condenser of a commercial display cause that is located within the unit is called Ans. B and C -Which of the following characteristics of early refrigeration systems which applies to today’s units. Ans. Relative expensive -Mechanical refrigeration system make it possible in. Ans. All of the above -_______is method used to reduce the dehydration of sensitive fruits and vegetable during the chilling process Ans. Water chilling -Which of the following is not a step in preparing vegetables for frozen storage? Ans. Coating with syrup -Loss of food juices by osmosis is a consequence of the ______ freezing process. Ans. Immersion -Heat transfer that occurs primarily by conduction is used for ____ freezing process. Ans. All of the above -A disadvantage of sharp freezing is the Ans. All of the above -Frozen storage chamber temperature set points are usually. Ans. -20.5 degree C -Which of the following is not a unit of density? Ans. Lb/ft3 -Wrapping and refrigerating food products extends the storage life of Ans. Meat -The approximate life of strawberries is. Ans. None of the above -Which of the following is not category of refrigeration application? Ans. Transportation air conditioning

-Which of the following is not an industrial application Ans. Meat display cases -Which of the following variable is not most often maintain by a commercial air conditioning unit? Ans. Temperature -Providing clean, filtered air for trouble free operation of equipment and instrumentation is a function of? Ans. Industrial air conditioning process -______ is one of the most common application of mechanical refrigeration. Ans. Preservation of perishable commodities. -The air conditioning systems for cabins on a luxury ocean liner belongs to the ____ category. Ans. Commercial air conditioning Mechanical refrigeration system makes it possible to; Ans. All of the above -As the liquid changes to a vapor. The enthalpy Ans. Increases -Which of the following systems has the most complicated oil return system? Ans. Direct staged -Which of the following process does not after the kinetic energy level of a substance? Ans. Fusion -200 degrees F is equal to Ans. None of the above -An evaporator in a refrigeration unit makes use of which heat transfer mode? Ans. All of the above -When a service technician places his/her hand on a suction line to check the operation of a system, he/she is using which heat transfer modes? Ans. Conduction -Energy added to a vapor is known as Ans. B and C -The rate of changes velocity is called: Ans. Acceleration -Which of the following devices should be used to measure a pressure of 90kpa? Ans. All of the above -The height of the mercury column in a barometer placed on a mountain with a local pressure of 12.5 psi will be? Ans. All of the above. -A compound gauge measuring a pressure of 22.44 inches of mercury is equivalent to an absolute pressure of Ans. 11.0 Psia -The electric meter on a home or building measure the amount of that was consumed over the billing period. Ans. Energy -Which of the following is not a method of food preservation? Ans. Grilling -Which of the following is not plant, animal or fungi? Ans. Enzyme

-Which of the following refrigerant group denote a more toxic vapor? Ans. B3 -What is the molecular weight of helium in 3600 Btu lb K? Ans. 4.0 lb mol -In an isobaric process changes in pressure can be caused by changes in. Ans. None of the above -Boyles law states that pressure and volume changes in gas process is ___related. Ans. Inversely -As a gas is heated in an isothermal gases the volume Ans. Increases -As a gas is heated in an isobaric gases the volume Ans. Decreases -As a gas is heated in an isometric gases the volume Ans. Remains constant -As the volume of a gas decreases, its specific gas constant Ans. Remains constant -Vapor do not behave as ideal gases because they experience Ans. Friction -The unit of latent heat of vaporization are: Ans. None of the above -The unit of latent heat of fusion are: Ans. Btu-lbm -A smaller temperature difference between the refrigerant in the liquid line and that in the evaporator___ the mass flow rate of the refrigerant needed per ton of refrigeration effect. Ans. Decreases -Oil separator are used in system where: Ans. All of the above -Which evaporator design incorporates secondary surface heat transfer? Ans. Finned-tube -Eutectic filled plate evaporator have a higher_____ than evacuated plate evaporators: Ans. All of the above -Which evaporator circuit configuration is less effective when used in expansion load applications. Ans. Cross-flow -Which of the following evaporators feed methods produces the greatest surface without the used of a liquid pump. Ans. Flooded -What type of force convection unit cooler is used in vegetable storage application that require a velocity of approximately 120m/min? Ans. Medium velocity -Which of the following is not a hydrocarbon refrigerants Ans. inane -Which of the following refrigerants is most hazardous? Ans. R-717 -A mixture of two or more refrigerants is called a___

Ans. Zoetrope -Dessicants are used to remove___ from refrigerants Ans. Moisture -As an oil-miscible refrigerant mixes with oil, the viscosity of the oil___ Ans. Decreases -The society that sponsors research on refrigerants is called: Ans. ASHRAE -The scale factor for hard water used in a condenser is: Ans. 0.002 -As the altitude of a forced air-condenser increases, the surface are of the coil or volume flow rateof the fan must: Ans. Increase -As the water temperature entering a water-cooled condenser decrease, the power drawn by the compressor. Ans. Decrease -Which of the does not increase the volumetric efficiency of a compressor? Ans. Decreasing the discharge pressure -What is the compressor operating with a 30 psig suction pressure and a discharge pressure? Ans. 3.13 -As the length of the tubes in a chiller barrel increases, the pressure drop across the inlet and outlet of the barrel: Ans. Increases -_____ decreases the volumetric efficiency of a compressor Ans. Decreasing the suction pressure -The mechanical efficiency of a compressor_____ as the compressor load increases. Ans. Remain the same -The saturated suction temperature used in the selection of a compressor corresponds to the pressure of the vapor at the Ans. Inlet of the compressor -A smaller temperature difference between the refrigerant in the liquid line and that in the evaporator ____ the mass flow rate of the refrigerant needed per ton of refrigerating effect. Ans. Decreases -The difference between the saturation pressure in the evaporator and that in the condenser___ as the suction temperature decreases. Ans. Increases -The COP and the efficiency of a refrigeration cycle____ as the vaporizing temperature increases Ans. Increases -The volume of vapor that the compressor must handle per minute per ton____ as the vaporizing temperature increases. Ans. Decreases -The quantity of heat rejected at the condenser per unit capacity per minute____ as the vaporizing temperature decreases. Ans. Decreases

-As the amount of scale on the refrigerant tubes of an evaporative condenser increases, the scale formation of hot surfaces Ans. Decreases -As the cycles of concentration in a tower or evaporative increases, the scale formation of hot surfaces: Ans. Increases -The____ can only be effectively employed in industrial process that has constant refrigeration load. Ans. Hand expansion valve -The device used to transfer forces across a sealed boundary is called: Ans. Diaphragm -The device used to measure the effects of a large pressure drop across the evaporator is called a Ans. External equalizer -The mass flow rate of refrigerant per unit capacity____ as the condensing pressure decreases. Ans. Decreases -It consists of a tapered glass tube set vertically in the fluid or gaseous piping system with its large end at the top and a metering float free to move vertically in the tube Ans. Rotameter -Also termed as monitor light, it indicates which among a number of normal conditions of a system or device exist. Ans. Pilot light -Which of the following is not a caliper? Ans. Feeler gage -An underground formation that contains sufficient saturated permeable material to yield significant quantities of water. Ans. Aquifer -Chemical used to speed up sewage sedimentation: Ans. Lime -Power expenditure when a current of the one ampere flow across two points having a voltage drop of one volt. Ans. Watt -Prandtl number for air is generally in the order of Ans. 0.70 Relative humidity of air to be used in standard air cooling: Ans. 55%-60% A refrigerating system in which the pressure-imposing element is mechanically operated: Ans. Compression A cushioning device all the end of a trolley, bridge or other moving parts of a crane operating on rails to minimize shock in the event of collision: Ans. Buffer A boom type mobile crane mounted on endless track or treated belts: Ans. Crawler crane

An apparatus for raising or lowering a load by the application of a pulling force but does not include a car or platform riding in guides: Ans. Hoist Most widely used industrial pressure gage applied to both pressure and vacuum: Ans. Bourdon tube gage Hydrocarbons found in liquefied petroleum gas: Ans. All of the above Which of the following is not a solid fuel: Ans. Tar Amount of cooling produced by 2000 lbs of ice in melting over a period of 24 hrs.: Ans. B and C It smoothens the flow due to the nature of flow of the liquid from a reciprocating pump: Ans. Air Chamber Science of force exerted by water in motion: Ans. Hydrodynamics Lowest permissible water level of a boiler without internal furnace: Ans. 1/3 height of shell In pumps, it transmits power from the driver to the impeller: Ans. Shaft Cavitation occurs when the pressure at any point inside a pump drops below the vapor pressure corresponding to the temperature of liquid. Its effect include: Ans. All of the above Positive displacement pump consisting of a fixed casing containing gears, cams, screws, vanes, plungers or similar elements actuated by rotation of the drive shaft. Ans. Rotary pumps Branch of which is larger than the run. Ans. Bull head tee A larger pipe or drum into which each group of boiler is connected: Ans. Header In pipe identification, the color fro pipe used for electricity: Ans. Light orange Has the same equipment as the refrigerating circuits each with a condenser, evaporator, and a pressure imposing elements, where the evaporator of one circuit cools the condenser of the other circuit. Ans. Cascade The temperature of air to be used for comfort cooling shall be in the range of: Ans. 20 deg to 23.3 deg A valve held closed by a spring or other means and designed automatically relieve pressure in excess of its settling. Ans. Pressure relieve device.

A refrigerants which should not be used in public assembly occupancies. Ans. Group 3 refrigerants Fire involving flammable liquids and gases. Ans. Class B fires Mechanics of water or other liquids whether at rest or motion. Ans. Hydraulics Measure the pressure of water discharging from a nozzle by having its open end on the water and the end connected to a manometer. Ans. Pitot Tube A vertical turbine pump with the pump and the motor closed coupled and designed to be installed underground. Ans. Submersible pump Which of the following is a type of deep well pump. Ans. All of the above. Steel pipe coated with zinc to resist corrosion. Ans. Galvanized pipe A fitting with a number of branches in the connecting the smaller pipes. Ans. Manifold The ratio of peak load to the Average load is termed as _____ in variable load nomenclature. Ans. Load factor A heat exchange device used to provide heat transfer between exhaust gases and the air prior to its entrance to the combustor. Ans. Regenerator In a sensible heating process, the moisture content: Ans. Remains constant Flow on both sides on a normal shockwave: Ans. Supersonic on one side, Subsonic on the other A change of phase directly from vapor to solid, without passing through the liquid state: Ans. Deposition What reaction occurs when the enthalpy of the product is less than the enthalpy of the reactants? Ans. Exothermic A well designed engine has a volumetric efficiency within the ranged: Ans. 75%-90% It is referred to as the maximum continuous power available from a hydroelectric power plant even under the most adverse hydraulic condition: Ans. Firm power There are how many feed water lines connected to the boiler? Ans. Two It is the difference in pressure as measured above or below the atmospheric pressure. Ans. Draft

The compression ratio of a gas turbine is in the range of 5-8. Ans. 5 to 8 The average pH of a normal rainfall is generally: Ans. Slightly less than 7 Another term used for liquid valve. Ans. King valve The coefficient of performance of a domestic (local) refrigerator is always: Ans. Greater than one A belt-Coleman cycle is a reversed : Ans. Joule cycle In an air compressor system, the function of a receiver is to: Ans. Collects water and grease suspended in air General layout plan for each floor drawn to scale should not be less than scale of: Ans. 1:200 Highest vapor drop in a vapor compression refrigeration cycle occurs in the: Ans. Expansion valve Which of the following pairs represent the two broad classification of lubricating oils? Ans. Straight and Additives A form of misalignment between the pump and the driver shaft wherein the shafts are concentric but not parallel. Ans. Angular Misalignment In the field of metal corrosion, it is the process wherein it exhibits in the quality deterioration of metals: Ans. Passivation It is the ratio of the density of liquid substance to the density of water at standard conditions: Ans. Specific gravity Term associated with an increase on pressure on a pipe caused by sudden velocity decrease. Ans. Water hammer A process which takes place without change in volume. Ans. Isochoric Weir refers to an opening: Ans. Having partial full flow The Westphal balance is a laboratory instrument used to: Ans. Specific gravity Generally, permissible velocity of water flowing through concrete tunnel is: Ans. 4-5 m/s Refrigerant used in passenger aircraft bins. Ans. Air

It refers to atoms of the same atomic number but differ in atomic masses and molecular weights. Ans. Isotopes Piston rings are normally made of: Ans. Cast Iron Two isothermal and two reversible adiabatic process comprise a: Ans. Carnot cycle In terms of viscosity, density can be expressed as: Ans. Dynamic viscosity/ kinematic viscosity Compressor often used in supercharging Diesel engine: Ans. Root blower type In treating a person whose eyes accidentally got in contact with Freon use: Ans. Sterile mineral oil A device in vapor compression refrigeration system whose primary function is to meter the flow of Refrigerant to the evaporator: Ans. Thermostatic expansion valve From the mathematical perspective, a thermodynamic property is: Ans. A point function In the production of beer, a raw materials called yeast is added in the: Ans. Starting tubs Purpose is to keep the moisture from entering the system. Ans. Dehydrator Medium pressure as applied to valves and fittings implies that a working pressure of _____ is suitable enough. Ans. 862-1200psi A liquid-vapor mixture with a dryness factor is allowed to absorb heat. Which of the following is likely to occur? Ans. Increases Work rooms referring to the maintenance shop and machine rooms shall be _____ in the height from floor to the ceiling? Ans. 3,000mm Defined as a passageway made of sheet metal or other suitable material not necessarily leak tight, used for conveying air or other gases at low temperature. Ans. Duct As a good practical rule, the foundation depth may be taken as _____ times the engine stroke, the lower factor for well-balanced multi-cylinder engine and the higher factor for engines with fewer cylinder s or on less firm soil: Ans. 3.2 to 4.2 What is the suggested maximum permissible dos (MDP) of a gamma ray exposed for individuals now working in a nuclear environment in rem year? Ans. ½

Most commercially petroleum lubricating oil deteriorates starting from temperature of: Ans. 200 F States that the external pressure applied to a confined liquid increases the pressure of every point in the fluid by an amount equal to the external pressure: Ans. Pascal’s Law The average fuel-oil temperature range of the oil in the discharge line to the boilers. Ans. 180-200 F Boiler gage glasses should be blown down: Ans. At the start of every watch The lowest portion of the storage basin where the water is not drawn: Ans. Dead storage Which among the following do not measure relative humidity? Ans. Piezometer Water behind the dam of a hydro electric power plant? Ans. Pondage Converts energy of water to mechanical energy: Ans. Turbine Ratio of average load to the rating of the equipment supplying the load. Ans. Plant factor Ratio of actual maximum demand load to the connected load. Ans. Demand factor A device which preheats feed water by utilizing the heat of the flue gases. Ans. Economizer Study of the proportion of water vapor content of air. Ans. Air conditioning Actual temperature of air. Ans. Dry bulb temperature For a machine foundation with class A mixture what is the proportion of cement, sand and gravel? Ans. 1:2:4 Component included in the proximate analysis of fuel. Ans. Ash, moisture, Volatile matter, fixed carbon The prime mover must have its main steam line: Ans. In loops Intercooler are primarily used with: Ans. Gas compressors In the psychrometric chart, the diagonal lines represents: Ans. Wet bulb temperature Moderator in certain types of nuclear reactors. Ans. Heavy water

The main advantage of turbulous boilers. Ans. Steam pressure can be raised in short time The law which states that the entropy of all perfect crystalline solids is zero at absolute zero temperature. Ans. 3rd law of thermodynamics Operates between two constant temperature reservoir. Ans. Carnot engine Specific measurements of moisture content of air. Ans. Degree of saturation Simultaneous on site generation of electric energy and steam from the same plant output. Ans. Co-generation Instrument used for pressure readings. Ans. Manometer Instrument used to analyze gases. Ans. Orsat Apparatus The chemical formula of R-12 or dichlorofluoromethane. Ans. CCL₂F₂ Which of the following is not desirable property of a refrigerant? Ans. Low thermal conductivity Refrigerant widely used in room air conditions. Ans. R-22 What occurs when pumps are connected in parallel? Ans. Increase discharge, same head Which of the following is not a cause of cavitation? Ans. Low velocity Pipe attached to the penstock to be able to let the water be at atmospheric pressure. Ans. Surge chamber Which of the following is not considered as gaseous fuel? Ans. Bunker Ratio of the density of a liquid to the density of water. Ans. Specific Gravity Measurement of randomness or disorder. Ans. Entropy Energy cannot be created nor destroyed but can only be transformed from one form to another. Ans. First law of thermodynamics In standard Otto cycle when the compression ratio is increased then the thermal efficiency will: Ans. Increase Heat energy produced by the movement of molecule within a substance caused by its temperature. Ans. Internal energy

The law which states that one cannot operate a 100% efficient machine. Ans. 2 nd law of thermodynamics In relation to brake power the instrument used to measure torque: Ans. Dynamometer Ratio of the radiation of an actual body to the radiation of the black body. Ans. Emittance Refrigerant used in ice plant. Ans. Ammonia Measures the pressure of water discharging from the nozzle by having its open end in the water and the other end connected to a manometer. Ans. Pitot tube Defined as a wall designed to prevent the spread of fire having a fir resistance for four hours. Ans. Fire wall The work done per unit charge when the charge is moved from one point to another? Ans. Potential at a point In the relation of PVⁿ is constant, what value makes the process isobaric? Ans. Zero In actual gas behavior, molecular collisions are: Ans. Inelastic An ideal refrigeration should have: Ans. Low freezing point The immediate undesirable products from the petroleum based lubricating oil subjected to high pressure and temperature is referred as: Ans. Carbon Dioxide A rotary dynamic pump, fluid is at no time confined by moving boundaries in its passage through the pump. Ans. Non-positive displacement pump Also known as tangential or Pelton wheel. Ans. Impulse turbine Term given to a chimney made of steel. Ans. Stack An integral part of reaction turbine used to recover energy head. Ans. Draft tube Based on good practice, the vertical distance from the floor soil level to the top edge of the foundation must be around _____ as minimum distance in machiner foundation. Ans. 6 inches The geometric view factor for a black body is always. Ans. One For every 1000ft ascent, there is a corresponding pressure decrease approximately. Ans. 1 in of Hg. Used for impounding water storage and for creating head for the power plant. Ans. Dam

The temperature at which the phase change takes place at a given pressure. Ans. Saturation pressure Distinguishes the system from its surroundings may be at rest or in motion. Ans. Boundary Quality is a measure of: Ans. Dryness The highest pressure under which distinguished liquid vapor phase can exist in equilibrium: Ans. Critical pressure In a thermodynamic gaseous substance, the relationship between P-V-T given by the: Ans. Equation of state A correction factor for gas approaching real gas behavior: Ans. Compressibility factor Furl production process wherein heavy oil is changed into gasoline by means of a high pressure, high temperature and longer exposure time. Ans. Thermal cracking A shut off valve for controlling the flow of refrigerant. Ans. Stop valve Measured by the amount of its pressure below the prevailing atmospheric pressure. Ans. Partial vaccum A relief valve that can positively lift the disc from its seat at least 1.5 mm when there is no pressure in the boiler. Ans. Water relief valve Vapor produced in the vaporization of a small amount of liquid refrigerant downstream of the expansion valve. Ans. Flash gas Product of the rate of evaporation and the factor of evaporation. Ans. Equivalent evaporation Difference between the indicated power and the brake power. Ans. Friction power In Diesel power plant, its purpose is to reduce the weight-to-power ratio: Ans. Supercharging Change of phase from solid to gas. Ans. Sublimation The memory lost when operating power is removed. Ans. Volatile memory A refrigeration system using direct method in which refrigerant is delivered to two or more evaporators in space room or in refrigerators. Ans. Multi-pressure

The carbon dioxide concentration of air in all rooms when measured 910mm above the floor space not exceed: Ans. 100ppm Consists of helmets shall be removed immediately after having been used or the seal broken, must be removed atleast every: Ans. 2 years Pump whose purpose is to increase the effective water pressure by sucking water from public service main or private use water system: Ans. Booster pump A small fitting with a double offset, or shaped like the letter C with the ends turned out. Ans. Cross-cover Material interposed between two relatively moving machine elements to reduce wear and tear: Ans. Lubricant Measure temperature by electromotive force: Ans. Thermocouple According to SAE which of the following is a type of lubricating oil? Ans. All of the above A device that information in one form of an instrument signal and transmits an output signal on another form: Ans. Digital A device whose function to pass on information in an unchanged form or in some modified term. Ans. Switch A device that receives information on one form of one or more physical quantities modifies the information and/or its form and produces a resultant output signal. Ans. Transducer Which of the following is not a requirement in designing pipe installations? Ans. Galvanized pipe should be used for steam Part of a valve used to guide and support valve stem: Ans. Bonnet In piping installations, the color of pipe tube used for water is: Ans. Green The color code for pipes used in communication. Ans. White Which of the method is used for classifying coal. Ans. All of the above Alcohol frequently considered as fuel for internal combustion engine. Ans. Ethyl alcohol Measure of resistance to flow. Ans. Viscosity Converts Solar energy to electrical energy. Ans. Photovoltaic cell

Which of the following is not an instrument used to measure flow rates? Ans. Velometer Refrigeration follows what cycle? Ans. Reversed Carnot cycle An evaporator constructed on pipe tubing. Ans. Expansion coil Reinstalled or second hand boilers shall have a minimum factor of safety of? Ans. 6 A line that shows the rotation of the consumption and the load a steam turbine generator. Ans. Willans Line Instrument that indicate the percentage of carbon dioxide in flue gases in a power plant. Ans. Ranarex indicator For real process, the net entropy change in the universe is: Ans. Positive The sum of the internal energy and the product of the pressure and specific volume. Ans. Enthalpy Ratio of the maximum demand of the system to the rated capacity of the system. Ans. Utilization factor Source of thermal energy. Ans. All of the above Cycle that has two isentropic and two isometric process. Ans. Otto cycle Cycle that has two isentropic and two isobaric process. Ans. Brayton cycle # Two kilogram of gas is confined in a 1 m3 tank at 200 kpa and 88°C. What type of gas is in the tank? Solution: PV=mRT 200(1) = 2(8.314/M) (88+273) M = 30 Therefore: gas is Ethane # Find the enthalpy of helium if its internal energy is 200 KJ/kg. Solution: R = 8.314/4 = 2.0785 K = 1.667 for helium Cp = k R/(k-1) = 2.0785/(1.667 – 1) = 3.116 KJ/kg-K h/ = Cp/Cv /200 = 5.195/3.116 = ______________ANS # Compute the mass of a 2 m3 propane at 280 kpa and 40°C. Solution:

Propane is C3 H8 --------- M = 12(3) + 8(1) = 44 PV = mRT 280(2) = m(8.314/44)(40+273) m = ____________ ANS # A thermal power plant has a heat rate of a 11,363 Btu/kw-hr. Find the thermal efficiency of the plant. Solutionj: eth = 3412/Heat rate = 3412/11,363 = _________ANS. # What is the hydraulic gradient of a 1 mile, 17 inches inside diameter pipe when 3300 gal/min of water flow with f = 0.03. Solution: V = (3300/7.481)/(π/4)(17/12)2(60) = 4.66 ft/s L = 1 mile = 5280 ft hL = fLv2/2gD = 0.03(5280)(4.66)2/2(32.2)(17/12) = 37.7 ft Hydraulic gradient = 37.7/5280 = ___________ANS. # Find the loss of head in the pipe entrance if speed of flow is 10 m/s. Solution: Loss at entrance = 0.5(v2/2g) = 0.5[102 / 2(9.81)] = ___________ANS. Wet material, containing 220% moisture (dry basis) is to be dried at the rate of 1.5 kg/s in a continuous dryer to give a product containing 10% (dry basis). Find the moisture removed, kg/hr. Solution: Solid in wet feed = solid in dried product [1/(1 + 2.2)](1.5) = [1/(1 + 0.1)](x) x = 0,5156 kg/s (total dried product) Moisture removed = 1.5 – 0.5156 = 0.984 kg/s = ___________ANS. # Copra enters a dryer contacting 70% moisture and leaves at 7% moisture. Find the moisture removed on each pound of solid in final product. Solution: Solid in wet feed = solid in dried product 0.3x = 1 x = 3.333 lbs 1= 0.93y Y = 1.07527 lb Moisture removed = x – y = 3.333 – 1.07527 = _________ ANS. # A 1 m x 1.5 m cylindrical tank is full of oil with SG = 0.92. Find the force acting at the bottom of the tank in dynes. Solution: P = w h = (0.92 x 9.81) (1.5) = 13.5378 kpa F = PA = 13.5378(π/4 x 12) = 10,632 KN = 10,632.56 N x 10,000 dynes/N

F = ____________ANS. # Find the pressure at the 100 fathom depth of water in kpag Solution: H = 100 fathom x 6 = 600 ft P = w h = (600/3,281)(9.81) = ______________ANS. # Find the depth in furlong of the ocean (SG = 1.03) if the pressure at the sea bed is 2,032.56 kpag. Solution: P=wh 2,032.56 = (1.03 x 9.81) h H = 201.158 m x 3.281ft/m x 1 yd/3ft x 1 furlong/220 yd = 1 furlong # Find the mass of 10 quartz of water. Solution: V = 10 quartz x 1gal/4quartz x 3.785li/gal x 1m3/1000li V= 0.0094625 x 10-3 m3 W = m/V 1000 = m/0/0094625 x 10-3 m = __________ ANS. # Find the mass of carbon dioxide having a pressure of 20 psia at 200°F with 10 ft3 volume. Solution: PV = m R T (20 x 144)(10) = m (1545/44)(200 + 460) m = ___________ANS. # Find the heat needed to raise the temperature of water from 30°C to 100°C with 60% quality. Consider and atmospheric pressure of 101.325 kpa. Use the approximate enthalpy formula of liquid. Solution: At 100°C hf = Cp t = 4.187(100) 418.7 KJ/kg hfg = 2257 KJ.kg h2 = hf + xhfg = 418.7 + 0.60(2257) = 1,772.9 KJ/kg Q = 1(4.187)(100-30) + 1(1772.9 – 418.7) = _________ANS. # Find the enthalpy of water at 212°F and 14.7 psi if the dryness factor is 30%. Use the approximate enthalpy formula of liquid. Solution: hf = (°F – 32) = (212 – 32) = 180 Btu/lb hfg = 970 Btu/lb h = hf + xhfg h = 180 + 0.3(970) = ________ANS.

# An air compressor consumed 1200 kw-hr per day of energy. The electric motor driving the compressor has an efficiency of 80%. If indicated power of the compressor is 34 kw, find the mechanical efficiency of the compressor. Solution: Pim = 1200kw-hr/24 hrs = 50 kw BP = 50(0.80) = 40 kw em = 34/40 = _______ANS. # A refrigeration system consumed 28,800 kw-hr per month of energy. There are 20% of energy is lost due to cooling system of compressor and motor efficiency is 90%. If COP of the sustem is 6, find the tons of refrigeration of the system. Solution: Pim = 28,800/(24 x 30) = 40 kw BP = 40 (0.90) = 36 kw Wc = 36(1-0.20) = 28.80 kw COP = RE/Wc 6 = RE/28.80 RE = 172.8/3.516 = ______________ANS. # A 23 tons refrigeration system has a heat rejected of 100 kw. Find the energy efficiency ratio of the system. Solution: QR = RE + Wc 100 = 23(3.516) + Wc Wc = 19.132 kw COP = RE/Wc = (23 x 3.516) / 19.132 = 4.23 EER = 3.412 COP = 3.412(4.23) = ____________ANS. # A 200 mm x 250 mm, 8-cylinder, 4-stroke diesel engine has a break power of 150 kw. The mechanical efficiency is 80%. If two of the cylinders were accidentally cut off, what will be the new friction power? Solution: em = BP/IP 0.8 = 150/IP IP = 187.5 kw FP1 = IP – BP = 187.5 – 150 = 37.50 kw FP1 = FP2 = ____________ANS. # If the energy efficiency ratio of the refrigeration system is 12.6, what is the COP of the system? Solution: EER = 3.412 COP 12.6 = 3.412 COP COP = ____________ANS.

# An air compressor has a power of 40 kw at 4% clearance. If clearance will increase to 7%, what is the new power? Solution: The power of compressor will not be affected with the changes in clearance. Therefore the power will still be 40 kw. # What is the approximate value of temperature of water having enthalpy of 208 Btu/lb? Solution: h = °F – 32 208 = °F -32 °F = _________ANS. # An Otto cycle has a compression ratio of 8. Find the pressure ration during compression. Solution: P1V1k = P2V2k (V1/V2)k = (P2/P1) rK = rp rp = (8)1.4 = ___________ANS. # A diesel cycle has a cut off ratio of 2.5 and expansion ratio of 4. Find the clearance of the cycle. # Determine the atmospheric pressure at a location where barometric reading is 740 mm Hg and gravitational acceleration is g = 9.7 m/s2. Assume the temperature of mercury to be 10°C, at which the density is 13,570 kg/m3. Solution: P = (wg)h = (13,570 x 9.7)(9.74) (1Kpa/1000 N/m2) = ___________ANS. # The barometer of a mountain hiker reads 930 mbars at the beginning of a hiking trip and 780 mbars at the end. Neglecting the effect of altitude on local gravitational acceleration, determine the vertical distance climb. Assume g = 9.7 m/s 2. Solution: P1 – P2 = w h (0.93 – 0.78)(100 kpa/bar) = (1.2 x 0.00981) h h = 1274.21 m h = 1274.21 (9.81/9.7) = ___________ANS. # The lower half of a 10 m high cylindrical container is filled with water and the upper half with oil that has SG = 8.85. Determine the pressure difference between the top and bottom of the cylinder. Solution: P = Pw + Po = 9.81(5) + (0.85 x 9.81)(15) = ________ANS. # An ideal gas at 0.80 atmospheres and 87°C occupies 0.450 liter. How many moles are in the sample? (R = 0.0821 liter-atm/mole – K)

Solution: PV = nRT (0.80 atm)(0.450 li) = n (0.0821 )(87+273)K n = ___________ANS. # A certain gas at 101.325 Kpa and 10°C whose volume is 2.83 m 3 are compressed into a storage vessel of 0.31 m3 capacity. Before admission, the storage vessel contained the gas at a pressure and temperature of 173.8 Kpa and 26°C, after admission the pressure has increased to 1171.8 Kpa. What should be the final temperature of the gas in the vessel in Kelvin? Solution: Solving for the mass of gas which is to be compressed: PV = mRT 101.325(2.83) = m1R(10 + 273) m1 = 1.01325/R Solving for the mass of gas initially contained in the vessel: PV = mRT 137.8(0.31) = m2R(26 + 273) m2 = 0.14286/R Solving for the final temperature: m3 = m1 + m2 m3 = 1.01325/R + 0.14286/R = 1.156/R 1171.8(0.31) = (1.156/R)RT3 T3 = ____________ANS. # A perfect gas has a value of R = 58.8 ft-lb/lb-R and k = 1.23. if 20 Btu are added to 10 lbs of this gas at constant volume when initial temperature is 90°F, find the final temperature. Solution Q = m cv (t2 – t1) cv = R / (k -1) = = 0.29086 Btu/lb-F 20 = 10(0.29086)(t2 – 90) t2 = ___________ANS. # Ammonia weighing 22kgs is confirmed inside a cylinder equipped with a piston has an initial pressure of 413KPa at 38C. If 3200KJ of heat is added to the ammonia until its final pressure and temperature are 413KPa and 100oC respectively, what is the amount of work done by the fluid in KJ? SOLUTION Since the molecular weight of ammonia is 17. Then R = 8.3143/M = 8.3143/17 = 0.489 KJ/kgoK P1V1 = mRT1 413V1 = 22(0.489)(38 + 273) V1 = 8.101 m3 P2V2 = mRT2 413V2 = 22(0.489)(100 + 273)

V2 = 9.716 m3 W = P(V1-V2) = 413(9.716 – 8.101) = ANS. # A tank contains 90ft3 of air at a pressure of 350 psig; if the air is cooled until its pressure and temperature decreases to 200 psig and 70oF respectively. What is the decrease in internal energy? SOLUTION m = PV/RT = (200 + 14.7)(90)(144)/(53.342)(70 + 460) = 98.50 lbs for constant volume process: P1/T1 = P2/T2 T2 = 70 + 460 = 530oR = T1 = 900oR ΔU = mcv(T2 – T1) = 98.50(0.171)(530 – 900) = ANS. # A large mining company was provided with a 3m3 of compressed air tank. Air pressure in the tank drops from 700KPa while the temperature remains constant at 28oC. What percentage has the mass of air in the tank been reduced? SOLUTION Percent mass reduced = (700 – 150)/700 = ANS. # A 4m3/hr pump delivers water to a pressure tank. At the start, the gauge reads 138KPa until it reads 276KPa and then the pump was shut off. The volume of the tank is 180liters. At 276KPa the water occupied 2/3 of the tank volume. Determine the volume of the water that can be taken out until the gauge reads 138KPa. SOLUTION Consider the air pressure: V2 = 1/3 (180) = 60liters P1V1 = P2V2 (138 + 101.325)(V1) = (276 + 101.325)(60) V1 = 94.59 liters Amount of water to be removed = 2/3 (180) – (180 – 94.59) = ANS. # A refrigeration plant is rated at 15tons capacity. How many pounds of air per hour will it cool from 70 to 90oF at constant pressure. SOLUTION Tons of refrigeration = mcp(t2 – t1)/12000 m= ANS. # An air standard engine has a compression ratio of 18 and a cut – off ratio of 4. If the intake pressure and temperature are 100KPa and 27oC, find the work in KJ per kg. SOLUTION T1 = 27 + 273 = 300oK Process 1 to 2 is isentropic procees: T2 = T1(V1/V2)k-1 = 300(18)1.4-1 = 953.301oK Process 2 to 3 is constant pressure process: T3/T2 = V2/V3 = rc T3 = 953.301 (4) = 3813.205oK QA = mcp(T3 – T2) = 1(3813.205 -953.301) = 2860 KJ/kg

W = (.5531)(2860) = ANS. # Determine the air – standard efficiency of an engine operating on the diesel cycle with clearance of 6% when the suction pressure is 99.97KPa and the fuel injected for 7% of the stroke. Assume k = 1.4. SOLUTION V3 – V2 = 0.07VD V2 = 0.06VD V3 - 0.06VD = 0.07VD V3 = 0.13VD rc = V3/ V2 = 0.13VD/0.06VD = 2.167 rk = (1 + 0.06)/0.06 = 17.667 = ANS. # Steam at 2MPa and 250oC in a rigid cylinder is cooled until the quality is 30%. Find the heat rejected from the cylinder. At 2MPa and 250oC: v = 0.11144m3/kg u = 2679.6 KJ/kg At 2MPa, (saturated): vf = 0.0011767m3/kg, vg = 0.09963m3/kg uf = 906.44 KJ/kg, ufg = 1693.8KJ/kg SOLUTION Q = U2 – U1 U1 = 2679.6 KJ/kg U2 = Uf + xUfg = 906.44 + 0.30(1693.8) = 1414.58 KJ/kg Q = 1414.58 – 2679.6 = ANS. # At 1.3MPa, mixture steam and water has an entropy of 3KJ/kg - oK. Find the enthalpy of the mixture. At 1.3MPa: sf = 2.2515, sg = 6.4953, hf = 814.93, hfg = 1972.7 SOLUTION s = sf + xsfg 3 = 2.2515 + x(6.4953) x = 0.17614 h = hf + xhfg = 814.93 + 0.17614(1972.7) = ANS. # Mixture with 70% quality at 500KPa is heated isothermally until its pressure is 300KPa. Find the heat added during the process. At 500KPa: sf = 1.8607, sfg = 4.9606 o At 300KPa and 151.86 C s = 7.088 KJ/kg SOLUTION s1 = sf + xsfg = 1.8607 + 0.70(4.9606) = 5.333 KJ/kg s2 = 7.088 KJ/kg Q = T(s2 - s1) = (151.86 + 273)(7.088 – 5.333) = ANS. # A tank contains exactly one kilogram of water consisting of liquid and vapor equilibrium at 1MPa. If the liquid contains one – third and the remaining is vapor of the volume of the tank, what is the enthalpy of the contents of the tank? At 1MPa: vf = 0.0011273 vfg = 0.19444 hf = 762.81 hfg = 2015.3 SOLUTION Let V = total volume of the tank

T2 = T1(V1/V2)k-1 = 300(18)1.4 – 1 = 953.301oK Process 2 to 3 is constant process: T3 = 953.301(4) = 3813.205oK QA = mcp(T3 – T2) = 1(3813.205 - 953.301) = 2850 KJ/kg W = (0.5531)( 2850) = ANS. # Determine the air standard efficiency of an engine operating on the diesel cycle with clearance of 6% when the suction pressure is 99.97KPa and the fuel is injected for 7% of the stroke. Assume k = 1.4. SOLUTION V3 - V2 = 0.07VD V2 = 0.06VD V3 = 0.07VD + 0.06VD V3 = 0.13VD rc = V3/ V2 = 0.13VD /0.06VD = 2.167 rk = (1 + 0.06)/(0.060) = 17.667 e= = ANS. # A steam at 2MPa and 250oC in a right rigid cylinder is cooled until the quality is 30%. Find the heat rejected from the cylinder. At 2MPa and 250oC: v = 0.11144 m3/kg u = 2679.6 KJ/kg At 2MPa, (saturated): vf = 0.0011767 m3/kg, vg = 0.09963 3 m /kg uf = 906.44 KJ/kg ufg = 1693.8 KJ/kg SOLUTION Q = (u2 – u1) u1 = 2679.6KJ/kg u2 = uf + x ufg = 906.44 + 0.3(1693.8) = 1414.58KJ/kg Q = (1414.58 – 2679.6) = ANS # At 1.3MPa, mixture steam and water has an entropy of 3KJ/kg-oK. Find the enthalpy of the mixture. At 1.3MPa: sf = 2.2515, sg = 6.4953, hf = 814.93, hfg = 1972.7 SOLUTION s = sf + x(sg – sf) 3 = 2.2515 + x(6.4953 – 2.2515) x = 0.17637 h = hf + xhfg = 814.93 + 0.17637(1972.7) = ANS. # Mixture with 70% quality at 500KPa is heated isothermally until its pressure is 300KPa. Find the heat added during the process. At 500KPa: sf = 1.8607, sfg = 4.9606 At 300KPa and 151.86oC: s = 7.0888 SOLUTION For isothermal process, t1 = t2 s1 = sf + xsfg = 1.8607 + 0.7(4.9606) = 5.333 s2 = 7.0888 Q = T(s2 – s1) = (151.86 + 273)(7.0888 – 5.333) = ANS.

# A tank contains exactly one kilogram of water consisting of liquid and vapor in equilibrium at 1MPa. If the liquid contains one third and the remaining is vapor of the volume of the tank, what is the enthalpy of the contents of the tank? At 1MPa: vf = 0.0011273 vfg = 0.19444 hf = 762.81 hfg = 2015.3 SOLUTION Let V = total volume of tank mL = VL/vL = (V/3) / 0.0011273 = 295.69 V mv = Vv / vv = (2V/3) / 0.1944 = 3.429V x= = ANS. # Water substance at 70 bar and 65oC enters a boiler tube of constant inside diameter of 35mm. The water leaves the boiler tube at 50 bar and 700oK at velocity of 100m/s. Calculate the inlet volume flow(li/sec). SOLUTION From steam tables: At 70bar(7MPa) and 65oC v1 = 0.001017m3/kg at 50bar(5MPa) and 700oK(427oC) v2 = 0.06081m3/kg m1 = m2 Q1/ v1 = Q2/ v2 V1 = 1.672m/sec Q1 = A x v = (π/4)(0.035)2(1.672) = ANS. # Steam leaves an industrial boiler at 827.4KPa and 171.6 oC. A portion of the steam is passed through a throttling calorimeter and is exhausted to the atmosphere when the calorimeter pressure is 101.4KPa. How much moisture does the steam leaving the boiler contain if the temperature of the steam at the calorimeter is 115.6 oC? At 827KPa(171.6oC): hf = 727.25 KJ/kg, hfg = 2043.2 KJ/kg From table 3: At 101.4KPa and 115.6oC: h2 = 2707.6 KJ/kg SOLUTION Let x = quality of steam entering the throttling calorimeter h1 = h 2 hfg + xhfg = h2 727.25 + x(2043.2) = 2707.6 x = 0.9692; y = 1 - 0.9692 = ANS. # A throttling calorimeter is connected to the desuperheated steam line supplying steam to the auxiliary feed pump on a ship. The line pressure measures 2.5MPa. The calorimeter pressure is 110KPa and 150oC. Determine the entropy of the steam line. At 110KPa and 150.C: h2 = 2775.6KJ.kg At 2.5MPa: hf = 962.11KJ/kg, hfg = 1841 KJ/kg, sf = 2.5547, sfg = 3.7028 SOLUTION For throttling process: (h1 = h2) h1 = h2 = hf + x hfg 2775.6 = 962.11 + x(1841) x = 98.5%

s1 = sf + x sfg = 2.5447 + 0.985(3.7028) = ANS. o # Atmospheric pressure boils at 212 F. At the vacuum pressure at 24 in Hg, the temperature is 142oF. Find the boiling temperature when the pressure is increased by 40psia from the atmospheric. SOLUTION P2 = 14.7 + 40 = 54.7 psia P1 = -24(14.7/29.92) + 14.7 = 2.908 psia By interpolation: t2 = #

ANS. A certain coal has the following ultimate analysis: C = 69% N2 = 5% H2 = 2.5% S = 7% Determine the amount of oxygen of the heating value of fuel is 26 961.45 KJ/kg. SOLUTION Qh = 33 820C + 144 212 (H - ) + 9304S 26961.45 = 33820(.69) + 144212 (0.025 – O/8) + 9304(0.07) O= ANS. # A diesel engine consumed 945 liters of fuel per day at 35oC. If the fuel was purchased at 15.5oC and 30oAPI at P29.00/li, determine the cost of fuel to operate the engine per day. SOLUTION 16

Cost = P29.00/li(935.44li) = ANS. # A cylindrical tank 4m long and 3m diameter is used for oil storage. How many days can the tank supply the engine having 27oAPI with fuel consumption of 60kg.hr? SOLUTION V = π/4 D2 h = π/4 (3)2(4) = 28.274 m3 Density of fuel = 0.89274(1000kg/m3) = 892.74kg/m3 w = m/V V = 60/892.74 = 0.0672 m3/hr Number of days = 28.274/0.0672 = 420.75hrs = ANS. # A logging firm in Isabella operates a Diesel Electric Plant to supply its electric energy requirements. During a 24 period, the plant consumed 250 gallons of fuel at 80oF and produced 2900KW-hrs. Industrial fuel used is 30oAPI and was purchased at P30.00/li at 60oF. Determine the overall thermal efficiency of the plant. SOLUTION Qh = 41130 + 139.6xoAPI = 41130 + 139.6(30) = 45318 KJ/kg 60oF = 15.6oC 80oF = 26.6oC

At 26.6oC mf = 250gal/24hrs x 3.785li/gal x 0.869kg/li x 1hr/3600sec mf = 0.00952 kg/sec load = 2900/24 = 120.833KW overall efficiency = = ANS. # The dry exhaust gas from oil engine has the following gravimetric analysis: CO2 = 21.6% O2 = 4.2% N2 = 74.2% Specific heats at constant pressure for each component of the exhaust gas in Kcal/kgoC are: CO2 = 0.203 O2 = 0.219 N2 = 0.248 Calculate the specific gravity if the molecular weight of air is 28.97 kg/kg-mol. SOULTION Converting the gravimetric analysis to volumetric: CO2 = 0.219/44 = 0.004909 O2 of nitrogen in lb/= 0.042/32 = 0.001312 N2 = 0.742/28 = 0.026500 0.032721 mols/kg-mol Molecular weight = 1/0.032721 = 30.56kg/kg-mol SG = 30.56/28.97 = ANS. # A bituminous coal has the following composition: C = 71.5% H = 5.0% O = 7.0% N = 1.3% S = 3% Ash = 7.6% W = 3.4% Determine the theoretical weight of nitrogen in lb/lb of coal. SOLUTION Theo. A/F = 11.5C + 34.5(H – O/8) + 4.3S = 11.5(0.715) + 34.5 (0.05 – 0.07/8) + 4.3(0.03) = 9.8 lb air / lb coal N2 in air by weight = 76.8% therefore: theoretical weight of N2 = 0.768(9.8) = ANS. # A gaseous fuel mixture has a molal analysis: H2 = 14% CH4 = 3% CO = 27% O2 = 0.6% CO2 = 4.5% N2 = 50.9% Determine the air fuel ratio for complete combustion of molal basis. SOLUTION Chemical reaction with oxygen: 0.14H2 + 0.070O2 = 0.14H2O 0.03CH4 + 0.060O2 = 0.03CO2 + 0.06H2O 0.27Co + 0.135O2 = 0.27CO2 0.265O2 Actual O2 in product = 0.265 O2 – 0.006 O2 = 0.259 O2 Molal A/F = 0.259 + 0.259(3.76) = ANS. # A volumetric analysis of a gas mixture is as follows: CO2: 12% N2: 80% O2: 4% CO: 4%

What is the percentage of CO2 on a mass basis? SOLUTION Converting to mass basis: CO2 = 0.12 x 44 = 5.28 O2 = 0.014x32 = 1.28 N2 = 0.82x28 = 22.96 CO = 0.02x28 = 0.56 Total mass of product = 5.28 + 1.28 + 22.96 + 0.56 = 30.08 kg % mass of CO2 = 5.28/30.08 = ANS. # The following coal has the following ultimate analysis by weight: C = 70.5% H2 = 4.5% O2 = 6.0% N2 = 1.0% S = 3.0% ash = 11% moisture = 4% A stocker fired boiler of 195000kg/hr steaming capacity uses this coal as fuel. Calculate volume of air in m3/kg with air at 60oF and 14.7 psia pressure of boiler efficiency is 70% and FE = 1.10. SOLUTION Theo. A/F = 11.5C + 34.5(H – O/8) + 4.3S = 11.5(0.705) + 34.5(0.045 – 0.06/8) + 4.3(0.03) = 9.53 Actual A/F = 9.53(1.3) = 12.389 kg air/kg fuel

; mf = 23107.56 (12.389) = 286279.57 kg/hr 101.325(V) = 286279.57(0.287)(15.6 + 273) V= ANS. # 23.5 kg of steam per second at 5MPa and 400oC is produced by a steam generator. The feedwater enters the economizer at 145 oC and leaves at 205oC. The steam leaves the boiler drum with a quality of 98%. The unit consumes 3kg of coal per second as received having a heating value of 25102 KJ/kg. What would be the overall efficiency of the unit in percent? Steam properties: At 5MPa and 400oC: h = 3195.7KJ/kg At 5MPa: hf = 1154.23, hg =1640.1 At 205oC: hf = 875.04 At 145oC: hf = 610.63 SOLUTION = ANS. # In a Rankine cycle steam enters the turbine at 2.5MPa (enthalpies and entropies given) and condenser of 50KPa (properties given), what is the thermal efficiency of the cycle? At 2.5MPa: hg = 2803.1 sg = 6.2575 At 50KPa: sf = 1.0910 sfg = 6.5029 hf = 340.49 hfg = 2305.4 vf = 0.0010300 SOLUTION

h1 = 2803.1 KJ/kg solving for h2: s = sf + xsfg 6.2575 = 1.0910 + x(6.5029) x = 0.7945 h2 = hf + xhfg = 340.49 + 0.7945(2305.4) = 2172.13 KJ/kg h3 = 340.49 KJ/kg h4 = hf + vf (P2 – P1) = 340.49 + 0.00103(2500 - 50) = 342.98 KJ/kg Efficiency =

ANS.

# A two-stage air compressor air at 100 Kpa and 22⁰C discharges to 750Kpa. If intercooler intake is 105⁰C, determine the value of n. Solution: Px = = 273.86 Kpa

1.281 = n = _____________ ANS. # A single acting air compressor has a volumetric efficiency of 89%, operates at 500 rpm. It takes in air at 100kpa and 30⁰C and discharges it at 600 Kpa. The air handled is 8 m3/min measured at discharge condition. If compression is isentropic, find mean effective pressure in Kpa SOLUTION: P1V1k = P2V2k 100(V11.4) = 600(8)1.4 V1 = 28.37687 m3/min VD = 28.768/0.89 = 32.32 m3/min W= = W = Pm x V D 7562.19 = Pm x 32.32 Pm = ________ ANS. # A water-jacketed air compressor handles 0.343 m3/s of air at 96.5 kpa and 21⁰C and leaving at 480 kpa and 132⁰C; 10.9 kg/h of cooling water enters the jacket at 15⁰C and leaves at 21⁰C. Determine the compressor break power. SOLUTION:

n = 1.249 W = = 62.57 KW Q = heat loss = mCp(t2 – t1) = (10.9/3600)(4.187)(21 – 15) = 0.076 KW Brake power = W + Q = 62.57 + 0.076 = _______ ANS. # A double suction centrifugal pump delivers 20 ft3/sec of water at a head of 12 m and running at 650 rpm. What is the specific speed of the pump? SOLUTION: Ns =

Q= h = 12 x 3.281 = 39.37 ft Ns = __________ANS. # Determine the number of stages needed for a centrifugal pump if it is used to deliver 400 gal/min of water and pump power of 15 Hp. Each impeller develops a head of 30 ft. SOLUTION: Wp = w Q h 15 x 0.746 = 9.81(400 gal/min x 0.003785 m3/gal x 1/60) h h =45.20 m x 3.281 ft/m = 148.317 ft Number of stages = 148.317/30 = 4.94 stages # The suction pressure of a pump reads 3 in. of mercury vacuum and discharge pressure reads 140psi is use to deliver 120 gpm of water with specific volume of 0.0163 ft3/lb. Determine the pump work. SOLUTION: P1 = -3 in Hg x 101.325/29.92 = -10.15 Kpa P2 = 140 psi x 101.325/14.7 = 965 Kpa W = 1/v = 1/.0163 = 61.35 lb/ft3 x 9.81/62.4 = 9.645 KN/m3 h = = = 101.105 m Q = 120 gal/min x 3.785 li/1gal x 1m3/1000li x 1/60 = 0.00757 m 3/sec P = w Q h = 9.645(0.00757)(101.105) = ____________ ANS. # A submersible pump delivers 350 gpm of water to a height of 5 ft from the ground. The pump was installed 150 ft below the ground level and a draw down of 8 ft during the operation. If water level is 25 ft above the pump, determine the pump power. SOLUTION: h = 5+ 150 – (25 – 8) = 138/3.281 = 42.06 m Q = 350 gal/min x 0.003785 m3/gal x 1min/60sec = 0.02246 m3/sec Wp = w Q h = 9.819.02246)(42.06) = __________ ANS. # A vacuum pump is used to drain a flooded mine shaft of 20⁰C water. The pump pressure of water at this temperature is 2.34 Kpa. The pump is incapable of lifting the water higher than 16 m. What is the atmospheric pressure? SOLUTION:

P1 = ___________ ANS. # A submersible, multi-stage, centrifugal deep well pump 260 gpm capacity is installed in a well 27 feet below the static water level and running at 3000 rpm. Drawdown when pumping at rated capacity is 10 ft. the pump delivers the water into a 25,000 gallons capacity overhead storage tank. Total discharge head developed by pump, including friction in piping is 243 ft. Calculate the diameter of the impeller of this pump in inches if each impeller diameter developed a head of 38 ft. SOLUTION: V=πDN V= π D (3000/60) = D = 0.315 ft = _________ ANS.

# A fan draws 1.42 m3/sec of air at a static pressure of 2.54 cm of water through a duct 300 mm diameter and discharges it through a duct of 275 mm diameter. Determine the static fan efficiency if total fan mechanical is 75% and air is measured at 25⁰C and 760 mmHg. SOLUTION: wa = # A total head of fan is 187 m and has a static pressure of 210 mm of water gage, what is the velocity of air flowing if density of air is 1.15 kg/m3? SOLUTION: hs = 0.21(1000-1.15) = 182.61 m h = hs + hv 187 = 182.61 + hv hv = 4.39 m 4.39 = v2 /2(9.81) v = _________ANS. # A fan delivers 5.7 m3/sec at a static pressure of 5.08 cm of water when operating at a speed of 400 rpm. The power input required is 2.963 KW. If 7.05 m 3/sec are desired in the same fan and installation, find the pressure in cm of water. SOLUTION:

N2 = 494.74 rpm

h2 = __________ANS. # A rigid container is closed at one end and measures 8 in diameter by 12 in long. The container is held vertically and is slowly moved downward until the pressure in the container is 17 psia. What will be the depth of the top of the container from the free water surface? SOLUTION: Patm = Pgage + Patm 17 = Pgage + 14.7 Pgage = 2.30 psi Pgage = w h 2.30(144) = 62.4 (h) h = 5.3077ft x 12 =__________ans. # An empty, open can is 30 cm high with a 15 cm diameter. The can, with the open end down, is pushed under water with a density of 1000 kg/m 3. Find the water level in the can when the top of the can is 50 cm below the surface. SOLUTION: Consider the water pressure Pw = wh + 101.325 = (0.8 - x)(9.81) + 101.325 = 109.173 – 9.81x Consider the air pressure: P1V1 = P2V2 101.325(A x 0.3) = P2[A(0.3 – x)] P2 =

Pw = P 2 109.173 – 9.81x = 9.81x2 – 112.116x + 2.3705 = 0 By quadratic formula: X = 0.02118 m = _____________ANS # A cylindrical pipe with water flowing downward at 0.03 m3/s having top diameter of 0.08, bottom diameter of 0.04 m and height of 1.5 m. Find the pressure between the pipe. SOLUTION: Z1 – Z2 = 1.5 m Z2 – Z1 = -1.5 m V1 = V2 = + (-1.5) P1 – P2 = __________ANS. # Determine the size of pipe which will deliver 8 liters of medium oil (v = 6.10 x 10-6 m2/s) assuming laminar flow conditions. V= Re = For laminar flow, Re = 2000 2000 = d = 0.835 m = ________ANS. # The type of flow occupying in a 1 cm diameter pipe which water flows at a velocity of 2.50 m/s. Use v=1.13x10-6 m2/s for water. SOLUTION: Re =dV/ v Re = Since it is greater than 2000, then it is _________ANS # An insulated rigid tank initially contains 1.5 lb of helium at 80⁰F and 50 psia. Apaddle wheel with power rating of 0.02 hp is operated within the tank for 30 min. Determine the final temperature. SOLUTION: W= ΔU = mcv(T2 – T1) 0.02hp (0.50 hr)(2545 Btu/hr/hp) = 1.5 (0.171)(T2 – 80) T2 = __________ANS. # A 4 m2 asphalt pavement with emissivity of 0.85 has a surface temperature of 50⁰C. Find the maximum rate of radiation that can be emmited from the surface. SOLUTION: Qr = e ksvA Ksv = 5.67 x 10-8 (Stefan Boltzman constant) Qr = 0.85(5.67 x 108)(4)(50 +273)4 = ____________ANS. # Air at 10⁰C and 80 kpa enters a diffuser of a jet engine steadily with a velocity of 200 m/s. The inlet area of diffuser is 0.40 m2. Determine the mass flow rate of air. SOLUTION: w = P/RT = 80/0.287(10+273) = 0.985 kg/m3

m = wvA = 0.985(200)(0.40) = ___________ANS. # Consider a refrigeration whose 40watts light bulb remains on continuously as a result of a malfunction of the switch. If the refrigerator has a COP of 1.3 and the cost of electricity is 8 cents per kw-hr, determine the increase in the energy consumption of the refrigeration and its cost per year if the switch is not fixed. SOLUTION: COP = Re / W ref 1.3 = 40 / W ref Wref = W b + W ref = 40 + 30.769 = 70.77 watts W = 0.07077 KW Cost = 0.07077(8760)(P0.08) = ___________ANS. # A 75 hp motor that has an efficiency of 91% is worn out and is replaced by a highefficiency motor that has an efficiency motor that has an efficiency of 95.4%. Determine the reduction in heat gain of the room due to higher efficiency under full-load conditions. SOLUTION: P01 = (75x0.746)(0.91) = 50.91 KW P02 = (75x0.746)(0.954) = 53.376 KW Qreduced = 53.376 – 50.91 = _________ANS. # A household refrigerator that has a power input of 450 watts and a COP of 2.5 is to cool five large watermelons, 10 kg each, to 8⁰C. If the watermelons are initially at 20⁰C, determine how long will it take for the refrigerator to cool them. The watermelons can be treated as water whose specific heat is 4.2 KJ/kg-⁰K. SOLUTION: COP = Re/Wc 2.5 = Re/450 Re = 1,125 watts Re = mcp(t2 – t1) 450t = (10 x 5)(4.2)(20 – 8) t = ____________ANS. # When a man returns to his wall-sealed house on a summer day, he finds that the house is at 32⁰C. He returns on the air conditioner which cools the entire house to 20⁰C in 15 minutes. If COP is 2.5, determine the power drawn by the air conditioner. Assume the entire mass within the house is 800 kg of ait which cv= 0.72 KJ/kg-K, cp=1.0 KJ/kg-K. SOLUTION: Re = mcv(t2 – t1) = (800/15x60)(0.72)(32 – 20) Re = 7.68 KW Wc = 7.68 / 2.5 = ___________ANS. A heat source at 800°K losses 2000 KJ of heat to a sink at 500°K. Determine the entropy generated during the process. Solution: ∆Ssource = -2000/800 = -2.5 ∆Ssink = 2000/500 = 4 ∆Sgen = -2.5/4 = _____________ANS

Helium gas in compressed in an adiabatic compressor from an initial state of 14 psia and 50°F to a final temperature of 320°F in a reversible manner. Determine the exit pressure of Helium. Solution: T2/T1 = (P2/P1)n-1/n (320+460)(50+460) = (P2/14)1.667-1/1.667 P2 = _____________ANS Air pass thru a nozzle with efficiency of 90%. The velocity of air at the exit is 600 m/s. Find the actual Velocity at the exit. Solution: e = (Va/V3)2 0.9 = (Va/600)2 Va = _____________ANS A 50 kg block of iron casting at 500°K is thrown into a large lake that is at a temperature of 285°K. The iron block eventually reaches the thermal equilibrium with the lake water. Assuming average specific heat of 0.45 KJ/kg-K for the iron, determine the entropy generated during the process. Solution: ∆Siron = m c ln(T2/T1) = 50(0.45)ln (285/500) = -12.65 KJ/K ∆Slake = Q/T = [50(0.45)(500-285)] / 285 = 16.97 KJ/K ∆Sgen = -12.65 + 16.97 = _____________ANS A windmill with a 12 m diameter rotor is to be installed at a location where the wind is blowing at an average velocity of 10 m/s. Using standard condition of air (1 atm, 25°C), determine the maximum power that can be generated by the windmill. Solution: w = P/RT = 101.325 / (0.287)(25+273) = 1.1847 kg/m 3 m = w A v = 1.1847(π/4 x 122)(10) = 1,339.895 kg/s KE = v2/2000 = 102/2000= 0.05 KJ/kg Power = m KE = 1,339.895(0.05) = _____________ANS Consider a large furnace that can supply heat at a temperature of 2000°R at a steady rate of 3000 Btu/s. Determine the exergy of this energy. Assume an environment temperature of 77°F. Solution: W = e Q = 0.7315 (3000) = 2194.5 Btu/s = _____________ANS A heat engine receives hat from a source at 1200°K at a rate of 500KJ/s and rejects the waste heat to a medium at 300°K. The power output of the heat engine is 180 KW. Determine the irreversibility rate for this process. Solution: e = (1200-300) / 1200 = 0.75 W= 0.75(500) = 375 KW Irreversibilities = 375-180 = _____________ANS A dealer advertises that he has just received a shipment of electric resistance heaters for residential buildings that have an efficiency of 100 percent. Assuming an indoor temperature of 21°C and outdoor temperature of 10°C, determine the second law efficiency of these heaters.

Solution: COP1 = 100 efficient = 1 COP2 = (21+273) / (21-10) = 26.72 esl = COP1 / COP2 = 1 / 26.72 = _____________ANS A thermal power plant has a heat rate of 11,363 Btu/KW-hr. Find the thermal efficiency of the plant. Solution: e = 3412 / Heat rate = 3412 / 11363 = _____________ANS A rigid tank contains 2 kmol of N2 and 6 kmol of CO2 gases at 300°K and 115 Mpa. Find the tank volume using ideal gas equation Solution: Pm Vm = Nm Ru Tm 15,000 Vm = (6+2)(8.314)(300) Vm = _____________ANS A spherical balloon with a diameter of 6 m is filled with helium at 20°C and 200 kpa. Determine the mole number. Solution: PV=NRT (200) = N (8.314) (20+273) N = _____________ANS The air is an automobile tire with a volume of 0.53 ft3 is at 90°Fand 20 psig. Determine the amount of air that must be added to raise the pressure to the recommended value of 30 psig. Assume the atmospheric pressure to be 14.7 psia and the temperature and the volume to remain constant. Solution: PV=mRT (20+14.7)(144)(0.53) = m1 (53.3)(90+460) m1 = 0.09034 lb (30+14.7)(144)(0.53) = m2 (53.3)(90+460) m2 = 0.11634 lb madded = m2 – m1 = 0.11634 - 0.09034 = _____________ANS A rigid tank contains 20 lbm of air at 20 psia and 70°F. More air is added to the tank until the pressure and temperature rise to 35 psia and 90°F, respectively. Determine the amount of air added to the tank. Solution: P1 V1 = m1 R1 T1 (20 x 144) (V1) = 20(53.3)(70+460) V = 196.17 ft3 P2 V2 = m2 R2 T2 (35 x 144)(196.17) = m2(53.3)(90+460) m2 = 33.73 lbs madded = m2 – m1 = 33.73 – 20 = _____________ANS A rigid tank contains 5 kg of an ideal gas at 4 atm and 40°C. Now a valve is opened and half of mass of the gas is allowed to escape. If the final pressure in the tank is 1.5 atm, the final temperature in the tank is:

Solution: PV=mRT (4 x 9.81)(V) = 5(0.287)(40+273) V = 11.446 m3 PV=mRT (1.5 x 9.81)(11.446) = (5/2)(0.287)(T) T = _____________ANS The pressure of an automobile tire is measured to be 200 kpa(gage) before the trip and 220 kpa(gage) after the tip at a location where the atmospheric pressure is 90 kpa. If the temperature of the air in the tire before the trip is 25°C, the air temperature after the trip is: Solution: T2/T1 = P2/P1 T2 / (25+273) = (220+90) / (200+90) T2 = 318.55°K t2 = _____________ANS Water is boiling at 1 atm pressure in a stainless steel pan on an electric range. It is observed that 2 kg of liquid water evaporates in 30 min. The rate of heat transfer to the water is: Solution: Q = mL = = _____________ANS Consider a person standing in a breezy room at 20°C. Determine the total rate of heat transfer from this person if the exposed surface area and the average outer surface temperature of the person are 1.6 m2 and 29°C, respectively, and the convection heat transfer coefficient is 6W/ m2 with emissivity factor of 0.95. Solution: Qc = h A (t2-t1) = (6)(1.6)(29.20) = 86.40 watts Qf = (0.95)(5.67 x 10-8)(1.6)[(29+273)4-(20+273)4] = 81.7 watts Q = Qc + Qf = 86.40 + 81.7 = _____________ANS Water is boiled in a pan on a stove at sea level. During 10 minutes of boiling, it is observed that 200 grams of water has evaporated. Then the rate of heat transfer to the water is: Solution: Q = mL = (0.2/10) (2257) = _____________ANS An aluminum pan whose thermal conductivity is 237 W/m-C has a flat bottom whose diameter is 20 cm and thickness 0.4 cm. Heat is transferred steadily to boiling water in the pan through its bottom at a rate of 500 watts. If the inner surface of the bottom of the pan is 105°C, determine the temperature of the surface of the bottom of the pan. Solution: A = π/4 (0.20)2 = 0.0314m2

t2 = _____________ANS For a heat transfer purposes, a standing man can be modeled as a 30 cm diameter, 170 cm long vertical cylinder with both the top and bottom surfaces insulated and with the side surface at an average temperature of 34°C. For a convection heat transfer

coefficient of 15 W/m2-°C, determine the rate of heat loss from this man by convection in an environment at 20°C. Solution: Qc = k A (t2-t1) = 15(π x 0.30 x 1.7) (34-20) = _____________ANS A 5 cm diameter spherical ball whose surface is maintained at a temperature of 70°C is suspended in the middle of a room at 20°C. If the convection heat transfer coefficient is 15 W/m2-°C and the emissivity of the surface is 0.8, determine the total heat transfer from the ball. Solution: A = 4 π r2 = 4 π (0.05)2 = 0.0314 m2 Qc = h A (t2-t1) = 15(0.0314) (70-20) = 23.56 watts Qr = (0.80) (5.67 x 10-8) (0.0314) [(70+273)4-(50+273)4] = 9.22 watts Q = Qc + Qr = 23.56 + 9.22 = _____________ANS A frictionless piston-cylinder device and a rigid tank contain 1.2 kmol of an ideal gas at the same temperature, pressure, and volume. Now heat is transferred, and the temperature of both system is raised by 15°C. The amount of extra heat that must be supplied to the gas in the cylinder that is maintained at constant pressure. Solution: Q = m Cp (t2-t1) = (1.2 x 8.314)(1)(15) = _____________ANS A supply of 50 kg of chicken at 6°C contained in a box to be frozen to -18°C in a freezer. Determine the amount of heat that needs to be removed. The latent heat of the chicken is 247 KJ/kg, and its specific heat is 3.32 KJ/kg-C above freezing and 1.77 KJ/kg-C below freezing. The container box is 1.5 kg, and the specific heat of the box material is 1.4 KJ/kg-C. Also the freezing temperature of chicken is -2.8°C. Solution: Qchicken = 50[3.32(6+2.8) + 247 + 1.77(-2.8+18)] = 15,156 KJ Qbox = 1.5(1.4)(6+18) = 50.4 KJ Q = 15,156 + 50.4 = _____________ANS Water is being heated in a closed pan on top of a range while being stirred by a paddle wheel. During the process, 30 KJ of heat is transferred to the water and 5 KJ of heat is lost to the surrounding air. The paddle-wheel work amounts to 500 N-m. Determine the final energy of the system if its initial energy is 10 KJ. Solution: Final energy = QA + ∆U – Qlose + W = 30 + 10 – 5 + 0.50 = _____________ANS A classroom that normally contains 40 people is to be air-conditioned with window airconditioning units of 5 KW cooling capacity. A person at rest may be assumed to dissipate heat at rate of about 360 KJ/hr. There are 10 light bulbs in the room, each with a rating of 100 watts. The rate of heat transfer to the classroom through the walls and the windows is estimated to be 15,000 KJ/hr. If the room to be maintained at a constant temperature of 21°C, determine the number of window air-conditioning units required. Solution: Q = total heat load = 40(360/3600) + 10(0.100) +15,000/3600 = 9.167 KW No. of air conditioning = 9.167/5 = 1.833 = _____________ANS A 4m x 5m x 6m room is to be heated by a baseboard resistance heater. It is desired that the resistance heater be able to raise the air temperature in the room from 7 to 23°C within 15 minutes. Assuming no heat losses from the room and an atmospheric

pressure of 100 kpa, determine the required power of the resistance heater. Assume constant specific heats at room temperature. Solution: w = P/RT = 100 / (0.287)(7+273) = 1.244 kg/m3 m = 1.244(4 x 5 x 6) = 149.28 kg Q= m Cv (t2-t1) = 149.28(0.7186)(23-7) = 1,716.36 KJ Power = 1,716.36 / (15 x 60) = _____________ANS A student living in a 4m x 6m x 6m dormitory room turns on her 150 watts fan before she leaves the room on a summer day, hoping that the room will be cooler when she comes back in the evening. Assuming all the doors and windows are tightly closed and disregarding any heat transfer through walls and the windows, determine the temperature in the room when she comes back 10 hours later. Use specific heat values at room temperature, and assume the room to be at 100 kpa and 15°C in the morning when she leaves. Solution: w = P/RT = 100 / (0.287)(15+273) = 1.2098 kg/m3 m = 1.2098(4 x 6 x 6) = 174.216 kg Q= m Cv (t2-t1) 0.15(10 x 3600) = 174.216(0.7186)(t2 - 15) t2 = _____________ANS A piston-cylinder device whose piston is resting on top of a set of stops initially contains 0.50 kg of helium gas at 100 kpa and 25°C. The mass of the piston is such that 500 kpa of pressure is required to raise it. How much heat must be transferred to the helium before the piston starts rising? Solution: For helium: Cv = R/(k-1) = (8.314/4)(1.667-1) = 3.116 KJ/kg-K T2 = (25+273)(500/100) = 1,490°K T1 = 25+273 = 298°K Q= m Cv (T2-T1) = 0.50(3.116)(1490-298) = _____________ANS In order to cool 1 ton (1000 kg) of water at 20°C in an insulated tank, a person pours 80 kg of ice at -5°C into the water. Determine the final equilibrium temperature in the tank. The melting temperature and the heat of fusion of ice at atmospheric pressure are 0°C and 333.7 KJ/kg, respectively. Solution: Qwater = Qice 1000(4.187)(20-tB) = 80(2.09)(0+5) + 80(333.7) + 80(4.187)(tB-0) tB = _____________ANS A fan is powerd by a 0.5 hp motor delivers air at a rate of of 85 m 3/min. Determine the highest value for the average velocity of air mobilized by the fan. Take the density of air to be 1.18 kg/m3. Solution: P=wQh 0.50(0.746) = (1.18 x 0.00981)(85/80)(h) = _____________ANS An Ocean-Thermal Energy Conversion power plant generates 10,000 KW using a warm surface water inlet temperature of 28°C and a cold deep-water temperature of 15°C. On

the basis of a 3°C drop in the temperature of the warm water and a 3°C rise in the temperature of the cold water due to removal and addition of heat, calculate the power required in KW to pump the cold-deep water to the surface and through the system heat exchanger if the required pumping pressure increase is 12 kpa.Assume a Carnot cycle efficiency and density of cold water to be 1000 kg/m3. Solution: e = (TH-TL)/TH = [(28+273)-(15+273)(28+273)] = 0.03679 e = W/QA 0.03679 = 10,000/QA QA = 271,812.99 KW QR = QA – W = 271,812.99 -10,000 = 261,813 KW QR = m Cp (∆t) 261,813 = m (3)(4.187) m = 20,843.32 kg/s 20,843.32 kg/s or 20,843.32 lit/s = 20.843 m3/s h = P/w = 12/9.81 = 1.223 m Wc = w Q h = 9.81(20.843)(1.223) = _____________ANS A plane-type of solar energy collector with an absorbing surface covered by a glass plate is to receive an incident radiation of 800 W/m2. The glass plate has a reflectivity of 0.12 and a transmissivity of 0.85. The absorbing surface has an absorptivity of 0.90. The area of the collector is 5 m2. How much solar energy in watts is absorbed by the collector? Solution: Q = heat absorbed from the sun Q = 16.3(π/4 x 0.008)[15-(-190)] = _____________ANS An elastic sphere containing gas at 120 kPa has a diameter of 1.0 m. Heating the sphere causes it to expand to a diameter of 1.3 m. During the process the pressure is proportional to the sphere diameter. Calculate the work done by the gas in KJ. Solution: PαD P = kD 120 = k(1) k = 120 P = 120 D V = 4/3 π (10/2)3 dV = 12/24 π D2 dD W = _____________ANS # An ideal gas with a molecular weight of 7.1 kg/kg mol is compressed from 600 kPa and 280 K to a final specific volume of 0.5 m3/kg. During the process the pressure varies according to p = 620 + 150v + 95v2 where p is in kPa and v in m3/kg. Calculate the work of compression in KJ/kg. Solution: V1 = RT/P = (8.314/7.1)(280)/(600) = 0.546 m3/kg W = __________________ANS.

# A one cubic meter container contains a mixture of gases composed of 0.02 kg-mol of oxygen and 0.04 kg-mol of helium gas at a pressure of 220 kPa. What is the temperature of this ideal gas mixture in degrees Kelvin? Solution: V = V1 + V2 VT = m1R1T1/P1 + m2R2T2/P2 1 = (0.02x32)(8.314/32)(T)/220 + (0.04x32)(8.314/4)(T)/220 T = __________ANS. # Methyl alcohol (CH3OH) is burned with 25% excess air. How much unburned oxygen in kg-mol-oxygen/kg-mol-fuel will there be in the products if the combustion is complete? Solution: CH3OH + O2 +3.76N2 = CO2 + H2O + 3.76N2 CH3OH + 1.5O2 + (1.5) (3.76) N2 = 1CO2 + H2O + 1.5(3.76) N2 Consider 25% excess air: CH3OH + 1.5O2 + 1.25(1.5) (3.76) N2 = 1CO2 + H2O + 1.25(1.5) (3.76) N2 + 0.25(1.5) O2 Unburned O2 = 0.25(1.5) = _________ANS. # A 12 DC electrical motor draws a current of 15 amps. How much work in KJ does this motor produce over a 10-minute period of operation? Solution: W=E=QV W = (15 x 10 x 60)(12) = 108,000 J = _____________ANS. # A 4liter (2-liter per revolution at standard pressure and temperature) spark ignition engine has a compression ratio of 8 and 2200 KJ/kg heat addition by the fluid combustion. Considering a cold air-standard Otto cycle model, how much power will the engine produce when operating at 2500 rpm? Solution: w= 1.2 kg/m3 (standard density of air) m = 2 li/rev x 2500 rev/min / 1.2 kg/m3 x 1 m3/1000 li x 1 min/60 sec = 0.10 kg/s e = W/QA e = 1 – 1/81.4-1 = 0.5647 0.5647 = W/2200 W = 1,242.34 KJ/kg (0.10 kg/s) = 124.23 KW = _____________ANS. # A simple Rankine cycle produces a 40 MW of power, 50 MW of process heated and rejects 50MW of heat to the surroundings. What is the utilization factor of this cogeneration cycle neglecting the pump work? Solution: QA = WT + W process W P = 40 + 50 + 50 = 140 KW UF = (Qprocess + W T)/QA = (50 +40)/140 = _______________ANS.

# The rate of heat transfer to the surroundings from a person at rest is about 400 KJ/hr. Suppose that the ventilation system fails in an auditorium containing 120 people and assuming that the energy goes into the air of volume 1500 m3 initially at 300K and 101 kPa, calculate the rate in °C/min of air temperature change. Solution: Q = m CV PV = mRT 101(1500) = m(0.287)(300) m = 1,759.58 kg Q = m CV 120(400/60) = 1759.58(0.7186) = __________ANS. # An insulated box containing helium gas falls from a balloon 4.5 km above the earth’s surface. Calculate the temperature rise in °C of the helium when box hits the ground. Solution: Cv of helium = 3118.9 J/kg-C m g h = m Cv m (9.81) (4500) = m (3118.9) = ___________ANS. # Consider two Carnot heat engines operating in series. The first engine receives heat from the reservoir at 2400 K and rejects the waste heat to another reservoir at temperature T. The second engine receives heat by the first one, convert some of it to work, and rejects the rest to a reservoir at 300K. If thermal efficiencies of both engines are the same, determine the temperature T. Solution: e1 = e 2 T = ___________ANS. # An ideal gas mixtrure consists of 2 kmol of N2 and 6 mol of CO2. The mass fraction of CO2 is: Solution: M = (2/8)(28) + (6/8)(44) = 40 R = 8.314/M = 8.314/40 = __________ANS. # A Carnot cycle operates between the temperature limits of 300K and 1500K, and produces 600 KW of net power. The rate of entropy change of the working fluid during the head addition process is: Solution: W= 600 = = ___________ANS.

# Air in an ideal Diesel cycle is compressed from 3 L to 0.15 L and then it expands during the constant pressure heat addition process to 0.3 L. Under cold air standard conditions, the thermal efficiency of this cycle is: Solution: rK = 3/0.15 = 20 rC = 0.3 / 0.15 = 2 e = 1 – (1/rKk-1)[(rCK – 1)/k(rC – 1)] = 0.6467 = ________ANS. # Helium gas in an ideal Otto cycle is compressed from 20°C and 2 L to 0.25 L and its temperature increases by an additional 800°C during the head addition process. The temperature of helium before the expansion process is: Solution: rk = 2/0.25 = 8 T2 = (20 + 273)(8)1.667-1 = 1,172 K T3 = T2 + 800 = 1172 + 800 = 1972K = ______________ANS. # In an ideal Otto cycle, air is compressed from 1.20 kg/m3 and 2.2 L to 0.26 L and the net work output of the cycle is 440 KJ/kg. The mean effective pressure for the cycle is: Solution: VD = V2 – V1 = 2.2 x 10-3 m3 – 0.26 x 10-3 m3 = 1.94 x 10-3 m3 W = 440 KJ/kg (1.2 kg/m3 x 2.2 x 10-3 m3) = 1.1616 KJ Pm = W/VD = 1.1616 / 1.94 x 10-3 = _____________ANS. # An ideal Brayton cycle has a net work output of 150 KJ/kg and backwork ratio of 0.4. Of both the turbine and the compressor had an isentropic efficiency of 80%, the net work output of the cycle would be? Solution: Backwork Ratio = W C / W T WC = 0.40 W T Wnet = W T - W C Wnet = 250 KJ/kg Wnet’ = 250(0.80) – 0.40(200)/0.80 = ___________ANS. # Air enters a turbojet engine at 200 m/s at a rate of 20 kg/s, and exits at 800 m/s relative to the aircraft. The thrust developed by the engine is: Solution: Thrust developed = m (v2 – v1) = 20(800-200) = ____________ANS. # A thermal power plant has a net power 10 MW. The backwork ratio of the plant is 0.005. Determine the compressor work. Solution: Wnet = W T + WP BW = W P / WT WP = 0.005W T 10,000 = W T – 0.005WT WT = 10,050.25 KW

W C = 0.005(10,050.25) = ____________ANS. # A heat engine receives heat from a source at 1200K at a rate of 500 KJ/s and rejects the waste heat to a sink at 300K. If the power output of the engine is 200 KW, the second law efficiency of this heat engine is? Solution: ea = 200/500 = 0.40 et = (TH - TL)/TH = (1200-300)/1200 = 0.75 es = 0.40/0.75 = _________ANS. # A water reservoir contains 100,000 kg of water at an average elevation of 60m. The maximum amount of electric power that can be generated from this water is: Solution: P = mh = (100,000x0.00981)(60) = 58,860 KJ P = 58,860 KJ x KWh/3600KJ = ____________ANS. # A house is maintained at 22°C in winter by electric resistance heaters. If the outdoor temperature is 5°C, the second law efficiency of the resistance heaters is: Solution: ea = 100% resistance heaters et = (22-15)/(22+273) = 5.8% es = 5.8/100 = _________ANS. # A thermoelectric refrigerator that resembles a small ice chest is powered by a car battery, and has a COP of 0.10. If the refrigerator cools a 0.350 L canned drink from 20°C to 4°C in 30 min, determine the average electric power consumed by the thermoelectric refrigerator. Solution: Q = m cp (t2 – t1) =[(1 x 0.35)/(30 x 60)] (4.187) (20-4) = 13 watts COP = RE / W C 0.10 = 13/0.10 = __________ANS. MACHINE DESIGN ELEMENTS C # What is the polar section modulus of a 4-inch solid shaft? A. 25.13 in3 B. 12.57 in4 C. 12.57 in3 D. 25.13 in4 A # A hollow shaft carries a torque 3.4 KN-m at a shearing stress if 55 Mpa. The outside diameter is 1.25 times that of the inside diameter. Find the inside diameter in mm. A. 64.67 B. 46.67 C. 84.67 D. 74.84 A # It is specified that the angular deformation in a shaft should not exceed 1” in length of 1.8m. The allowable shearing stress is 53 Mpa. Determine the diameter of the shaft. The shaft material has G = 77 x 106 Mpa. A. 222.34 mm B. 234.22 mm C. 23.42 cm D. 24.22 cm B # What modulus of elasticity in tension is required to obtain a unit deformation of 0.00105 from a load producing a unit stress of 45,000 psi?

A. 40 x 106 psi B. 43 x 106 psi C. 45 x 106 psi D. 46 x 106 A # A thrust washer has an inside diameter of 0.5 inch and an outside diameter of 3 inches. For an allowable bearing pressure of 90 psi, how much axial load can it sustain? A. 618.5 lb B. 537.2 lb C. 702.2 lb D. 871.2 lb D # An air cylinder has a bore of 25 mm and is operated with shop air at a pressure of 90 ps. Find the push force extended by the piston rod in N. A. 127 B. 70 C. 402 D. 305 B # A line shaft is to transmit 200 Hp at 900 rpm. Find the diameter of the shaft. A. 2.18 inches B. 2.26 inches C. 3.18 inches D. 3.26 inches B # A main transmitting shaft transmits 350 KW to drive a generator at 2500 rpm, what is the required diameter of the shaft? A. 58.5 mm B. 82.7 mm C. 85.3 mm D. 56.2 mm D # A round steel shaft rotates at 200 rpm and is subjected to a torque of 275 N-m and a bending moment of 415 N-m. Determine the equivalent twisting moment. A. 597.84 N-m B. 456.42 N-m C. 546.43 N-m D. 497.85 N-m A # A 4 inches shaft using a flat key, whose width is 1 inch, is transmitting a torque of 53,000 in-lb. If the design shearing stress is 5000 psi, determine the safe length of key. A. 6.3 inches B. 5.3 inches C. 4.3 inches D. 7.0 inches A # a 75 mm diameter shaft is transmitting 300 KW at 600 rpm. A flange coupling is used and has 6 bolts, each 18 mm in diameter. Find the required diameter if the bolt circle based on an average shearing stress of 27.5 Mpa. A. 227.4 mm B. 477.2 mm C. 274.7 mm D. 247.7 mm C # A heavy duty shaft coupling is to be secured woth 25 mm bolts at a distance of 150 mm from the shaft center. The shaft transmits 4330 KW of power at a speed of 1200 rp,. If the allowable shearing stress for bolts is 100 Mpa, how many bolts are required? A. 3 B. 6 C. 5 D. 4 B # A 1.75-inch diameter shaft is supported by two sleeve bearings. The total load on the wo bearings is 2900 lb. Find the friction power loss, in Hp, if the coefficient of friction between shaft and bearing is 0.10 and the shaft rotates 200 rpm. A. 0.88 Hp B. 0.78 Hp C. 0.98 Hp D. 0.68 Hp B # Find the horsepower lost when a collar is loaded with 1000 lb, rotates at 25 rpm, and a coefficient of friction at 0.15. The outside diameter of the collar is 4 inches and the inside diameter is 2 inches. A. 0.0629 Hp B. 0.0925 Hp C. 0.0269 Hp D. 0.0692 Hp C # A sleeve bearing has an outside diameter of 38.1 mm and a length of 50.1 mm, the wall thickness is 3/16 inch. The bearing is subjected to radial load of 450 lb, determine the bearing pressure. A. 100 psi B. 150 psi C. 200 psi D. 250 psi A # A vertical steel cylinder water tank is 30 m in diameter and 45 m high. The allowable stress of the steel plate is 120 Mpa. Without reinforcing angle bars and rods, determine the thickness of the steel plate. A. 55.2 mm B. 56.2 mm C. 65.2 mm D. 52.6 mm

B # Determine the bursting steam pressure of a hemispherical steel shell with a diameter of 100 inches and made of 0.0635 m thick steel plate. The joint efficiency is 70% and the tensile strength is 60,000 psi. A. 4020 psi B.4200 psi C. 2500 psi D. 2040 psi A # A cylinder having an internal diameter of 508 mm and external diameter of 814.4 mm is subjected to an internal pressure of 69 Mpa and an external of 14 Mpa. Determine the hoop stress at the inner surface of the cylinder 90.11 Mpa B. 91.10 Mpa C. 911.0 Mpa D. 19.10 Mpa A # The root diameter of a double square thread is 0.55 inch. The screw has a pitch of 0.2 inch. Find the outside diameter and the number of threads per inch. A. 0.75 inch and 5 threads/inch C. 0.50 inch and 5 threads/inch B. 0.75 inch and 4 threads/inch D. 0.50 inch and 4 threads/inch B # Two shafts 3.6 m between centers carry pulleys 1.2 m in diameter and 0.91 m in diameter respectively, connected by a crossed belt. It is desired to put the belt on as an open belt. How long a piece must be cut of it? 300 mm B. 350 mm C. 400 mm D. 250 mm B # A flat belt is 6 inches wide and 1/3 inch thick and transmits 15 Hp. The center distance is 8 ft. The driving pulley is 6 inches in diameter and rotates at 2000 rpm such that the loose side of the belt is on top. The driven pulley is 18 inches in diameter. The belt material is 0.035 lb/in3 and the coefficient of friction is 0.30. Determine the net belt tension. 167.56 lb B. 157.56 lb C. 156.75 lb D. 175.56 lb A # Two pulleys, 80 cm apart, carry a belt in an open connection. If the diameter of the pulleys are 40 cm and 15 cm, what is the length of the belt needed? 248.35 cm B. 348.35 cm C. 265.45 cm D. 305.35 cm A # A pulley 600 mm in diameter transmits 40 KW at 500 rpm. The arc of contact between the belt and pulley is 144o , the coefficient of friction between belt and pulley is 0.35 and the safe working stress of the belt is 2.1 Mpa. Determine the belt tension ratio, neglecting the effect of centrifugal force. 2.41 B. 2.14 C. 1.24 D. 4.12 B # A roller chain and sprocket is to drive vertical discharge bucket elevator. The pitch of chain connecting sprockets is 1.75”. The sprocket is rotating at 120 rpm and has 11 teeth while the driven sprocket is rotating at 38 rpm. Determine the number of teeth of driven sprocket. 33 teeth B. 35 teeth C. 30 teeth D. 37 teeth C # A disc clutch has 6 pairs of contacting friction surfaces with an outside diameter of 200 mm and an inside diameter of 100 mm. The coefficient of friction of the clutch materials is 0.4 and the axial force is 1500 N. The shaft speed is 1200 rpm. Find the Hp that can be transmitted by the clutch assuming uniform pressure. 35.2 Hp B. 23.5 Hp C. 47.2 Hp D. 27.4 Hp A # Determine the power capacity of a cone clutch under uniform pressure and assuming the following pressure and assuming the following conditions; major diameter = 250 mm, minor diameter = 200 mm, length of conical elements in contact-125 mm, rotating speed = 870 rpm, coefficient of friction = 0.30, and allowable pressure = 70 kpa. 19.2 KW B. 21.9 KW C. 29.1 KW D. 12.9 KW

D # A flywheel has a mean diameter of 4ft and is required to handle 2250 ft-lb of kinetic energy. It has a width of 8 inches, mean operating speed is 300 rpm and the coefficient of fluctuation is to be 0.05. Find the weight of rim, assuming that the arms and hub are equivalent to 10% of the total rim weight. The flywheel is made up of cast iron with specific weight of 0.26 lb per cubic inch. 334 lb B. 434 lb C. 433 lb D. 343 lb A # A 20o involute spur gear has a tooth whole depth of 16.95 mm, a tooth thickness of 13.2 mm, and a pitch of 3. Determine the circular pitch of the gear. 26.6 mm B. 16.6 mm C. 25.6 mm D. 24.6 mm C # A parallel helical gear-set consists of a 19-tooth pinion driving a 57-tooth gear. The pinion has a left-hand helix of 20o, a normal pressure angle of 14 1/2 deg . and a normal diametral pitch of 10 tooth/inch. If the pinion is to transmit 50 Hp at a speed of 1750 rpm, determine the center distance of the two gears. 2.02 inch B. 6.06 inch C. 4.04 inch D. 2.06 inch A # A right-handed single-thread hardened steel worm has a catalog rating of 2.25 KW at 650 rpm when meshed with a 48-tooth cast-steel gear. The axial pitch of the worm is 25 mm, normal pressure angle is 14.5o, and the pitch diameter of the worm is 100 mm. The coefficient of friction is 0.085. Determine the shaft center distance. 241 mm B. 142 mm C. 412 mm D. 124 mm o A # A 20 straight-tooth bevel pinion having 14 teeth and a diametral pitch of 6 tooth/inch drives a 42-tooth gear. The two shafts are at right angles and in the same plane. Find the pitch angle of the pinion. 18.4o B. 20o C. 14.5o D. 20.5o A # A triple thread worm has a lead angle of 17o and a pitch diameter of 2.2802 inches. Find the center distance when the worm is mated with a wheel of 48 teeth. 6.72 inches B. 7.26 inches C. 6.27 inches D. 7.62 inches B # A double thread worm has a pitch diameter of 3 inches. The wheel has 20 teeth and a pitch diameter of 5 inches. Find the gear helix angle. 4.69o B. 9.46o C. 6.49o D. 6.94o C # What is the polar section modulus of a 4-inch solid shaft? A. 25.13 in3 B. 12.57 in4 C. 12.57in3 D. 25.13in4 A # A hollow shaft carries a torque 3.4 KN-m at a shearing stress Mpa. The outside diameter is 1.25 times that of the inside diameter. Find the inside diameter in mm. 64.87 B. 46.87 C. 84.67 D. 74.64 A # It is specified that the angular deformation in a shaft should not to exceed 1” in length of 1.8 m. The allowable shearing stress is 83 Mpa. Determine the diameter of the shaft. The shaft material has G = 77 x 108 Mpa. 222.34 mm B. 234.22 mm C. 23.42 mm D. 24.22 mm B # What modulus of elasticity in tension is required to obtain a unit deformation of 0.00105 from a load producing a unit stress of 45,000 psi? 40 x 106 psi B. 43 x 106 psi C. 45 x 106 psi D. 46 x 106 psi A # A thrust washer has an inside diameter of 0.5 inch and an outside diameter of 3 inches. For an allowable bearing pressure of 90 psi, how much axial load can it sustain? 618.5 lb B. 537.2 lb C. 702.2 lb D. 871.2 lb D # An air cylinder has a bore of 25 mm and is operated with shop air at a pressure of 90 psi. Find the push force exerted by the piston rod in N.

127 B. 70 C. 402 D. 305 B # A line shaft is to transmit 200 Hp at 900 rpm. Find the diameter of the shaft. 2.18 inches B. 2.28 inches C. 3.18 inches D. 3.28 inches B # A main transmitting shaft transmits 350 KW to drive a generator at 2500 rpm, what is the required diameter of the shaft? A.58.5 mm B. 62.7 mm C. 65.3 mm D. 56.2 mm D # A round steel shaft rotates at 200 rpm and is subjected to a torque of 275 N-m and a bending moment of 415 N-m. Determine the equivalent twisting moment. 597.84 N-m B. 456.42 N-m C. 546.43 N-m D. 497.85 N-m A # A 4 inches shaft using a flat key, whose width is 1 inch, is transmitting a torque of 63,000 in-lb. If the design shearing stress is 5000 psi, determine the safe length of key. 6.3 inches B. 5.3 inches C. 4.3 inches D. 7.0 inches A # A 1200 mm cast iron pulley is fastened to a 112.5 mm shaft by means of a 28.13 mm square key 175 mm long. The key and the shaft have a shearing stress of 14,000 psi. Determine the force acting at the pulley rim that will shear the key. 10015 lb B. 11005 lb C. 11050 lb D. 10501 lb A # A 75 mm diameter shaft is transmitting 300 KW at 600 rpm. A flange coupling is used and has 6 bolts, each 18 mm in diameter. Find the required diameter of the bolt circle based on an average shearing stress of 27.5 Mpa. 227.4 mm B. 477.2 mm C. 274.7 mm D. 247.7 mm C # A heavy duty shaft coupling is to be secured with 25 mm bolts at a distance of 150 mm from the shaft center. The shaft transmits 4330 KW of power at a speed of 1200 rpm. If the allowable shearing stress for bolts is 100 Mpa, how many bolts are required? 3 B. 6 C. 5 D. 4 B # A 1.75-inch diameter shaft is supported by two sleeve bearings. The total load on the two bearings is 2800 lb. Find the friction power loss, in Hp, if the coefficient of friction between shaft and bearing is 0.10 and the shaft rotates 200rpm. 0.88 Hp B. 0.78 Hp C. 0.98 Hp D. 0.68 Hp B # Find the horsepower lost when a collar is loaded with 1000 lb, rotates at 25 rpm, and a coefficient of friction at 0.15. The outside diameter of the collar is 4 inches and the inside diameter is 2 inches. 0.0629 Hp B. 0.0926 Hp C. 0.0269 Hp D. 0.0692 Hp C # A sleeve bearing has an outside diameter of 38.1 mm and a length of 50.1 mm, the wall thickness is 3/16 inch. The bearing is subjected to radial load of 450 lb, determine the bearing pressure. 100 psi B. 150 psi C. 200 psi D. 250 psi A # A vertical steel cylinder water tank is 30 m in diameter and 45 m high. The allowable stress of the steel plate is 120 Mpa. Without reinforcing angle bars and rods, determine the thickness of the steel plate. 55.2 mm B. 56.2 mm C. 65.2 mm D. 52.6 mm B # Determine the bursting steam pressure of a hemispherical steel shell with a diameter of 100 inches and made of 0.0635 m thick steel plate. The joint efficiency is 70% and the tensile strength is 98 psi. 4020 psi B. 4200 psi C. 2500 psi D. 2040 psi

A # A cylinder having an internal diameter of 508 mm and external diameter of 914.4 mm is subjected to an internal pressure of 69 Mpa and an external of 14 Mpa. Determine the hoop stress at the side surface of the cylinder. A. 90.11 Mpa B. 91.10 Mpa C. 911.0 Mpa D. 19.10 Mpa A # The root diameter of a double square thread is 0.55 inch. The screw has a pitch of 0.2 inch. Find the outside diameter and the number of threads per inch. A. 0.75 inch and 5 threads/inch C. 0.50 inch and 5 threads/inch A. 0.75 inch and 4 threads/inch D. 0.50 inch and 4 threads/inch B # Two shafts 3.6 m between centers carry pulleys 1.2 m in diameter and 0.91 m in diameter respectively, connected by a crossed belt. It is desired to put the belt on as an open belt. How long a piece must be cut of it? A. 300 mm B. 350 mm C. 400 mm D. 250 mm B # A flat belt is 6 inches wide and 1/3 inch thick and transmits 15 Hp. The center distance is 8 ft. The driving pulley is 6 inches in diameter and rotates at 2000 rpm such that the loose side of the belt is on top. The driven pulley is 18 inches in diameter. The belt material is 0.035 lb/in3 and the coefficient of friction is 0.30. Determine the net belt tension. A. 167.56 lb B. 157.56 lb C. 156.75 lb D. 175.56 lb A # Two pulleys, 80 cm apart, carry a belt in an open connection. If the diameter of the pulleys are 40 cm and 15 cm, what is the length of the belt needed? A. 248.35 cm B. 348.35 cm C. 265.45 cm D. 305.35 cm A # A pulley 600 mm in diameter transmits 40 KW at 500 rpm. The arc of contact between the belt and pulley is 144o , the coefficient of friction between belt and pulley is 0.35 and the safe working stress of the belt is 2.1 Mpa. Determine the belt tension ratio, neglecting the effect of centrifugal force. A. 2.41 B. 2.14 C. 1.24 D. 4.12 B # A roller chain and sprocket is to drive vertical discharge bucket elevator. The pitch of chain connecting sprockets is 1.75”. The sprocket is rotating at 120 rpm and has 11 teeth while the driven sprocket is rotating at 38 rpm. Determine the number of teeth of driven sprocket. A. 33 teeth B. 35 teeth C. 30 teeth D. 37 teeth C # A disc clutch has 6 pairs of contacting friction surfaces with an outside diameter of 200 mm and an inside diameter of 100 mm. The coefficient of friction of the clutch materials is 0.4 and the axial force is 1500 N. The shaft speed is 1200 rpm. Find the Hp that can be transmitted by the clutch assuming uniform pressure. A. 35.2 Hp B. 23.5 Hp C. 47.2 Hp D. 27.4 Hp A # Determine the power capacity of a cone clutch under uniform pressure and assuming the following pressure and assuming the following conditions; major diameter = 250 mm, minor diameter = 200 mm, length of conical elements in contact-125 mm, rotating speed = 870 rpm, coefficient of friction = 0.30, and allowable pressure = 70 kpa. A. 19.2 KW B. 21.9 KW C. 29.1 KW D. 12.9 KW D # A flywheel has a mean diameter of 4ft and is required to handle 2250 ft-lb of kinetic energy. It has a width of 8 inches, mean operating speed is 300 rpm and the coefficient of fluctuation is to be 0.05. Find the weight of rim, assuming that the arms and hub are

equivalent to 10% of the total rim weight. The flywheel is made up of cast iron with specific weight of 0.26 lb per cubic inch. A. 334 lb B. 434 lb C. 433 lb D. 343 lb o A # A 20 involute spur gear has a tooth whole depth of 16.95 mm, a tooth thickness of 13.2 mm, and a pitch of 3. Determine the circular pitch of the gear. A. 26.6 mm B. 16.6 mm C. 25.6 mm D. 24.6 mm C # A parallel helical gear-set consists of a 19-tooth pinion driving a 57-tooth gear. The pinion has a left-hand helix of 20o, a normal pressure angle of 14 1/2 deg . and a normal diametral pitch of 10 tooth/inch. If the pinion is to transmit 50 Hp at a speed of 1750 rpm, determine the center distance of the two gears. A. 2.02 inch B. 6.06 inch C. 4.04 inch D. 2.06 inch A # A right-handed single-thread hardened steel worm has a catalog rating of 2.25 KW at 650 rpm when meshed with a 48-tooth cast-steel gear. The axial pitch of the worm is 25 mm, normal pressure angle is 14.5o, and the pitch diameter of the worm is 100 mm. The coefficient of friction is 0.085. Determine the shaft center distance. A. 241 mm B. 142 mm C. 412 mm D. 124 mm A # A 20o straight-tooth bevel pinion having 14 teeth and a diametral pitch of 6 tooth/inch drives a 42-tooth gear. The two shafts are at right angles and in the same plane. Find the pitch angle of the pinion. A. 18.4o B. 20o C. 14.5o D. 20.5o A # A triple thread worm has a lead angle of 17o and a pitch diameter of 2.2802 inches. Find the center distance when the worm is mated with a wheel of 48 teeth. A. 6.72 inches B. 7.26 inches C. 6.27 inches D. 7.62 inches B # A double thread worm has a pitch diameter of 3 inches. The wheel has 20 teeth and a pitch diameter of 5 inches. Find the gear helix angle. A. 4.69o B. 9.46o C. 6.49o D. 6.94o C # What is the polar section modulus of a 4-inch solid shaft? A. 25.13 in3 B. 12.57 in4 C. 12.57in3 D. 25.13in4 A # A hollow shaft carries a torque 3.4 KN-m at a shearing stress Mpa. The outside diameter is 1.25 times that of the inside diameter. Find the inside diameter in mm. A. 64.87 B. 46.87 C. 84.67 D. 74.64 A # It is specified that the angular deformation in a shaft should not to exceed 1” in length of 1.8 m. The allowable shearing stress is 83 Mpa. Determine the diameter of the shaft. The shaft material has G = 77 x 108 Mpa. A. 222.34 mm B. 234.22 mm C. 23.42 mm D. 24.22 mm B # What modulus of elasticity in tension is required to obtain a unit deformation of 0.00105 from a load producing a unit stress of 45,000 psi? A. 40 x 106 psi B. 43 x 106 psi C. 45 x 106 psi D. 46 x 106 psi A # A thrust washer has an inside diameter of 0.5 inch and an outside diameter of 3 inches. For an allowable bearing pressure of 90 psi, how much axial load can it sustain? A. 618.5 lb B. 537.2 lb C. 702.2 lb D. 871.2 lb D # An air cylinder has a bore of 25 mm and is operated with shop air at a pressure of 90 psi. Find the push force exerted by the piston rod in N. A. 127 B. 70 C. 402 D. 305 B # A line shaft is to transmit 200 Hp at 900 rpm. Find the diameter of the shaft. A. 2.18 inches B. 2.28 inches C. 3.18 inches D. 3.28 inches

B # A main transmitting shaft transmits 350 KW to drive a generator at 2500 rpm, what is the required diameter of the shaft? A. 58.5 mm B. 62.7 mm C. 65.3 mm D. 56.2 mm D # A round steel shaft rotates at 200 rpm and is subjected to a torque of 275 N-m and a bending moment of 415 N-m. Determine the equivalent twisting moment. A. 597.84 N-m B. 456.42 N-m C. 546.43 N-m D. 497.85 N-m A # A 4 inches shaft using a flat key, whose width is 1 inch, is transmitting a torque of 63,000 in-lb. If the design shearing stress is 5000 psi, determine the safe length of key. A. 6.3 inches B. 5.3 inches C. 4.3 inches D. 7.0 inches A # A 1200 mm cast iron pulley is fastened to a 112.5 mm shaft by means of a 28.13 mm square key 175 mm long. The key and the shaft have a shearing stress of 14,000 psi. Determine the force acting at the pulley rim that will shear the key. A. 10015 lb B. 11005 lb C. 11050 lb D. 10501 lb A # A 75 mm diameter shaft is transmitting 300 KW at 600 rpm. A flange coupling is used and has 6 bolts, each 18 mm in diameter. Find the required diameter of the bolt circle based on an average shearing stress of 27.5 Mpa. A. 227.4 mm B. 477.2 mm C. 274.7 mm D. 247.7 mm C # A heavy duty shaft coupling is to be secured with 25 mm bolts at a distance of 150 mm from the shaft center. The shaft transmits 4330 KW of power at a speed of 1200 rpm. If the allowable shearing stress for bolts is 100 Mpa, how many bolts are required? A. 3 B. 6 C. 5 D. 4 B # A 1.75-inch diameter shaft is supported by two sleeve bearings. The total load on the two bearings is 2800 lb. Find the friction power loss, in Hp, if the coefficient of friction between shaft and bearing is 0.10 and the shaft rotates 200rpm. A. 0.88 Hp B. 0.78 Hp C. 0.98 Hp D. 0.68 Hp B # Find the horsepower lost when a collar is loaded with 1000 lb, rotates at 25 rpm, and a coefficient of friction at 0.15. The outside diameter of the collar is 4 inches and the inside diameter is 2 inches. A. 0.0629 Hp B. 0.0926 Hp C. 0.0269 Hp D. 0.0692 Hp C # A sleeve bearing has an outside diameter of 38.1 mm and a length of 50.1 mm, the wall thickness is 3/16 inch. The bearing is subjected to radial load of 450 lb, determine the bearing pressure. A. 100 psi B. 150 psi C. 200 psi D. 250 psi A # A vertical steel cylinder water tank is 30 m in diameter and 45 m high. The allowable stress of the steel plate is 120 Mpa. Without reinforcing angle bars and rods, determine the thickness of the steel plate. A. 55.2 mm B. 56.2 mm C. 65.2 mm D. 52.6 mm B # Determine the bursting steam pressure of a hemispherical steel shell with a diameter of 100 inches and made of 0.0635 m thick steel plate. The joint efficiency is 70% and the tensile strength is 98 psi. A. 4020 psi B. 4200 psi C. 2500 psi D. 2040 psi A # A cylinder having an internal diameter of 508 mm and external diameter of 914.4 mm is subjected to an internal pressure of 69 Mpa and an external of 14 Mpa. Determine the hoop stress at the side surface of the cylinder. D # A 35-tooth pinion turning at 300 rpm drives a 120-tooth gear of 14.5° involute full depth pressure angle. Determine the rpm of the driven gear.

A. 60 rpm B. 45 rpm C. 75 rpm D. 90 rpm B # Two parallel shafts connected by cylinders in pure rolling contact and turning in the same direction having a speed ratio of 2.75. Determine the center distance of the two shafts if the diameter of the smaller cylinder is 22 cm. A.18.25 B. 19.25 cm C. 20.25 cm D. 17.25 cm A # Three extension springs are hooked in parallel that supports a single weight of 100 kg. The first spring is rated at 4 KN/m and the other two springs are rated at 6 KN/m each. Determine the equivalent stiffness of the three springs. A. 1.71 KN/m B. 5 KN/m C. 2.71 KN/m D. 3.71 KN/m B # Three extension springs are hooked in parallel that support a single weight of 100 kg. The springs are rated 4 KN/m, 5 KN/m, and 6 KN/m, respectively. Determine the equivalent spring constant of the three springs. A. 10 KN/m B. 15 KN/m C. 9 KN/m D. 11 KN/m A # Two extension coil springs are hooked in series that support a single weight of 100 kg. The first spring is rated at 4 KN/m and the other spring is rated at 6 KN/m. Determine the total deflection of the springs. A. 410 mm B. 310 mm C. 510 mm D. 210 mm A # If the ultimate shear strength of a steel plate is 42,000 psi, what force is necessary to punch inch diameter hole in a 5/8 inch thick plate/ A. 61,850 lb B. 65,810 lb C. 61,580 lb D. 60,185 lb C # In a straight bevel gear, the angle between an element of the pitch cone and an element of the face cone is called: A. Face angle B. Pitch angle C. Addendum angle D. Dedendum angle D # A four bar mechanism in which one of the links can perform a full rotation relative to the other three links. A. Geneva mechanism C. Triple rocker mechanism B. Crossover-position mechanism D. Grashof mechanism A # A Grashof four-bar mechanism in which the shorted link is the frame or the fixed link and the other two cranks completely rotate with their axes. A. Drank link mechanism C. Double-rocker mechanism B. Crank-rocker mechanism D. Triple-rocker mechanism C # “For planar four bar linkage, the sum of the shortest and the longest lengths cannot be greater than the sum of the remaining two links” lengths if there is to be a continuous relative rotation between the two member” The preceding statement is known as: A. Grubler’s law B. Coriolli’s law C. Grashof’s law D. Freudentein’s law B # Which of the following is not true for an instant center or centro of planar linkages? A. Centro is a point common to two bodies having the same velocity in each. B. Centro is a point in one body about which another body does tend to rotate. C. Centro is a point in one body about which another body actually turns. D. Centro is a point in one body about which another body tends to turn. A # The most common work holding devices of a shaper machine with the base graduated in degrees that make it possible to swivel any angle.

A. Shaper vise B. Parallel bars and hold down bars C. Lathe holder D. Swivel head A # Shaper operation which is shaping the given stock and having the excess material remain with a tolerable allowance for finishing. A. Roughing B. Finishing C. Angular Cutting D. Contouring C # A cutting tool that has two or more cutting edges as in drill presses and milling machine cutters. A. Grinder B. Single-point cutting tool C. Multi-point cutting tool D, Two-point cutting tool A # The trade name for a patented alloy made up chiefly of cobalt, chromium, and tungsten varying proportions. A. Stellite B. Carbology C. Stainless Steel D. Copper A # The transformation of concepts and ideas into useful machinery is called as: A. Design B.Synthesis C. Analysis D. Theorem C # It is a combination of mechanisms and other components that transform, transmits, or uses energy, load, or motion for a specific purpose. A. Mechanism B. Engine C. Machine D. Linkage B # It is defined as synergistic collection of machine elements; synergistic because as a design it represents an idea or concept greater than the sum of the individual parts. A. System of mechanism B. Mechanical system C. Design system D. Expert system C # It may be defined the displacement per length produced in a solid as the result of stress. A. Deformation B. Elongation C. Strain D. Stress D # The combination of applied normal and shear stresses that produces maximum principal normal stress, with a third principal stress between or equivalent to the extremes. A. Principal shear stress C. Maximum shear stress B. Principal normal stress D, Bending and shear stress C # It is a load applied transversely to longitudinal axis of member. A. Combined loads B. Concentrated load C. Bending load D. Distributed load C # It is the intensity and direction of internal force acting at a given point on particular plane. A. Load B. Strain C. Stress D. Sustained load A # It is the capacity of a material to absorb energy when it is deformed elastically and then, upon unloading, to release this energy. A. Resilience B. Toughness C. Rigidity D. Ductility D # It is the strain energy per unit volume required to stress a material from an unloaded state to the point of yielding. A. Modulus of roughness C. Modulus of rigidity B. Modulus of elasticity D. Modulus of resilience A # The ability of the material to absorb energy up to fracture. A. Toughness B. Rigidity C. Resilience D. Stiffness B # The Maximum Shear Stress Theory, as a failure prediction theory, is also known as:

A. von Mises criterion B. Tresca yield criterion C. Coulomb-Mohr Theory D. Modified Mohr Theory B # A failure prediction theory, which states that a part subjected to any combination of loads, will fail (by yielding or fracturing) whenever the maximum shear stress exceeds a critical value. A. Distorsion-energy theory C. Internal friction theory B. Maximim-shear-stress theory D, Modified Mohr theory A # A theory in a cyclic and impact loading, which states that damage at any stress level is proportional ro the number of cycles. A, Miner’s rule B. Paris Power Law C. Goodman Rule D. Manson-Coffin Relationship B # A journal bearing where the radius of the journal is less than the radius of the bushing or bearing. A. Fitted journal bearing C. Partial journal bearing B. Clearance journal bearing D. Full journal bearing B # A lubrication where the load-carrying surfaces of the bearing are separated by a relatively thick film of lubricant, so as to prevent metal to metal contact, and where the stability of the film can be explained by the laws of fluid mechanics. A. Hydrostatic lubrication C. Elastohydrodynamic lubrication B. Hydrodynamic lubrication D. Boundary lubrication A # A lubrication condition where non-conformal surfaces are completely separated by lubricant film and no asperities are in contact. A. Elastohydrodynamic lubrication C. Hydrodynamic lubrication B. Boundary lubrication D. Hydrostatic lubrication C # A speed at which rotating shaft becomes dynamically unstable. A. Normal Speed B. Variable Speed C. Critical Speed D. Average Speed B # A ball bearing with race contacting pronounced groove for rolling elements. A. Crown bearing B. Conrad bearing C. Angular-contact bearing D. Cylindrical bearing A # A machining process for producing internal straight cylindrical surface or profiles, with process characteristics and tooling similar to those for turning operations. A. Boring B. Drilling C. Reaming D. Milling B # A machining operation for all types of metallic and nonmetallic materials and is capable of producing circular parts with straight of various profiles. A. Boring B. Turning C. Drilling D. Milling B # A set of specification for parts, materials, or processes intended to achieve uniformity, efficiency, and a specified quality. A. Code B. Standard C. Law D. Theorem A # A set of specification for the analysis, design, manufacture, and construction of something; the purpose of which is to achieve a specified degree of safety, efficiency, and performance or quality. A. Code B. Standard C. Law D. Theorem # It is defined as synergistic collection of machine elements; synergistic because as a design it represents an idea or concept greater than the sum of the individual parts. A. System of mechanism B. Mechanical system C. Design system

D. Expert system # It may be define the displacement per length produced in a solid as a result of stress. A. Deformation B. Elongation C. Strain D. Stress # The combination of applied normal and shear stresses that produces maximum principal normal stress, with a third principal stress between or equivalent to the extremes. A. Principal shear stress C. Maximum shear stress B. Principal normal stress D. Bending and shear stress # It is a load applied transversely to longitudinal axis of member. A. Combined load B. Concentrated load C. Bending load D. Distributed load. # It is the intensity and direction of internal force acting at given point on particular plane. A. Load B. Strain C. Stress D. Sustained load # It is the capacity of a material to absorb energy when it is deformed elasticity and then, upon unloading, to release this energy. A. Resilience B. Toughness C. Rigidity D. Ductility # It is the strain energy per unit volume required to stress a material from an unloaded state to the point of yielding. A. Modulus of roughness B. Modulus of elasticity C. Modulus of rigidity D. Modulus of resilience # The ability of the material to absorb energy up to the fracture. A. Toughness B. Rigidity C. Resilience D. Stiffness # The Maximum Shear Stress Theory, as a failure prediction theory, is also known as: A. von Mises criterion B. Tresca yield criterion C. Coulomb-Mohr theory D. Modified Mohr-theory # A failure prediction theory, which states that a part subjected to any combination of loads, will fail (by yielding and fracturing) whenever the maximum shear stress exceeds a critical value. A. Distorsion-energy theory C. Internal friction theory B. Maximum-shear-stress theory D. Modified Mohr theory # A theory in cyclic and impact loading, which states that damage at any stress level is proportional to the number of cycles. A. Mine’s rule B. Paris Power Law C. Goodman Rule D. Manson-coffin Relationship

# A journal bearing where the radius of the journal is less than the radius of the bushing or bearing. A. Fitted journal bearing C. Partial journal bearing B. Clearance journal bearing D. Full journal bearing # A lubrication where the load-carrying surfaces of the bearing are separated by a relatively thick film of lubricant, so as to prevent metal to metal contact, and where the stability of the film can be explained by the laws of fluid mechanics. A. Hydrostatic lubrication C. Elastohydrodynamic lubrication B. Hydrodynamic lubrication D. Boundary lubrication # A lubrication condition where non-conformal surfaces are completely separated by lubricant film and no asperities are in contact. A. Elastohydrodynamic lubrication C. Hydrodynamic lubrication B. Boundary lubrication D. Hydrostatic lubrication # A speed at which rotating shaft becomes dynamic unstable. A. Normal speed B. Variable speed C. Critical speed speed

D. Average

# A ball bearing with race containing pronounced groove for rolling elements. A. Crown bearing B.Conrad bearing C.Angular-contact bearing D.Cylindrical bearing # A machining process for producing internal straight cylindrical surface or profiles, with process characteristics and tooling similar to those for turning operations. A.Boring B.Drilling C.Reaming D.Milling # A machining operation for all types of metallic and nonmetallic materials and is capable of producing circular parts with straight or various profiles. A.Boring B.Turning C.Drilling D.Milling # An American nonprofit society, founded in 1921, whose objectives are to improve and advance the use of fabricated structural stress. A. American Iron Steel Institute (AISI) B. American Institute of Steel Construction (AISC) C. American Society for Metals (ASM) D. American Society of Testing and Materials (ASTM) # A sketch of a machine, a machine element, or part of a machine element that shows all acting forces, such as applied load and gravity forces, and all reactive forces. A. Schematic diagram B. Free body diagram C. Moment diagram D. Skeletal diagram # The size to which a limit of deviations is assigned and is the same for both members of the fit, it is the exact theoretical size.

A. Nominal stress

B. Basic size

C. Maximum size

D. Minimum size

# The algebraic difference between a size and the corresponding basic size. Tolerance B. Allowance C. Deviation D. Limit # The algebraic difference between the maximum limit and the corresponding basic. A. Fundamental deviation B. Upper Deviation C. Lower deviation D. Tolerance # The algebraic difference between the minimum limit and the corresponding basic. A. Fundamental deviation B. Upper Deviation C. Lower deviation D. Tolerance # Either the upper of the lower deviation, depending on which is closer to the basic size. A. Fundamental deviation B. Upper Deviation C. Lower deviation D. Tolerance # The difference between the maximum and minimum size limits of a part. A. Allowance B. Tolerance C. Deviation D. Basic size # The stated maximum and minimum dimensions. A. Tolerance B. Limits C. Nominal size

D. Basic size

# A general term that refers to the mating of cylindrical parts such as bolt or a hole; it is used only when the internal member is smaller that the external member. A. Clearance B. Interference C. Allowance D. Tolerance # The opposite of clearance, for mating cylindrical parts in which the internal member is larger than the external member. A. Clearance B. Allowance C. Tolerance D. Interference # The minimum stated clearance or the maximum stated interference for mating parts. A. Clearance B. Allowance C. Tolerance D. Interference # The property of a material that measures the degree of plastic deformation sustained at fracture. A. Toughness B. Stiffness C. Ductility C. Brittleness # Compounds of metallic elements, most frequently oxides, nitrides, and carbides. A. Plastic B. Polymers C. Ceramics D. Alloy # A material having different properties in all directions at point in solid. A. Isotropic material B. Anisotropic material C. Orthropic material D. Ceramic material

# A material having different properties in three mutually perpendicular directions at point in solid and having three mutually perpendicular planes of material symmetry. A. Orthotropic material B. Isotropic material C. Anistropic material D. Thermoplastic material # The combination of two or more materials, usually consisting of fiber and thermosetting polymer. A. Brittle materials B. Composite materials C. Polymers Ceramics

D.

# A theorem stating that “when a body is elastically deformed by a system of loads, the deflection at any point p in any direction a is equal to the partial derivatives of the strain energy (with the system of loads acting) with respect to a load at p in the direction a”. A. Poisson’s Theorem B. Newton’s Theorem C. Castigliano’s Theorem D. Mohr’s Theorem # A principal or method that a deflection at any point in bar is equal to sum f deflection caused by each load acting separately. A. Summation Method B. Method of balancing C. Method of superposition D. Shear and Moment diagram method # A failure prediction theory in which failure is caused by the elastic energy associated with shear deformation. A. Maximum-shear-stress theory C. Maximum-normal-stress theory B. Distorsion-energy theory D. Internal friction theory

View more...

Comments

Copyright ©2017 KUPDF Inc.
SUPPORT KUPDF